You are on page 1of 155

Group Theory- I Notes

IIT JAM Mathematics

IIT JAM : Group Theory- I Notes |

Preliminaries

The basics of set theory : sets, ∩, U, ∈, etc. should be familiar to


the reader. Our notation for subsets of a given set A will be
B = {a ∈ A I . . . (conditions on a) . . . }.
The order or cardinality of a set A will be denoted by IAl. If A is a
finite set, the order of A is simply the number of elements of A.
It is important to understand how to test whether a particular x ∈ A
lies in a subset B of A. The Cartesian product of two sets A and B
is the collection A x B = {(a, b) | a ∈ A, b ∈ B}, of ordered pairs of
elements from A and B.
We shall use the following notation for some common sets of
numbers: 

1. 𝕫 = {0, ±1, ±2, ±3, ...} denotes the integers (the 𝕫 is for the
German word for numbers: “Zahlen”). 
2. 𝕢 = {a/b I a, b ∈ Z and b ≠ 0} denotes the rational numbers
(or rationales). 
3. 𝕣 = {all decimal expansions ± d 1d2 ... dn.a1a2a3 ...} denotes
the real numbers (or reals). 
4. 𝕔 = {a + bi I a, b ∈ 𝕣, i2 = -1} denotes the complex numbers. 
5. 𝕫 +, 𝕢+ and 𝕣 + will denote the positive (nonzero) elements in
Z, Q and R, respectively.

We shall use the notation f : A → B or A   B to denote a


function f from A to B and the value of f at a is denoted f(a) (i.e., we
shall apply all our functions on the left). We use the words function
and map interchangeably. The set A is called the domain of f and B
is called the codomain of f. The notation f : a → b or a → b if f is
understood indicates that f(a) = b, i.e., the function is being
specified on elements.
The set f(A) = {b ∈ B I b = f(a), for some a ∈ A} is a subset of B,
called the range or image of f (or the image of A under f). For each
subset C of B the set f -1(C) = {a ∈ A I f(a) ∈ C} consisting of the
elements of A mapping into C under f is called the preimage or
inverse image of C under f. For each b ∈ B, the preimage of {b}
under f is called the fiber of f over b. Note that f -1 is not in general a
function and that the fibers of f generally contain many elements
since there may be many elements of A mapping to the element b.

If f : A → B and g : B → C, then the composite map g of : A → C is


defined by (g o f)(a) = g(f(a)).
Let f : A → B.

1. f is injective or is an injection if whenever a 1 ≠ a2, then f(a1) ≠


f(a2).
2. f is surjective or is a surjection if for all b ∈ B there is some a
∈ A such that f(a) = b, i.e., the image of f is all of B. Note that
since a function always maps onto its range (by definition) it
is necessary to specify the codomain B.
3. f is bijective or is a bijection if it is both injective and
surjective. If such a bijection f exists from A to B, we say A
and B are in bijective correspondence.
4. f has a left inverse if there is a function g : B → A such that g
o f : A → A is the identity map on A, i.e., (g o f)(a) = a, for all
a ∈ A.
5. f has right inverse if there is a function h : B → A such that f
o h : B → B is the identity map on B.

Proposition : Let f : A → B.

1. The map f is injective if and only if it has a left inverse.


2. The map f is surjective if and only if f has a right inverse.
3. The map f is a bijection if and only if there exists g : B → A
such that (f o g) is the identity map on B and (g o f) is the
identity map on A.
4. If A and B are finite sets with the same number of elements
(i.e., IAI = IBI), then f : A → B is bijective if and only if f is
injective if and only if f is surjective.

A permutation (rearrangement) of a set A is simply a bijection from


A to itself.
If A ⊆ B and f : B → C, we denote the restriction of f to A by fl A.
When the domain we are considering is understood we shall
occasionally denote fl A again simply as f even though these are
formally different functions (their domains are different).
If A ⊆ B and g : A → C and there is a function f : B C such that fl A =
g, we shall say f is an extension of g to B (such a map f need not
exist nor be unique).

Let A be a nonempty set.

1. A binary relation on set A is a subset 𝕣 of A * A and we write a


~ b if (a, b) ∈ 𝕣.
2. The relation ∼ on A is said to be :
(a) reflexive if a ~ a, for all a ∈ A,
(b) symmetric if a ~ b implies b ~ a for all a, b ∈ A,
(c) transitive if a ~ b and b ~ c implies a ~ c for all a, b, c ∈ A.
A relation is an equivalence relation if it is reflexive, symmetric
and transitive.
3. If ~ defines an equivalence relation on A, then the
equivalence class of a ∈ A is defined to be {x ∈ A I x ~ a}.
Elements of the equivalence class of a are said to be
equivalent to a. If C is an equivalence class, any element of C
is called a representative of the class C.
4. A partition of A is any collection {A( I i ∈ I} of nonempty
subsets of A (I some indexing set) such that
(a) A = U i∈I Ai, and
(b) Ai ∩ A = ∅, for all i, j ∈ I with i ≠ j
i.e., A is the disjoint union of the sets in the partition.
The notations of an equivalence relation on A and a partition
of A are the same:

Proposition : Let A be a nonempty set.

1. If - defines an equivalence relation on A then the set of


equivalence classes of ~ form a partition of A.
2. If {Ai I i ∈ I} is a partition of A then there is an equivalence
relation on A whose equivalence classes are precisely the
sets Ai, i ∈ I.

Properties of The Integers

The following properties of the integers Z (many familiar from


elementary arithmetic)

1. (Well Ordering of 𝕫) If A is any nonempty subset of 𝕫 +, there is


some element m ∈ A such that m ≤ a, for all a ∈ A (m is called
a minimal element of A).
2. If a, b ∈ 𝕫 with a ≠ 0, we say a divides b if there is an element
c ∈ 𝕫 such that b = ac. In this case we write a I b; if a does not
divide b we write  .
3. If a, b ∈ 𝕫 - {0}, there is a unique positive integer d, called the
greatest common divisor of a and b (or g.c.d. of a and b),
satisfying:
(a) d I a and d I b (so d is a common divisor of a and b), and
(b) if e I a and e I b, then e I d (so d is the greatest such
divisor).
The g.c.d. of a and b will be denoted by (a, b). If (a, b) = 1, we
say that a and b are relatively prime.
4. The a, b ∈ 𝕫 - {0}, there is a unique positive integer l, called
the least common multiple of a and b (or l.c.m. of a and b),
satisfying:
(a) a | l and b | l (so l is a common multiple of a and b), and
(b) if a | m and b | m, then l | m (so l is the least such
multiple).
The connection between the greatest common divisor d and
the least common multiple l of two integers a and b is given dl
= ab.
5. The Division Algorithm: if a, b ∈ 𝕫 - {0}, then there exist
unique q, r ∈ 𝕫  such that a = qb + r and 0 < r < |b|,
6. The Euclidean Algorithm is an important procedure which
produces a greatest common divisor of two integers a and b
by iterating the Division Algorithm: if a, b ∈ 𝕫 - {0}, then we
obtain a sequence of quotients and remainders
a = q0b + r0 ...(0)
b = q1r0 + r1 ...(1)
ro = q2r1 + r2 ...(2)
r1 = q3r2 + r3 ...(3)

rn-2 = qnrn-1 + rn ...(n)


rn-1 = qn+1rn ...(n+1)

where rn is the last nonzero remainder. Such an r n exists since |b| >
|r0| > |r1| > ... > Ir nl is a decreasing sequence of strictly positive
integers if the remainders are nonzero and such a sequence
cannot continue indefinitely. Then r n is the g.c.d. (a, b) of a and b.

Example : Suppose a = 57970 and b = 10353. Then applying the


Euclidean Algorithm we obtain:

57970 = (5)10353 + 6205


10353 = (1)6205 + 4148
6205 = (1)4148 + 2057
4148 = (2)2057 + 34
2057 = (60)34 + 17
34 = (2)17
which shows that (57970, 10353) = 17.

Modular Arithmetic

Another application of the division algorithm that will be important


to us is modular arithmetic. Modular arithmetic is an abstraction of
a method of counting that you often use.
If a and b are integers and n is a positive integer, we often write a =
b mod n whenever n divides a - b.
(ab) mod n = ((a mod n)(b mod n)) mod n.
Similarly, (a + b) mod n = ((17 mod 10) + (23 mod 10)) and 10
Examples (7 + 3) mod 10 = 10 mod 10 = 0,
(17 • 23) and 10 = ((17 mod 10)(23 mod 10)) mod 10
(7 • 3) mod 10 = 21 mod 10 = 1.

Mathematical Induction 

 First Principle of Mathematical Induction


Let S be a set of integers containing a. Suppose S has the
property that whenever some integer n ≥ a belongs to S,
then the integer n + 1 also belongs to S. Then, S contains
every integer greater than or equal to a.
 Second Principle of Mathematical Induction
Let S be a set of integers containing a. Suppose S has the
property that n belongs to S whenever every integer less
than n and greater than or equal to a belongs to S. Then, S
contains every integer greater than or equal to a.
 Definition Partition
A partition of a set S is a collection of nonempty disjoint
subsets of S whose union is S. Figure illustrates a partition
of a set into four subsets.

 Equivalence Classes Partition


The equivalence classes of an equivalence relation on a set
S constitute a partition of S. Conversely, for any partition P
of S, there is an equivalence relation on S whose
equivalence classes are the elements of P.
Proof: Let the equivalence relation on A be denoted by ~
and cl(a) denotes equivalence class of a ∈ A.
We first note that since for any a ∈ A, a ~ a, a must be in
cl(a), whence the union of the cl(a)’s is all of A. We now
assert that given two equivalence classes they are either
equal or disjoint. For, suppose that cl(a) and cl(b) are not
disjoint; then there is an element x ∈ cl(a) ∩ cl(b). Since x ∈
cl (a), a ~ x; since x ∈ cl (b), b ~ x, whence by the symmetry
of the relation, x ~ b. However, a - x and x ~ b by the
transitivity of the relation forces a - b. Suppose, now that y ∈
cl(b); thus b ~y. However, from a ~ b and b ~y, we deduce
that a ~y, that is, that y ∈ cl(a). Therefore, every element in
cl(b) is in cl(a), which proves that cl(b) ⊆ cl(a). The argument
is clearly symmetric, whence we conclude that cl(a) ⊆ cl(b).
The two opposite containing relations imply that cl(a) = cl(b).
We have thus shown that the distinct cl(a)’s are mutually
disjoint and that their union is A. This proves the first half of
the theorem. Now for the other half!
Suppose that A = U A a where the Aa are mutually disjoint,
nonempty sets (a is in some index set T). How shall we use
them to define an equivalence relation? The way is clear;
given an element a in A it is in exactly one A a. We define for
a, b å A, a ~ b if a and b are in the same A a. We leave it as
an exercise to prove that this is an equivalence relation on A
and that the distinct equivalence classes are the A a s.

Definition

1. A binary operation * on a set G is a function * : G * G → G.


For any a, b ∈ G we shall write a * b for *(a, b).
2. A binary operation * on a set G is associative if for all a, b, c ∈
G we have a * (b * c) = (a * b) * c.
3. If * is a binary operation on a set G we say elements a and b
of G commute if a * b = b * c. We say * (or G) is commutative
if for all a, b ∈ G, a * b = b * a.

Examples :

1. + (usual addition) is a commutative binary operation on 𝕫 (or


on 𝕢, 𝕣, or 𝕔 respectively).
2. x (usual multiplication) is a commutative binary operation on 𝕫
(or on 𝕢, 𝕣, or 𝕔 respectively).
3. - (usual subtraction) is a noncommutative binary operation on
𝕫, where - (a, b) = a - b. The map a → -a is not a binary
operation (It is unary).
4. - is not a binary operation on 𝕫 + (nor 𝕢 +, 𝕣 +) because for a, b ∈
𝕫 + with a < b, a - b ∉ 𝕫 +, that is, - does not map 𝕫 + * 𝕫 + into 𝕫 +.
5. Taking the vector cross-product of two vectors in 3-space 𝕣 3 is
a binary operation which is not associative and not
commutative.

Group

Definition : 

A group is an ordered pair (G, *) where G is a set and * is a binary


operation on G satisfying the following axioms:

1. (a * b) * c = a * (b * c), for all a, b, c ∈ G, i.e., * is associative,


2. there exists an element e in G, called an identity of G, such
that for all a ∈ G we have a * e = e * a = a,
3. for each a e G there is an element a -1 of G, called an inverse
of a, such that a * a -1 = a-1 * a = e.

Examples

1. 𝕫, 𝕢, 𝕣 and 𝕔 are groups under + with e = 0 and a -1 = -a, for all
a.
2. 𝕢 - {0}, R - {0}, 𝕔 - {0}, 𝕢 +, 𝕣 + are groups under * with e = 1 and
a-1 = 1/a, for all. Note however that 𝕫 - {0} is not a group
under * because although * is an associative binary operation
on 𝕫 - {0}, the element 2 (for instance) does not have an
inverse in 𝕫 - {0}. 

Example : The set of integers Z (so denoted because the German


word for numbers is Zahlen), the set of rational numbers 𝕢 (for
quotient), and the set of real numbers 𝕣 are all groups under
ordinary addition. In each case, the identity is 0 and the inverse of
a is -a.

Example : The set of integers under ordinary multiplication is not a


group. Since the number 1 is the identity, property 3 fails. For
example, there is no integer b such that 5b = 1.

Example : The subset {1,-1, i, -i} of the complex numbers is a


group under complex multiplication. Note that -1 is its own inverse,
whereas the inverse of i is -i, and vice versa.

Example : The set 𝕢 + of positive rationals is a group under ordinary


multiplication. The inverse of any a is 1/a 5 a -1.

Example : The set S of positive irrational numbers together with 1


under multiplication satisfies the three properties given in the
definition of a group but is not a group. Indeed, V√2 • V√2 = 2, so S
is not closed under multiplication.

Example : A rectangular array of the form   is called 2 x 2


matrix. The set o f all 2 x 2 matrices with real entries is a group
under component wise addition. That is, 

The identity is   and the inverse of 

Example : The set 𝕫 n = {0, 1, ..., n - 1} for n > 1 is a group under
addition modulo n. For any j > 0 in 𝕫 n, the inverse of j is n - j.
This group is usually referred to as the group of integers modulo n.
As we have seen, the real numbers, the 2 x 2 matrices with real
entries, and the integers modulo n are all groups under the
appropriate addition.

Table : Summary of Group Examples (F can be any of Q, R, C, or


Zp; L is a reflection)
Elementary Properties of Groups

1. Theorem : Uniqueness of the Identity


In a group G, there is only one identity element.
Proof. Suppose both e and e' are identities of G. Then,
(a) ae = a for all a in G, and
(b) e’a = a for all a in G.
The choices of a = e’ in (1) and a = e in (2) yield e’e = e’ and
e’e = e. Thus, e and e’ are both equal to e’e and so they are
equal to each other.
2. Theorem : Uniqueness of the Inverse
In a group G, there is only one inverse element.
Proof : if a1 and a2 are both inverses of a, then
a1 * (a * a2) = a1* e = a1 
and (a 1 * a)*a2 = e*a2 = a2
By the associative law, a 1 * (a * a2) = (a1 * a) * a) * a2; hence a 1 =
a2. This shows that in every group, each element has exactly one
inverse.

 Theorem 1 If G is a group and a, b, c are elements of G, then


(i) ab = ac implies b = c and
(ii) ba = ca implies b = c
It is easy to see why this is true: if we multiply (on the left)
both sides of the equation ab = ac by a-1, we get b = c. In the
case of ba = ca, we multiply on the right by a-1. This is the
idea of the proof; now here is the proof :
Suppose ab = ac
Then a-1(ab) = a -1(ac)
By the associative law, (a -1a)b = (a-1a)c
that is, eb = ec
Thus, finally, b = c
Part (ii) is proved analogously.
In general, we cannot cancel a in the equation ab = ca.
 Theorem 2 If G is a group and a, b are elements of G, then
ab = e implies a = b 1 and b = a-1
The proof is very simple: if ab = e, then ab = aa -1 so by the
cancellation law, b = a -1. Analogously, a = b -1.
This theorem tells us that if the product of two elements is
equal to e, these elements are inverses of each other. In
particular, if a is the inverse of b, then b is the inverse of a.
The next theorem gives us important information about
computing inverses.
 Theorem 3 If G is a group and a, b are elements of G, then
(i) (ab) -1 = b-1a-1 and
(ii) ( a-1)-1 = a

The first formula tells us that the inverse of a product is the product
of the inverses in reverse order. The next formula tells us that a is
the inverse of the inverse of a. The proof of (i) is as follows:
(ab)(b -1a-1) = a[(bb-1)a-1] by the associative law
= a[ea-1] because bb -1 = e  
= aa-1
=e
Since the product of ab and b ’ a 1 is equal to e, it follows by
Theorem 2 that they are each other’s inverses. Thus, (ab) -1 = b-1a-1.
The proof of (ii) is analogous but simpler: aa -1 = e, so by Theorem 2
a is the inverse of a -1, that is, a= (a -1)-1.
The associative law states that the two products a(bc) and (ab)c
are equal; for this reason, no confusion can result if we denote
either of these products by writing abc (without any parentheses),
and call abc the product of these three elements in this order.
We may next define the product of any four elements a, b, c, and d
in G by abed = a(bcd)
By successive uses of the associative law we find that a(bc)d =
ab(cd) = (ab)(cd) = (ab)cd
Hence the product abed (without parentheses, but without
changing the order of its factors) is defined without ambiguity.
In general, any two products, each involving the same factors in the
same order, are equal. The net effect of the associative law is that
parentheses are redundant.
Having made this observation, we may feel free to use products of
several factors, such as a 1a2 ... an, without parentheses, whenever
it is convenient. Incidentally, by using the identity (ab) -1 = b-1a-
1
 repeatedly, we find that 
If G is a finite group, the number of elements in G is called the
order of G. It is customary to denote the order of G by the symbol |
G|.

Abelian Group

The group G is said to be abelian if a • b = b • a for all a, b ∈ G.


An abelian group is a nonempty set A with a binary operation +
defined on A such that the following conditions hold:

1. (Associativity) for all a, b, c ∈ A, we have a + (b + c) = (a + b)


+ c;
2. (Commutativity) for all a, b ∈ A, we have a + b = b + a;
3. ( Existence o f an additive identity there exists an element O ∈
A such that 0 + a = a for all a ∈ A;
4. (Existence of additive inverses) for each a ∈ A there exists an
element -a ∈ A such that -a + a = 0.

EXAMPLE OF SEMI-GROUP BUT NOT MONOID : Nonzero real


numbers under the operation * defined by a*b = |a|b.

EXAMPLE OF MONOID BUT NOT GROUP : {0,1} under


multiplication.

Non-Abelian Group

In mathematics, a nonabelian group, also sometimes called a


noncommutative group, is a group (G, *) in which there are at least
two elements a and b of G such that a * b ≠ b * a. The term
nonabelian is used to distinguish from the idea of an abelian group,
where all of the elements of the group commute.

Example : Show that the set {1, - 1, i, - 1} where i =   is a


finite abelian group for multiplication of complex numbers.

G = {1, -1, i -i}.


We prepare the composition table for (G, x) as follows :
On observing the table, it is clear that :

1. All the entire in the table are elements of G. So


multiplication of complex numbers has induced a binary
composition in G.
2. The number 1 is the identity element for the multiplication
composition.
3. The inverses of 1, - 1 , i, - i are 1, - 1 , - i and i
respectively.

Since the associativity and commutativity of the multiplication


of numbers is obvious, so G is a finite (of order 4) abelian
group for multiplication.

Example : Show that the set {z ∈ C : |z| = 1} is a multiplicative


group.

Let G = {z ∈ C : |z| = 1} and z 1, z2 ∈ G; then


z1, z2 ∈ G ⇒ |z1| = 1, |z2| = 1
⇒ |z1||z2| = 1.1 = 1
⇒ |z1,z2| = 1 [∵ |z1z2| = |z1||z2|]
⇒ z1, z2 ∈ G
∴ G is closed for multiplication.

Verification of group axioms in (G, x) :

[G1] Since multiplication operation is associative in numbers,


therefore this operation will also be associative in G.
[G2] ∵ |1| = 1 ⇒ 1 ∈ G, which is the identity for multiplication.
[G3] For any element z of G
z ∈ G ⇒ |z| = 1
⇒ 
which is the multiplicative inverse of z.
Thus there exist inverse of every element in G.
Hence G is a multiplicative group.

Example : Prove that the set of n, nth roots of unity is a


multiplicative finite abelian group.

Here 11/n = (1 + i0) 1/n = (cos 0 + i sin 0) 1/n


= (cos 2rπ + i sin 2rπ) 1/na, where r is any integer

 [by DeMoivre’s theorem]

= ei(2rπ)/n r = 0, 1,2, .... (n - 1)


If we take e i(2rπ)/n = θr, then the set of n complex roots
G = {1, θ, θ2, θ3, .... θn-1}
If a, b ∈ G, then an = 1 and bn = 1
Now (ab)n = an bn = 1.1 = 1
∴ ab is also the nth root of unity, therefore ab ∈ G
i.e., a, b ∈ G ⇒ ab ∈ G
Hence G is closed for multiplication.
[G1] Associativity : Complex roots are associative for
multiplication.
[G2] Identity element 1 for multiplication is in G.
[G3] If any element θr, 0 ≤ r ≤ (n - 1) is in G, then there exist an
element θn-r in G such that θ n-r•θr = θn = i (identity) [∵ 0 is the
nth root of 1]
i.e., there exist inverse θ n-r of θr
[G4] Commutativity : Complex numbers are commutative for
multiplication. Hence (G, x) is a finite abelian group of order n.

Example : Show that Z5 = {0, 1, 2, 3, 4} is an abelian group for


the operation *+ 5’ defined as follows :
The composition table of (Z 5, +5) is as follows :

From the table it is observed that

1. All the elements are members of Z 5 therefore ‘+5’ is a


binary composition in Z 5.
2. 0 is the identity element for the composition.
3. The inverse of 0, 1, 2, 3, 4 are 0, 4, 3, 2, and 1
respectively.

Again the base of '+ 5’ is the addition composition of numbers


which is associative and commutative. Therefore '+ 3’ is also
associative and commutative.
Hence (Z5,+5) is a commutative group.

Example : Show that the set {f 1, f2, f3, f4, f5, f6} is a finite non-
abelian group for the operation “composite of functions”
where fi, i = 1, 2,..., 6 are transformations on the infinite
complex plane defined by :

Let G = {f 1, f2, f3, f4, f5, f6} and o denote composite of functions.
Now calculating all the possible products under this operation,
we have the following:
From these we obtain the following composition table of (G,
o) :

The group axioms can be easily verified from the above


composition table. There fore (G, o) is a finite group.
More over since the table is not symmetrical about the leading
diagonal so it is a non commutative finite group.

Example : Show that the set Q + of the positive relational


numbers forms an abelian group for the operation * defined
as :
a * b = ab/2  

∴ * is a binary composition in Q +
Verification of group axioms in (Q +, *) :
[G1] Associativity : Let a, b ∈ Q+, then
and 
∴ (a *b )*c = a *(b *c)
[G2] Existence of identity :
2 ∈ Q+ is the identity element of the operation *
because for a ∈ Q+, 2*a = a*2 = a.2/2 = a
[G3] Existence of Inverse :
For every a ∈ Q+, (4/a) ∈ Q+ (∵ a ≠ 0) which is the inverse of a

Since 
So every element of Q + is invertible.
[G4] Commutativity : For any a, b ∈ Q+

 [∵ multiplication is commutative]


= b*a
∴ the composition * is commutative.
Hence (Q+, *) is an abelian group.

Example : If G = {(a, b) | a, b ∈ R, a ≠ 0} and o is the operation


defined in G as (a, b) o (c, d) = (ac, be + d) ; then show that (G,
o) is a non-abelian group.

Verification of group axioms in (G, o) :


[G1] : Let (a, b); (c, d); (e, f) be any elements of G, then [(a, b)
o (c, d)] o (e, f) = (ac, be + d) o (e, f)
= ((ac)e, (be + d)e + f)
= (ace, bee + de + f)
and (a, b) o [(c, d) o (e, f)] = (a, b) o (ce, de + f)
= (a(ce), b(ce) + de + f)
= (ace, bee + de + f)
∴ [(a, b) o (c, d)] o (e, f) = (a, b) o [(c, d) o (e, f)]
Therefore o is associative.
[G2] : Here (1, 0) ∈ G is the identity element of the operation
because for every (a, b) ∈ G (1, 0) o (a, b) = (1a, 0a + b) = (a,
b)
and (a, b) o (1, 0) = (a1, b1 + 0) = (a, b)
[G3] : If (a, b) ∈ G, then a ≠ 0

∴ 

Again 

and 

∴   is the inverse of (a, b)


Thus the inverse of every element of G also exist in G.
Hence (G, o) is a group.
[G4] : The composition of the group is not commutative
because if a, b, c, d are different real numbers, then (a, b) o
(c, d) = (ac, bc + d)
≡ (ca, da + b) = (c, d) o (a, d)
⇒ (a, b) o (c, d) ≠ (c, d) o (a, b)
As such (G, o) is a non abelian group.

Example : Show that G =   is a


commutative group under matrix multiplication.

Let   be any two elements of G, where a, b ∈

R0; then AB =  ∵ a ∈ R0, b ∈ R0 ⇒ ab ∈ R0


∴ AB = G which shows that G is closed for matrix
multiplication.
Verification of group axioms in (G, x) :
[G1] : Since Matrix multiplication is associative.
Therefore this is associative in G also.

[G2] : E =   is the identity element because


[G3] : For every   (∵ a ∈ R0 ⇒ 1/a
∈ R0)
which is inverse of A because

 (Identity)

[G4] : Since AB =   and ab = ba (by


commutativity of multiplication of real numbers),
Therefore, AB = BA
As such matrix multiplication is commutative in G.
Hence G is a commutative group for matrix multiplication.

Example : Show that the set of all the matrices of the

form   is an abelian group for matrix


multiplication.

Let 
and Aα, Aβ, ∈ G, where α, β ∈ R, then,

= Aα+β ∈ G [∵ α + β ∈ R]
∴ 
Consequently G is closed for matrix multiplication.
Verification of group axioms in (G, x) :
[G1] : Matrix multiplication is associative.
Therefore, this is associative in G also.
[G2]: For α = 0

which is the identity for matrix multiplication.


[G3]: For every Aα ∈ G, A(-α), ∈ G which is the inverse of A α,
because Aα x A(-α) = Aα-α = A0 and A(-α) x Aα = A(-α + α) = A0
Therefore, the inverse of every element of G exists in G.
[G4]: For any two elements A α and Aβ of G
Aα x Aβ = Aα+β [proved above]
= Aβ+α [∵ α, β ∈ R ⇒ α + β = β + α]
Therefore, matrix multiplication is commutative in G.
As such G is a commutative group for matrix multiplication.

Example : The set G = {1, 2, 3, 4...(p - 1)}, x p. p being prime is


an abelian group of order (p - 1) with respect to multiplication
modulo p.

Let a, b ∈ G, then 1 ≤ a ≤ (p - 1), 1 ≤ b ≤ (p - 1)


By definition, a x P b = r, where r is the least non negative
remainder when ordinary product is divided by p.
Now since p is prime, therefore ab is not divisible by p
Consequently, r ≠ 0, 1 ≤ r ≤ (p - 1)
therefore a x p b ∈ G ∀ a, b ∈ G
⇒ G is closed for the multiplication modulo p.
[G1] : Associativity : Let a, b, c ∈ G then
a xp (b xp c) = a xp (bc) [∵ b xp c = be (mod p)]
= the least non-negative remainder obtained on dividing a(bc)
by p
= the least non-negative remainder obtained on dividing (ab)c
by p
= (ab) xp c = (a xp b) xp c [∵ ab = a xp b(mod p)]
Therefore multiplication modulo p is associative.
[G2] : 1 ∈ G is the identity element because for every a ∈ G
1 xp a = a xp 1 = a
[G3] : Let s ∈ G then 1 ≤ s ≤ (p - 1).
Consider the following (p - 1) products :
1 xp s, 2 xp s, 3 xp s, ... (p - 1) x p s
All these are elements of G and no two of these are equal
since if 1 ≤ i ≤ (p - 1), 1 ≤ j ≤ (p - 1) and i > j, then  i xp s = j xp s
⇒ (i s) and (j s) leave the same non-negative remainders when
divided by p
⇒ (i s - j s) is divisible by p
⇒ (i - j)s is divisible by p
But this is not possible because
1 ≤ (i - j) ≤ (p - 1), 1 ≤ s ≤ (p - 1) and p is prime.
∴ i xp s ≠ j xp s
and (p - 1) distinct elements of G and so one of the products
must be equal to 1.
Let r xp s = 1 ⇒ is the inverse of s
Therefore every element of G is invertible.
and (p - 1) distinct elements of G and so one of the products
must be equal to 1.
Let r xp s = 1 ⇒ r is the inverse of s
Therefore, every element of G is invertible.
[G4] : a xp b = the least non-negative remainder obtained on
dividing ab by p
= the least non-negative remainder obtained on dividing (ba)
by p  
= b xp a (∵ ba = ab)
Therefore, multiplication modulo p is commutative.
Hence G is an abelian group of order (p - 1).
Remark. If p is not prime, then G will not be a group because
G will not be closed for multiplication.

Example : The set of residue classes modulo m is an abelian


group with respect to the addition of residue classes.

Let I = { . . . , -3 , -2 , -1 , 0 , 1, 2, 3, ...} be the set of integers.


If a ∈ I, then [a] is a residue class modulo m of I if [a] = {x : x ∈
I and x - a is divisible by m}.
Let Im be the set of all residue class of I mod m
i.e., Im = {[a] : a ∈ I}. We have [a] = [b] ⇔ a = b (mod m). The
set lm has m distinct elements [0], [1], [2]......[m - 1]. Thus we
have
lm = {[0], [1], [2], .... [m - 1]}.
Let a and b are any two integers, then we define the addition
of residue classes [a] and [b] as follows :
[a] + [b] = [a + b].
If [a] = [c] and [b] = [d], then it can be easily seen that [a] + [b]
= [c] + [d].
Therefore, our addition of residue classes is well-defined.
Now we shall show that l m is a group with respect to addition of
residue classes.

Closure Property : 

If [a], [b] ∈ lm, then by definition [a] + [b] = [a + b]. Since a + b is an


integer, therefore [a + b] ∈ lm. Thus l m is closed with respect to
addition of residue classes.
Associativity : Let [a], [b], [c] be any three elements of l m. Then [a]
+ ([b] + [c]).
= [a] + [b + c] [by def. of addition of residue classes]
= [a + (b + c)] [by def. of addition of residue classes]
= [(a + b) + c] [∵ addition of integers is a associativity]
= [a + b] + [c] = ([a] + ([b] + [c].
Commutativity : Let [a], [b] ∈ lm. Then [a] + [b] = [a + b] = [b + a] =
[b] + [a].
Existence of Identity : We have [0] ∈ Im. If [a] ∈ lm, then we have
[0] + [a] = [0 + a] = [a] = [a] + [0]. Therefore, the residue class [0] is
the identity element.
Existence of Inverse : We have [0] ∈ lm. If [a] ∈ lm, then we have
[0] + [a] = [0 + a] = [0 + a] = [a] + [0]. Therefore, the residue class
[0] is the identity element.
Existence of Inverse : Let [a] ∈ lm be arbitrary. Since a ∈ I - a ∈ I,
therefore [-a] is also an element of l m. We have [-a] + [a] = [-a + a] =
[0] = [a] + [-a]. Thus, [-a] is the inverse of [a].
Thus l m is an abelian group with respect to addition of residue
classes.
Since the number of distinct elements in l m is m, therefore the order
of this group is m,
Note : If [r] = lm and 0 ≤ r < m, then the inverse of [r] is [m - r]. We
have [r] + [m - r] = [r + (m - r)] = [m] = [0].
Note that m I m ⇒ [m] = [0].

Elementary Properties of Groups

1. Uniqueness of the Identity


In a group G, there is only one identity element.
Proof. Suppose both e and e’ are identities of G. Then,
1. ae = a for all a in G, and
2. e’a = a for all a in G.

The choice of a = e’ in (1) and a = e in (2) yields e’ e = e’ and e’ e =


e. Thus, e and e’ are both equal to e’ e and so are equal to each
other.

Theorem : (Uniqueness of inverse)


The inverse of an element in a group is unique.

Proof. Let a be any element of the group (G, *) which has two


inverses b and c in the group.
a-1 = b ⇒ ba = e = ab
and a-1 = c ⇒ ca = e = ac
Now ba = e ⇒ (ba) c = ec
⇒ b(ac) = c [by G 1 and G2]
⇒ be = c [by (2)]
⇒ b = c [by G3]
Therefore, the inverse of every element of a group is unique.

Remark. The inverse of the identity of a group is itself.

Theorem : If G is a group then for a, b ∈ G:


(a) (a -1)-1 = a (b) (ab) -1 = b-1 a-1
i.e. the inverse of the produced of two elements is the product of
their inverses in the reverse order.
Proof, (a) Since a-1 is the inverse of a, therefore aa -1 = e = a-1 a
⇒ a-1 a = e = aa -1
⇒ inverse of a -1 = a, i.e. (a-1)-1 = a.

Remark. For the additive operation - (-a) = a.

(b) Since a, b, a-1, b-1, ab, b-1 a-1 all are element of G, therefore


(ab)(b -1a-1) = a(bb -1)a-1  [by G1]
= aea-1 [by G3]
= aa-1 [by G2]
=e
∴ (ab)(b -1a-1) = e ...(1)
Again (b-1a-1)(ab) = b-1(a-1 a)b
= b-1eb  [by G1]
= b-1b [by G3]
= e [by G 2]
∴ (b-1a-1)(ab) = e ...(2)
From (1) and (2),
(ab)(b -1a-1) = e = (b-1a-1)(ab)
⇒ (ab) -1 = b-1 a-1

Generalised reversal law :

By principle of induction, the above theorem can be generalised


as :
(a1a2a3...an)-1 =

Remark : If the composition is addition (+) then this can be written
as :
-(a + b) = (-b) + (-a)

Remark : If G is a commutative group, then for a, b ∈ G


(ab) -1 = a-1 b-1

Theorem : Cancellation law :


If a, b, c are elements of a group G, then :
(a) ab = ac ⇒ b = c
(b) ba = ca ⇒ b = c
Proof. ∵ a ∈ G ⇒ a-1 ∈ G [by G 3]
∴ ab = ac ⇒ a-1 (ab) = a -1 (ac)
⇒ (a-1 a)b = (a-1 a)c [by G 1]
⇒ eb = ec [by G 3]
⇒ b = c [by G2]
Similarly, it can be proved that ba = ca ⇒ b = c

Theorem : If a, b are elements of a group G, then the equations ax


= b and ya = b have unique solutions in G.
Proof. ∵ a ∈ G ⇒ a-1 ∈ G [by G 3]
∴ a ∈ G, b ∈ G ⇒ a-1b ∈ G
Now a(a -1 b) = (aa -1)b [by G 1]
= eb [by G3]
=b
Therefore, x = a -1 b is a solution of the equation ax = b in G.

Uniqueness : Let the equation ax = b have two solutions x =


x1 and x = x2 in G, then ax 1 = b and ax 2 = b
⇒ ax1 = ax2
⇒ x1 = x2 [by left cancellation law]
Therefore, the solution of ax = b is unique in G.
Similarly, its can also be proved that the solution of the equation ya
= b is unique in G. 

Theorem : Alternate definition of a group :


If for all elements a, b of a semigroup G, equations ax = b and ya =
b have unique solutions in G, then G is a group.

Proof. G being semi group, is a non empty set. Therefore let a ∈ G,


then the equations ax = a and ya = a will have unique solutions in
G. Let these solutions be denoted by e 1 and e2 respectively, then
ae1 = a and e2a = a ...(i)
Again if b be any other element of g, then by the given property
x, y ∈ G so that ax = b and ya = b ...(ii)
Now ya = b ⇒ (ya)e1 = be1
⇒ y(ae1) = be1 [by associativity in G]
⇒ ya = be 1 [by (i)]
⇒ b = be 1 [by (ii)]
∴ e1 is the right identity in G.
Again ax = b ⇒ e2(ax) = e2b
⇒ (e2a)x = e2b [by associativity in G]
⇒ ax = e 2b [by (i)]
⇒ b = e2b [by (ii)]
∴ e2 is the left identity in G.
Since e 1 is right identity in G and e 2 is in G ⇒ e2e1 = e2
Also e2 is left identity in G and e 1 is in G ⇒ e2e1 = e1 ⇒ e1 = e1
Hence there exists an identity e 1 = e2 = e (say) in G.
Again using the given property for the elements a, e e G we find
that the equations ax = e and ya = e have unique solutions in G.
Let these solutions be x a and ya respectively. So
axa = e and yaa = e ...(iii)
⇒ xa and ya are right and left inverses of a in G.
Now axa = e ⇒ ya(axa) = yae
⇒ (yaa)xa = ya [by associativity in G]
⇒ exa = ya [by (iii)]
⇒ xa = ya
∴ there exist the inverse of a in G.
Since the identity exist and every element of G is invertible in the
semigroup G, therefore G is a group.

Remark. If G is a semi group such that for a, b in G, only the


equation ax = b (or ya = b) has a unique solution in G, then G may
not be a group.
This can be observed by the following example :

Example : Let G be any non-empty set having at-least two


elements.
Define a binary operation (*) in G as follows :
a*b = b ∀ a, b ∈ G  
∵ For a, b, c ∈ G (a*b)*c = b*c = c
and a*(b*c) = a*c = c
∴ (a*b)*c = a*(b*c)
Therefore the composition * of G is associative.
Hence (G, *) is a semi group.
Again we see that a * b = b ⇒ x = b i s a solution of ax = b in G.
But trivially (G, *) is not a group as there is a no right identity in G.

Theorem : If G is finite semigroup such that for any a, b, c ∈ G


(a) ab = ac ⇒ b = c
and (b) ba = ca ⇒ b = c
then G is a group.

Proof. Since G is a non void finite set, so let


G = {a1, a2, a3......an}
If a ∈ G, then the n products aa 1, aa2, ..., aa n ...(1)
are all elements of G (∵ G is a semi group)
Again all these are distinct elements because
if aai = aaj, then by the given property
aai, = aaj ⇒ ai = aj
So all elements of (1) are n distinct elements of G, placed possibly
in a different order i.e. G = {aa 1, aa2......aa n}
Now if b ∈ G, then b is one of the above n products.
Therefore, let aa r = b, ar ∈ G
Hence we see that for any a, b ∈ G, the equation ax = b has a
unique solution in G.
Similarly by considering the n products a 1a,a2a....ana it can be
shown that for every pair a, b ∈ G, the equation ya = h has a
unique solution in G. Hence by theorem G is a group.

Remark : If G is an infinite semi group satisfying cancellation law


but it is not a group. 

Example : The positive integers under multiplication form a


cancellative semigroup, but is not a group.

Theorem : Definition of a group based on left axioms.


A finite semigroup G is a group iff :
(i) there exists e ∈ G such that ea = a, ∀ a ∈ G (Left Identity)
(ii) there exists b ∈ G such that ba = e, ∀ a ∈ G (Left Inverse)
Proof. If G is a group, then by the definition of a group the above
properties (i) and (ii) must obviously hold true.
Conversely, if a semi group G satisfies both the properties (i) and
(ii), then we have to prove that G is a group. For this we have to
show that e is the identity in G and b is the inverse of a i.e.
ea = a = ae, ∀ a ∈ G
and ba = e = ab
Let a ∈ G, then by the given property (ii), there exists an element b
such that
ba = e ...(1)
Again by the same property there exists c such that
cb = e ...(2)
Now for a, b ∈ G ab = e(ab) [by property (i)]
= (cb)(ab) [by (2)]
= c(ba)b [by associativity in G]
= (ce)b [by property (ii)]
= c(eb)
= cb  [by property (i)]
= e [by (2)]
∴ ba = e = ab which prove that b is the inverse of a.
Further, we see that ae = a(ba) [∵ e = ab]
= (ab)a  [by associativity in G]
= ea [∵ ab = e]
= a [by property (i)]
∵ ea = a = ea which proves that e is the identity in G.
Therefore, G is a group.

Theorem : Definition of a group based on right axioms :


A semigroup G is group iff :
(i) ∃ e ∈ G such that ae = a, ∀ a ∈ G
(ii) ∃ b ∈ G such that ab = e, ∀ a ∈ G
This can be proved similar to the previous theorem.

Caution : It should be noted that a semi group G may be such that
G contains a left identity e and each element a in G has a right
inverse related to e. In such a case G may fail to be a group. This
will be clear by the following example :

Example : Let G be a set containing atleast two elements.


Define a composition (•) in G as follows :
a • b = b, ∀ a, b ∈ G
Obviously G is a semi group. Let e be any fixed element of G. Then
by definition, for every a ∈ G, ea = a. So e is a identity of a.
Also for each a ∈ G, ae = e ⇒ e is a right inverse of a wrt e.
But G is not a group as G does not have any right identity.

Finite and Infinite group


A group (G, *) is said to be finite if its underlying set G is a finite set
and a group which is not finite is called an infinite group.

Order of a Group
The number of elements of a group (finite or infinite) is called its
order. We will use IGI to denote the order of G.
Thus, the group Z of integers under addition has infinite order,
whereas the group U(10) ={1, 3, 7, 9} under multiplication modulo
10 has order 4.

Integral powers of an element in a group

Let (G, *) be a group and a ∈ G, then a*a, a*a*a, a*a*a, are all
elements of G and shall be written as a 2, a3, a4 respectively.
Thus (a) for any positive integer n,
an = a*a*a*...*a (n times)

(b) If n is a negative integer : Let n = -n where m is a positive


integer, then a" can be defined as follows :
an = a-m = (a-1)m = a-1 * a-1 *...* a-1 (m times)
Therefore a -2 = a-1 * a-1, a-3 = a-1 * a-1 * a-1 etc.
More over we define a 0 = e, ∀ a ∈ G where e is the identity in G.

Law of indices : If a ∈ G and m, n are integers, then by


mathematical induction, the following laws can be easily
established :
(a) am + n = am an (b) (am)n = an m
Note. When the composition of a group is addition (+), then a n is
written as n and in that case if m and n are positive integers, then
na = a + a + ... + a (n times)
(-m)a = (-a) +...+ (-a) (m times)
(m + n)a = ma + na
and n(ma) = (nm)a

Order of an Element
The order of an element g in a group G is the smallest positive
integer n such that g n = e. (In additive notation, this would be ng =
0). If no such integer exists, we say g has infinite order. The order
of an element g is denoted by Igl.
So, to find the order of a group element g, we need only compute
the sequence of products g, g 2, g3,..., until you reach the identity for
the first time. The exponent of this product (or coefficient if the
operation is addition) is the order of g. If the identity never appears
in the sequence, then g has infinite order.

Remark. From the above definition, it is clear that for any element


a of a group if a n = e then 0(a) ≤ n.

Example : In the group (Z,+), the order of 0 is 1 and the order of
every non zero integer a is infinite, because for any a ∈ Z(a ≠ 0)
there exists no positive integer n such that na = 0.

Example : In the group (Q 0, *), O(1) = 1, O(-1) = 2 and order of all
the remaining elements is infinite.

Example : In the multiplicative group {1, ω, ω 2} (ω3 = 1)


0(1) = 1, O((ω) = 3, 0(ω 2) = 3.

Example : In the multiplicative group { 1 , - 1 , i, - i } since 1 1 = 1, (-


1)2 = 1, i4 = 1, and (-i) 4 = 1 so O(1) = 1, O(-1) = 2, O(i) = 4 and O(-i)
= 4.

Example : In the group [{0, 1, 2, 3}, + 4]

Here 0 is the identity element


0 +4 0 = 0 ⇒ O(0) = 1
1 +4 0 = 1, 1 + 4 1 = 2, 1 + 4 2 = 3, 1 + 4 3 = 0 → O(1) = 4
2 +4 0 = 2, 2 + 4 1 = 3, 2 + 4 2 = 0 ⇒ O(2) = 3
3 +4 0 = 3, 3 + 4 1 = 0 ⇒ O(3) = 2

Example : In Klein’s 4 group every element except identity is of


order 2.

Properties of the order of an element of a group


Theorem : The order of the identity of every group is 1 and it is the
only element of order 1.
Proof. Let e be the identity of any group G.
∵ e1 = e, ∴ O(e) = 1
Again if a ∈ G and O(a) = 1, then
O(a) = 1 ⇒ a1 = e ⇒ a = e
Therefore O(a) = 1 ⇒ a = e

Theorem : The order of every element of a finite group is finite and


less than or equal to the order of the group i.e.,
O(a) ≤ O(G), ∀ a ∈ G
Proof. Let G be a finite group of order n and a ∈ G.
We see that a 0, a1, a2, .... an are all elements of G But G contains n
elements and the number of these elements is (n + 1), so all can
not be distinct. Therefore atleast two of these elements are equal.
Let ai = ai, 0 ≤ j ≤ i ≤ n  
⇒ aia-j = aj a-j
⇒ ai-j = a0 = e
⇒ ar = e, where 0 < r = i - j ≤ n
⇒ O(a) ≤ r ≤ n = O(G)
Hence O(a) is also finite and less than or equal to O(G).

Remark : The above theorem can also be stated as follows :


If G is a finite group of order n, then for any a ∈ G, there exists a
positive integer r ≤ n such that a r = e.
Theorem : If order of an element a of a group (G, *) is n, then a m =
e, iff m is a multiple of n.
Proof. First of all, suppose that a m = e 
Now since m and n are integers, so by division algorithm in integer,
there exist integers q and r such that
m = nq + r, 0 ≤ r < n
∴ am = e ⇒ anq+r = e ⇒ anq ar = e
⇒ (an)q ar = e [∵ (am)n = amn]
⇒ eq-ar = e [∵ O(a) = n ⇒ an = e]
⇒ ar = e ⇒ r = 0 [∵ 0 ≤ r < n]
⇒ m = nq i.e., m is a multiple of n.

Conversely : Let m be a multiple of n i.e. m = nq(q   then m =


m nq n q q
nq ⇒ a  = a  = (a )  = e  = e
Therefore, a m = e ⇔ m is a multiple of O(a).
Cor. : The order of any integral power of an element a of a group
(G,*) can not exceed the order of the element.
Proof. Let a be any element of a group G and O(a) = n.
Then for any k ∈ 
(ak)n = ank = (an)k = ek = e
⇒ O(ak) ≤ n ⇒ O(ak) ≤ O(a)
Cor. : The order of an element a of a group (G, *) is equal to that of
its inverse a -1 i.e., O(a) = O(a -1).
Proof. Let O(a) = n and O(a -1) = m
Now a-1 is an integral power of a ⇒ 0(a-1) ≤ O(a)
⇒ m ≤ n ...(1)
Again a = (a-1) ⇒ a is the integral power of a -1
⇒ O(a) ≤ O(a -1)
⇒ n ≤ m ...(2)
(1) and (2) ⇒ m = n, i.e. O(a -1) = O(a)
Cor. : If any element of a group G is of order n, then order of a k is
n/m where m is g.c.d. of n and k.
You have two things to show. Namely that:

1. (ak)n/gcd(n.k)  = e(where e denotes the identity)


and that n/gcd(n, k) is the smallest positive power p of a k such
that (ak)p = e:
2. For all m > 0: (a k)m = e ⇒ n/gcd(n, k) ≤ m

The first part is easy:


(ak)n/gcd(n,k)  = (an)k/gcd(n,k) = ek/gcd(n,k) = e
For the second part, let   be such that (a k)m = akm = e. Since
the order of a is n, it follows that n I km. Therefore we also

have 
Now gcd(n/gcd(n, k), k/gcd(n, k)) = 1 (try to prove this), so it follows
that n/gcd(n, k) I m
Hence in particular n/gcd(n, k) ≤ m.

Theorem : For any element a of a group G :


O(a) = O(x -1 ax), ∀ x ∈ G
Proof. Let a ∈ G, x ∈ G, then
(x-1 ax)2 = (x-1 ax)(x-1 ax)
=  x-1 (xx-1)ax [by associativity]
= x-1 aeax = x-1 (aea)x
= x-1 a2x
Again let (x -1ax)n-1 = x-1 an-1 x , where(n - 1) 
⇒ (x-1 ax)n-1 (x-1 ax) = (x-1 an-1x) (x-1 ax)
⇒ (x-1 ax)n = x-1an-1 (xx-1)ax = x-1an-1 (eax)
= x-1an-1ax = x-1 anx
Therefore, by induction method,
(x-1 ax)n = x-1 an x, 
Now let O(a) = n and O(x -1 ax) = m, then (x -1 ax)n = x-1anx = x-1 ex =
e
⇒ 0(x-1 ax) ≤ n ⇒ m ≤ n ...(1)
Again 0(x-1 ax) = m ⇒ (x-1ax)m = e ⇒ x-1 am x = e
⇒ x(x-1 amx)x-1 = xex-1 = e
⇒ (xx-1) am(xx-1) = e
⇒ eam e = e ⇒ am = e
⇒ O(a) ≤ m ⇒ n ≤ m ...(2)
(1) and (2)
⇒ n = m ⇒ O(a) = O(x -1 ax)
If O(a) is infinite, then 0(x -1 ax) will also be infinite.
Cor. : If a and b are elements of a group G; then O(ab) = O(ba)
Let n and m be the order of ab and ba, respectively. That is, (ab) n =
e, (ba) m = e,
where e is the identity element of G.
We compute
= a(ba) n-1 b.
From this, we obtain(ba) n-1 = a-1 b-1 = (ba) -1,  and thus we have
(ba) n = e.
Therefore the order m of ba divides n.
Similarly, we see that n divides m, and hence m = n.
Thus the orders of ab and ba are the same.

Proof. ∵ b-1 (ba)b = (b -1b)(ab) [by associativity]


= e(ab) = ab
Therefore by the above theorem, O(ba) = O(b -1(ba) b) = O(ab)

Theorem : If the order of an element a of a group G is n, then the


order of ap is also n provided p and n are relatively prime.
Proof. Let O(ap) = m
∵ O(a) = n ⇒ an = e ⇒ anp = eP = e
⇒ (ap)n = e ⇒ O(ap) ≤ n
⇒ m ≤ n ...(1)
Again since p and n are relatively prime, therefore GCD of p and n
=1
⇒ there exist two integers x and y such that px + ny = 1
⇒ a1 = apx+ny = apx • any
= apx .(an)y = apx.ey
= apx .e = apx
am = (apx)m = ampx = (amp)x = ex = e
⇒ O(a) ≤ m
⇒ n ≤ m ...(2)
(1) and (2) ⇒ n ≤ m ⇒ O(a) = O(ap)

Example : If for every element a of a group G, a 2 = e; then


prove that G is abelian.

Let a, b ∈ G, then by the given property  


a ∈ G ⇒ a2 = e ⇒ aa = e
⇒ a-1 = a ...(1)
Similarly b ∈ G ⇒ b-1 = b ...(2)
Now a ∈ G, b ∈ G ⇒ ab ∈ G
⇒ (ab) -1 = ab [by the given property]
⇒ b-1a-1 = ab [by reversal law]
⇒ ba = ab [by (1) and (2)]
∴ G is an abelian group.

Example : If a, b are any two elements of a group G; then show


that G is an abelian group if (ab) 2 = a2 b2.

Let G be an abelian group, then G is abelian ⇒ ab = ba, ∀ a, b


∈G
⇒ (ab)(ab) = (ba)(ab)
⇒ (ab) 2 = b(aa)b [by associativity]
⇒ (ab) 2 = (ba2)b
⇒ (ab) 2 = (a2b)b [∵ G is abelian]
⇒ (ab) 2 = a2(bb)
⇒ (ab) 2 = a2 b2

Conversely : For a, b ∈ G, (ab)2 = a2 b2, then (ab) 2 = a2 b2 ⇒


(ab) (ab) = (aa) (bb)
⇒ a(ba)b = a(ab)b [by associativity]
⇒ G is an abelian group.
Therefore G is an abelian group ⇔ (ab) 2 = a2 b2.

Example : If a is an element of a group G, then show that : a 2 =


a⇔a=e

Let a ∈ G and a2 = a then a2 = a ⇒ aa = ae


⇒ a = e [by left cancellation law]
Conversely : If a = e, then
a = e ⇒ aa = ea
⇒ a2 = a
Therefore a2 = a ⇔ a = e
Remark. For any element a of a group G, if a 2 = a, then a is
called an idempotent.

The identity element of the given group is 1, therefore O(a) =


1. For other elements, we see that
22 = 4, 23 = 3, 24 = 1 ⇒ O(2) = 4.
32 = 4, 33 = 2, 34 = 1 ⇒ O(3) = 4.
and 42 = 2 ⇒ O(4) = 2.

Example : If a, b are two elements of a group G such that


am bn = ba, ∀ m, n  ; then prove that : a mbn-2, am-2bn, ab-1 are
element of the same order in G.

ambm-2 = ambnb-2 = (ambn)b-2 = (ba)b -2


= (ba)(b -1b-1) [∵ ambn = ba]
= b(ab-1)b-1 = (b-1)-1 (ab-1) (b-1)
∴ O(ambn-2) = O(ab-1) [∴ O(a) = O(x -1 ax), ∀ x ∈ G] ...(1)
Again am-2bn = a-2 (ambn) = a-1a-1 ba [∵ ambn = ba]
am-2bn = a-1(a-1b)a
∴ O(am-2 bn) = O(a-1(a-1b)a)
= O(a-1b) [∵ O(a) = 0(x -1 ax), ∀ x ∈ G]
= O(a-1 b)-1 [∵ O(x) = 0(x -1)]
= O(b-1a) = O(eb-1 a)
= O(a-1 ab-1 a)
= O( a-1 (ab-1)a) = O(ab-1) ...(2)
From (1) and (2),
O(ambn-2) = O(am-2bn) = O(ab) -1

Example : If a is the only element of order 2 in a group G, then


show that ax = xa, ∀ x ∈ G.

We know that for any element a in a group G.


O(a) = O(x-1 ax), ∀ x ∈ G
∴ O(a) = 2 ⇒ O(x-1 ax) = 2
But a is the only element of order 2 in G, therefore
O(a) = O(x-1 ax) ⇒ a = x-1 ax
⇒ xa = x(x-1 ax)
⇒ xa = ax, ∀ x ∈ G

Example : If G is a group such that (ab) m = ambn for three


consecutive integers m, m + 1, m + 2 for all a, b ∈ G; show that
G is abelian.

a, b ∈ G, then as given
(ab)m = ambm ...(i)
(ab)m+1 = am+1 bm+1 ...(ii)
(ab)m+2 = am+2 bm+2 ...(iii)
Now (ab)m+2 = (ab)m+1 ab
⇒ am+2 bm+2 = am+1 bm+1 ab   [by (ii) and (iii)]
⇒ am+1 abm+1 b = bm+1 bm+1 ab
⇒ abm+1 = bm+1 a [by cancellation law]
⇒ am(abm+1) = am(bm+1 a)
⇒ (ama)bm+1 = am(bmb)a
⇒ am+1bm+1 = ambm(ba)
⇒ (ab) m+1 = (ab)m (ba)
⇒ (ab) m (ab) = (ba) m (ba)
⇒ ab = ba
∴ G is an abelian group.

Example : If a, b are elements of an abelian group G, then


prove that :
(ab)n = anbn, 

Case (i) When n = 0, then (ab)° = e = ee [by definition]


= a° b°
Case (ii) when n > 0; If n > 1, then (ab) 1 = ab = a1 b1
∴ The result is true for n = 1.
Let the result be true for n = k, then (ab) k = ak bk
⇒ (ab)(ab) k = (ab)(a k bk)
⇒ (ab) k+1 = a(bak)bk [by associativity]
⇒ = a(akb)bk [by commutativity in G]
⇒ = (aak)(bbk) = ak+1 bk+1
Thus if the result is true for n = k, it is also true for n = k + 1.
Hence by principle of induction it is true for all integers.
Case (iii) When n < 0, then let n = - m , where m ∈ Z+, then
(ab)n = (ab)-m = [(ab)m]-1
= (ambm)-1 [by case (ii)]
= (bm am)-1 [by commutativity in G]
= (am)-1 (bm)-1 = a-m b-m
= an bn
Combining all the three cases, we conclude that G is
commutative ⇒ (ab) n = anbn, ∀ 

Example : Prove that every group of order 4 is an abelian


group.

Let G = {e, a, b, c} be a group of order 4 with e as its identity.


Its composition table is as under :

Since every element of the group appears only once in each


row of the composition table, therefore in the second row
ab = e or ab = b or ab = c
But ab = b ⇒ a = e which is not possible (∵ a ≠ e)
∴ ab ≠ b, Hence ab = e or, ab = c
Similarly in the third row ba ≠ a
∴ ba = e or ba = c
Since every element of the group appears only once in each
column of the composition table, therefore

and 
Similarly it can be seen that
ac = ca and be = cb
Hence G is an abelian group.
Example : Consider   under addition modulo 10. Since 1-2 = 2,
2-2 = 4, 3-2 = 6, 4-2 = 8, 5-2 = 0, we know that |2| = 5. Similar
computations show that |0| = 1, |7| = 10, |5| = 2, |6| = 5.
Subgroup

If a subset H of a group G is itself a group under the operation of


G, we say H is subgroup of G. We use the notation H < G to mean
H is a subgroup of G. If we want to indicate that H is a subgroup of
G, but not equal to G itself, we write H < G. Such a subgroup is
called a proper subgroup. The subgroup {e} is called the trivial
subgroup of G, a subgroup that is not {e} is called a nontrivial
subgroup of G.
Equation Z n under addition modulo n is not a subgroup of Z under
addition, since addition modulo n is not the operation of Z

For example : The groups.

are contained in the group (C 0, x) and are groups for its induced
composition. Such groups are called subgroup of that large group.
Thus we see that any sub group H of any group (G, x) is called for
the composition of G and satisfies the axioms of the group for this
composition, then that is called a subgroup of G

Subgroup Tests

When determining whether or not a subset H of a group G is a


subgroup of G, one need not directly verify the group axioms.

One-Step Subgroup Test


Let G be a group and H a nonempty subset of G. Then, H is a
subgroup of G if ab -1 is an H whenever a and b are in H. (In
additive notation, H is a subgroup if a - b is in H whenever a and b
are in H.)
Proof : Sine the operation of H is the same as that of G, it is clear
that this operation is associative. Next, we show that e is in H.
Since H is nonempty, we may pick some x in H. Then, letting a = x
and b = x in the hypothesis, we have e = xx -1 is in H. To verify that
x-1 is in H whenever x is in H, all we need to do is to choose a = e
and b = x in the statement of the theorem. Finally, the proof will be
complete when we show that H is closed; that is, if x, y belong to H,
we must show that xy is in H also. Well, we have already shown
that y-1 is in H whenever y is; so letting a = x and b = y -1, we have
xy = x (y-1)-1 = ab-1 is in H.

Example : Let G be an Abelian group with identity e. Then H = {x ∈


G I x2 = e} is a subgroup of G Here, the defining property of H is the
condition x 2 = e. So, we first note that e 2 = e so that H is non-
empty. Now we assume that a and b belong to H. This means a 2 =
e and b 2 = e. Finally, we must show that (ab -1)2 = e. Since G is
Abelian, (ab -1)2 = ab-1ab-1 = a2 (b-1)2 = a2 (b2)-1 = ee-1 = e. Therefore,
ab-1 belongs to H and, by the One-Step Subgroup Test, H is a
subgroup of G.

Two-Step Subgroup Test


Let G be a group and H a nonempty subset of G. Then, H is a
subgroup of G if ab ∈ H whenever a, b, ∈ H (closed under
multiplication), and a -1 ∈ H whenever a ∈ H (closed under taking
inverses).
Proof. By Theorem it suffices to show that a, b g H implies ab -1 ∈
H. So, we suppose that a, b ∈ H. Since H is closed under taking
inverses, we also have b -1 ∈ H. Thus, ab -1 ∈ H by closure under
multiplication.

Finite Subgroup Test

Let H be a nonempty finite subset of a group G. Then, H is a


subgroup of G if H is closed under the operation of G.

Proof. In view of Theorem we need only prove that a~ 1 g H


whenever a g H. If a = e, then a -1 = a and we are done. If a ≠ e,
consider the sequence a, a 2, a3, ... Since H is finite and closure
implies that all positive powers of a are in H, not all of these
elements are distinct. Say, a i = aj and i > j. Then a i-j = e; and since
a ≠ e, i - j > 1. Thus, a i-j = a.ai-j-1 = e and, therefore a i-j-1 = a-1. But, i -
j - 1 ≥ 1 implies a i-j-1 ∈ H and we are done.
Proper and Improper (or Trivial) subgroup
Every group G of order greater than 1 has atleast two subgroups
which are :
(i) G (itself)
(ii) {e} i.e. the group of the identity alone.
The above two subgroups are known as improper or trivial
subgroups.
A subgroup other than these two is known as a proper subgroup.

Important Remark : If any subset of the group G is a group for any
operation other than the composition of G, then it is not called a
subgroup of G.

For example, the group {1, - 1 } is a part o f (C, +) which is group


for the multiplication but not for the composition (+) of the basic
group. Therefore this is not the subgroup of (C, +).

Examples of Sub Group

1. Additive Groups :
Example : (Z,+) is a subgroup of (Q,+)
Example : (Q,+) is a subgroup of (R,+).
Example : The set E of even integers is a proper subgroup
of additive group (E,+), whereas the set O of odd integers is
not a subgroup of the additive groups (Q,+), (Z,+).
Example : The set mZ of multiples of some given integer m
is a sub group of (Z,+),
Example : The group {0, 4} is a subgroup of (Z 8, +8) 
2. Multiplicative Groups :
Example : (Q+, x) is a subgroup of (R +, x).
Example : {1, - 1 } , {1, ω, ω 2}, {1, -1, i, - 1 } are subgroup of
(C0, x), the group of non zero complex numbers
Example : For multiplication operation ({1, -1}, x) is a
subgroup of {1, -1, i, -i}

Theorem : If H is a subgroup of a G, then :

 The identity of H is the same as that of G,


 The inverse of any element a of H is the same as the inverse
of the same regarded as an element of G
 The order of any element a of H is the same as the order of
a in G.

Proof.

 Let e and e’ be the identifies of G and H respectively.


If a ∈ H, then ae’ = e’ a = a ...(1)
Again a ∈ H ⇒ a ∈ G
∴ ae = ea = a ...(2)
From (1) and (2),
ae’ = ae
⇒ e’ = e [by cancellation law in G]
 If e is the identity of G, so by (a) e is also the identity of H.
Suppose that b and c are inverses of a in H and G
respectively.

Therefore the inverse of any element of subgroup is the


same as the of the subgroup and the original group.
 Let the order of a ∈ H be m and n in H and G respectively. If
e be the identity, then by the definition of order
am = e and an = e ⇒ am = an
⇒ am a-n = an a-n = a0
⇒ am-n = e
⇒ m - n = 0 [∵ m - n < m = O(a) in H]
⇒ m = n [m - n < n = O(a) in G]
Therefore, the order of any element of sub group is the same
as that of the sub group and the original group.
In the above results, we conclude that a subset H of a group
G is sub group iff :
(i) a ∈ H, b ∈ H, ⇒ ab ∈ H
(ii) e ∈ H where e is the identity of G
(iii) a ∈ H ⇒ a-1 ∈ H, where a-1 is the inverse of a in G.
Conversely : The above conditions are also sufficient because the
conditions (ii) and (iii) show the existence of identity and inverse
respectively and since the associative law holds in G, so also in H.

Algebra of complexes of a group

If H and K be two complexes of a group G, then their product HK


and the inverse H -1 are defined as follows :
HK = {hk | h ∈ H, k ∈ K}
H-1 = {h~1 | h ∈ H}
For example, if H = {-2, 4}, K = {1, 3} are two complex of the
group(Z, +), then HK = {(-2 + 1), (-2 + 3), (4 + 1), (4 + 3)}
= {-1, 1. 5, 7}
H-1 = {-2(-1), -4} = {2, -4}
and K-1 = {-1, -3} 

Properties of Complexes
The following properties can be easily established for the
complexes H, K and L of the group G:

1. (a) (HK) L = H(KL) [Associativity]


(b) (HK)-1 = K-1 H-1
2. If H is a subgroup of G, then :
(a) H-1 = H (but not conversely)
(b) HH = H

Remark : If HK = KH, then it does not mean that


hk = hk, ∀ h ∈ H, k ∈ K
∴ a ∈ H, b ∈ H ⇒ a ∈ H, b-1 ∈ H
⇒ ab-1 ∈ H [by closure property in H]
Therefore if H is a sub group of G, then the condition is necessary

Conversely : Suppose the given condition is true in H, then we


shall prove that H will be a sub group.
∵H≠φ
∴ Let a ∈ H
Now by the given condition,
a ∈ H, a-1 ∈ H ⇒ aa-1 = e ∈ H
Therefore identity exists in H.
Again by the same condition,
e ∈ H, a-1 ∈ H ⇒ ea-1 = a-1 ∈ H
Thus the inverse of every element exist in H.
Finally, a ∈ H, b ∈ H ⇒ a ∈ H, b-1 ∈ H
⇒ a(b-1)-1 = ab ∈ H
H is closed for the operation of G.
Therefore H is a sub group of G which proves that the given
condition is sufficient of H to be a sub group.

Remark. If the operation of the group is addition (+), then the


above condition will be :
a ∈ H, b ∈ H ⇒ (a - b) ∈ H
Using the complex property, the above condition may also be
written in the form of the following corollary:
A nonvoid subset of H of a group G is a subgroup iff :
(a) HH -1 ⊂ H (b) HH -1 = H

Criterion for a finite complex to be a sub group

Theorem : A nonvoid finite subset H of a group G is a subgroup if


a ∈ H, b ∈ H ⇒ ab ∈ H

Proof : Let H be a finite sub group of the group G, then H will be


closed for the operation of G.
So a ∈ H, b ∈ H ab ab ∈ H

Conversely : If a ∈ H, b ∈ H ab ∈ H,
then a ∈ H aa = a2 ∈ H
Again a ∈ H, a2 ∈ H ⇒ aa2 = a3 ∈ H
Thus we will see that
an-1 ∈ H, a ∈ H ⇒ an ∈ H, ∀ n ∈ N
Therefore a, a 2, a3, .... an, ... are all elements of H. But H is a finite
sub set of G, so all the powers of a can not be distinct.

Cor : A nonvoid finite subset H of a group G is a subgroup iff HH =


H.
Important Remark : It should be noted that the above condition is
necessary for all groups (finite or infinite) but it is not sufficient for
infinite groups.
Example : N is an infinite subset of the group (Z, +) which satisfies
the above conditions but is not a sub group because the identity 0
∈N
Example : Let (G = {2n | n ∈ Z}, x) and H = {1, 2, 2 2, 23, . . . } then H
⊆ G and closed wrt multiplication but it is not a sub group of G
because the multiplicative inverse of 2 i.e. (1/2) ∉ H.
Example : {1, -1} is a finite subgroup of (Q 0, x)

Intersection of subgroups

Theorem : The intersection of any two subgroups of a group G is


again a subgroup of G.

Proof. Let H1 and H2 be two sub groups of a group G.


∵ e ∈ H1, e ∈ H2 ⇒ e ∈ H1 ∩ H2 ⇒ H1 ∩ H2 ≠ φ
Now let a, b ∈ H1 ∩ H2, then
a, b ∈ H1 ∩ H2 ⇒ a, b ∈ H1 and a, b ∈ H2
⇒ ab-1 ∈ H1 and ab-1 ∈ H2 [∵ H1 and H2 are subgroups]
⇒ ab-1 ∈ H1 ∩ H2
∴ H1 ∩ H2 is a subgroup of G.

Generalisation
If H1, H2, .... Hn be a finite family of sub groups of G; then H 1 ∩ H2 ∩
.... ∩ Hn is also a subgroup of G.

Most Important Remark


The union of two subgroups is not necessarily a subgroup.
For example, the group G = (Z, +) has two subgroups
H =   then their union
HUK = {... -6, -4, -3, -2, 0, 2, 3, 4, 6, ...}
is not a subgroup of G because this is not closed for +.

For example, 2 ∈ H ∪ K, 3 ∈ H ∪ K
but their 2 + 3 = 5 ∉ H ∪ K
which shows that H ∪ K is not closed for addition.
Theorem : The union of two subgroups is a subgroup iff one is
contained in the other.
Proof. Let H1 and H2 be two subgroups of a group G.
First suppose that H 1 ∪ H2 is a subgroup of G, then we have to
show that either H 1 ⊂ H2, or H2 ⊂ H1.
Now let us suppose that H 1 ⊂ H2,
then H1 ⊄ H2 ⇒ ∃ a ∈ H1 and a ∉ H2 ...(1)
Let b ∈ H2, then
a ∈ H1, b ∈ H2 ⇒ a ∈ H1 ∪ H2 and b ∈ H1 ∪ H2
⇒ ab ∈ H1 ∪ H2 [∵ H1 ∪ H2 is a subgroup]
⇒ ab ∈ H1 or ab ∈ H2
Now ab ∈ H2, b ∈ H2 ⇒ (ab)b -1 = a ∈ H2
which contradicts our assumption (1).
Therefore ab ∉ H2. Hence ab ∈ H1
But ab ∈ H1 ⇒ a-1 (ab) = b ∈ H1
⇒ H2 ⊂ H1

Conversely : Now suppose that H 1 ⊂ H2 or H2 ⊂ H1, then H1 ∪ H2 =
H2 or H1
⇒ H1 U H2 is also a subgroup of G.

Criteria for the product of two subgroups of a group G

Theorem : Let H, K be subgroups of a group G. Then HK is a


subgroup of G if and only if HK = KH.
Proof. Let HK = KH, then we shall prove that HK is a subgroup of
G
∵ (HK)(HK)-1 = (HK)(K-1 H-1) [∵ (HK)-1 = K -1 H-1]
= H(KK-1)H-1 [by associativity]
= (HK)H-1 [∵ K is a subgroup ⇒ KK-1 = K]
= (KH)H-1 [∵ HK = KH]
= K(HH-1)
= KH [∵ H is a subgroup ⇒ HH-1 = H]
= HK
∴ HK is a subgroup of G.
Conversely : Now let HK be a subgroup of G, then
HK is a subgroup ⇒ (HK) -1 = HK
⇒ K-1 H-1 = HK
⇒ KH = HK [∵ H, K are subgroups ⇒ H-1 = H, K-1 = K]
Therefore HK is a sub group ⇔ HK = KH

Corollary : If H and K are subgroups of a commutative group then


HK is a subgroup of G.
Proof : ∵ G is abelian ⇒ HK = KH ⇒ HK is a subgroup

Example : Prove that the set of the n th roots of unity is a


subgroup of the multiplicative group 

Let H =   be n complex numbers in H.


Therefore H is a non empty subset of  then x,
y ∈ H ⇒ xn = 1 and yn = 1
⇒ xn.(1/yn) = 1.1. = 1
⇒ (x/y)n = 1 ⇒ (xy-1)n = 1
⇒ xy-1 ∈ H
∴ H is a subgroup of (C 0, x).

Example : Prove that H = {a + ib | a, b e  is a subgroup of the


group

Clearly H is a non empty subset of C.


Let x, y ∈ H where x = a 1 + ib1, y = a2 + ib2
then x - y = (a 1 + ib1) - (a2 + ib2)
= (a1 - a2) + i(b2 - b2) ∈ H [∵ a1 - a2 ∈ Q, b1 - b2 
∴ x ∈ H, y ∈ H ⇒ (x - y) ∈ H
Therefore H is a subgroup of (C, +).

Example : Prove that H is a subgroup of the group (C 0, x)


where
 

Clearly H is a non empty subset of C 0.


Let x, y ∈ H where x = a 1 + b1√2 and y = a2 + b2√2
then 

Now if  which is false.


Therefore 

Consequently  are rational.


Therefore xy-1 ∈ H. Hence H is a sub group of 

Example : Show that H is a subgroup of G for matrix


multiplication :

Clearly H is a non empty subset of G.


Let A, B ∈ G where

 a1, a2, b1, b2, d1, d2 ∈ R, a1d1 ≠ 0 a2d2 ∈ 0

then 

∴ 
 [∵ a2d2 ≠ 0, d2 ≠ 0]
Thus we see that A ∈ H, B ∈ H ⇒ AB-1 ∈ H
Therefore, H is a subgroup of G.

Example : If a is an element of a group G, then prove that its


normalizer N(a) = {x ∈ Glax = xa} is a subgroup of G :

∵ e ∈ G ⇒ ae = ea ⇒ e ∈ N(a) ⇒ N(a) ≠ φ
Therefore let x 1 x2 ∈ N(a), then by definition of N(a)
ax1 = x1a and ax2 = x2a
Now   
⇒ 
⇒ 
⇒ 
⇒ 
∴ 
Again   [by associativity]
 [∵ x1 ∈ N(a)]
 [by associativity]
 
 [by associativity]
∴ 
Thus we see that

Therefore N(a) is a subgroup of G.

Example : For any group G, prove that its centre Z(G) = {x ∈ GI


xg = gx, ∀ g ∈ G} is a subgroup of G.

∵ e ∈ Q ⇒ eg = ge, ∀ g ∈ G
⇒ e ∈ Z(G) ⇒ Z(G) ≠ φ
Therefore let x 1, x2 ∈ Z(G)
∴ x1g = gx1 and x2g = gx2, ∀ g ∈ G
Now 
⇒ 
⇒ 
⇒ 
⇒ 
∴ 
Again   [by associativity]
-1 -1
= x1(gx2 ) [∵ x2  ∈ Z(G)]
= (x1g)x2-1 [by associativity]
= (gx1)x2-1 [∵ x1 ∈ Z(G)]
= g(x1x2-1) [by associativity]
∴ x1x2-1 ∈ Z(G)
Thus we see that
x1 ∈ Z(G), x2 ∈ Z(G) ⇒ x1x2-1 ∈ Z(G)
Therefore Z(G) is a subgroup of G.

Example : Let G be the set of all ordered pairs (a, b) of real


numbers for which a ≠ 0. Let a binary operation * on G be
defined by (a, b) * (c, d) = (ac, be + d). Does the subset H of all
those elements of G which are of the form (1, b) form a
subgroup of G ?

If (a, b) ∈ G, then a ≠ 0

∴ 

Again 

and 

∴   ∈ G is the inverse of (a, b)


Thus the inverse of every element of G also exist in G.
Hence (G, o) is a group.
Clearly H is a non empty subset of G
Let (1, b),(1, c) ∈ H, then

= (1, b)*(1, -c)

= (1, b-c)[by def of * on G]


But (1, b - c) is definitely an element of H.
Therefore, (1, b),(1, c) ∈ H ⇒ (1, b)*(1, c)-1 ∈ H
Hence H is a subgroup of G.

Examples of Subgroups

1. <a> Is a Subgroup
Let G be a group, and let a be any element of G. Then, <a> is
a subgroup of G.
Proof : Since a ∈ <a>, <a> is not empty. Let a n, am ∈ <a>.
Then, an (am)-1 = an-m ∈ <a> ; so, by Theorem <a> is a
subgroup of G.
Example : In U (10), <3> = {3, 9, 7, 1} = U (10), for 3 1 = 3, 32 =
9, 33 = 7, 34 = 1, 35 = 34.3 = 1-3, 3 6.32 = 9, 3-1 = 7 (since 3.7 =
1), 3-2 = 9, 3-3 = 3, 3-4 = 1, 3-5 =3-4.3-1 = 1.7, 3 -6 = 3-4.3-2 = 1.9 =
9,...
2. Center of a Group
The center, Z(G), of group G is the subset of elements in G
that commute with every element of G. In
symbols, 
3. Center Is a Subgroup
The center of a group G is a subgroup of G.
Proof : For variety, we shall use Theorem to prove this result.
Clearly, e ∈  (G), so  (G) is nonempty. Now, suppose a, b
∈  (G). Then (ab)x = a(bx) = a(xb) = (ax)b = (xa)b = x(ab) for
all x in G ; and, therefore, ab ∈  (G).
Next, assume that a ∈ (G). Then, we have ax = xa for all x in
G. What we want is a -1x = xa-1 for all x in G. Informally, all we
need do to obtain the second equation from the first one is
simultaneously to bring the a’s across the equal sign:
becomes xa-1 = a-1x. (Be careful here; groups need not
be commutative. The a on the left comes across as a -1 on the
left and the a on the right comes across as a -1 on the right.)
Formally, the desired equation can be obtained from the
original one by multiplying it on the left and right by a -1, like
so:
a-1(ax)a-1 = a-1(xa)a-1,
(a-1a)xa-1 = a-1x(aa-1),
exa-1 = a-1xe,  
xa-1 = a-1x.
This shows that a -1 ∈  (G) whenever a is.
4. Centralizer of a in G
Let a be a fixed element of a group G. The centralizer of a in
G, C(a), is the set of all elements in G that commute with a. In
symbols, C(a) = {g ∈ G | ga = ag}.
5. C(a) Is a Subgroup
For each a in a group G, the centralizer of a is a subgroup of
G is defined as
CG(A) = {g ∈ G : gag -1 = a, ∀a ∈ A}
NOTE : Normalizer and Centralizer are same for an element.
Their difference can be understood only if you define them for
a subset of a Group.
Normalizer of a subset A of G is defined to be N G(A) = {g ∈ G :
gag -1 ∈ A, ∀a ∈ A}.

Normal Subgroup

We have seen that if H is a sub group of a group G, then, in


general aH ≠ Ha, a ∈ G is an abelian group, then it holds good for
every subgroup of such a group. We may also have subgroups H of
a non abelian group G which satisfy aH = Ha, ∀ a ∈ G,.
Therefore such a subgroup for which every left coset is equal to its
corresponding right coset is named as Normal Subgroup and it
plays a very important role in the further studies. We shall now
define such a subgroup in a slightly different form which will be
more helpful to derive its certain properties. 

Definition

A subgroup H of a group G is called a normal subgroup of G if aH =


Ha for all a in G. We denote this by 
Many students make the mistake of thinking that “ H is normal in G”
means ah = ha for a ∈ G and h ∈ H. This is not what normality of H
means; rather, it means that if a ∈ G and h ∈ H, then there exists
some h' ∈ H such that ah = h’a.

Proper and Improper Normal subgroups

It can be observed that every group G has atleast following two


normal subgroups :
(ii) G itself
and (ii) {e}, the group consisting of the identity alone.
These two subgroups are called Improper normal subgroup of G
and a normal subgroup other than these two is called a Proper
normal subgroup.

Simple Groups
A group which has no proper normal subgroups is called a simple
group

Example : Every group of prime order is simple because such a


group has no proper subgroup.

Hamiltonian Group
If all the subgroups of a non Abelian group are normal, then called
Hamiltonian group.
Remarks : The normal subgroup is also called a special subgroup
or invariant subgroup or self conjugate subgroup. 

Some properties of Normal subgroups


Theorem : Every subgroup of an abelian group is a normal group.
Proof. Let H be a subgroup of any commutative group G.
If x ∈ G and h ∈ H, then
xhx-1 = (hx)x-1 [∵ G is commutative]
= h(xx-1) [by associativity]
= he = h ∈ H
Thus, x ∈ G, h ∈ H ⇒ xhx-1 ∈ H,
∴ H is a normal subgroup of G.

Corollary. Every subgroup of a cyclic group is a normal subgroup.


Theorem : A subgroup H of a group G is a normal subgroup iff :

Proof. Let  , then
x ∈ G, h ∈ H ⇒ xhx-1 ∈ H
⇒ xHx-1 ⊂ H ...(1)
Again ∀ x ∈ G, xHx-1 ⊂ H ⇒ x-1 ⊂ H  ...(1)
⇒ x-1Hx ⊂ H 
⇒ x(x-1Hx)x-1 ⊂ xHx-1
⇒ (xx-1)H(xx-1) ⊂ xHx-1
⇒ H ⊂ xHx-1 ...(2)
(1) and (2) ⇒ xHx-1 = H.
∴   ⇒ xHx-1 = H,  ∀ x ∈ G.

Conversely : Let xHx -1 = H, ∀ x ∈ G.


then xHx-1 = H ⇒ xHx-1 ⊂ H
⇒ xhx-1 ∈ H, ∀ h ∈ H, x ∈ G.
∴ 
Hence   ⇔ xHx-1 = H, ∀ x ∈ G.

Theorem : A subgroup H of a group G is a normal subgroup iff


each left coset of H is right coset of H (and hence also iff each right
coset of H is a left coset of H) i.e.
 ⇔ xH = Hx, ∀ x ∈ G
Proof. Let  , then by theorem
 ⇒ xHx -1 = H, ∀ x ∈ G.
⇒ (xHx-1)x = Hx
⇒ (xH)(x-1x) = Hx
⇒ xH = Hx,  ∀ x ∈ G.
Conversely : If xH = Hx, ∀ x ∈ G, then
xH = Hx ⇒ xHx-1 = Hxx-1
⇒ xHx-1 = He = H
⇒ 
Therefore   ⇔ xH = Hx, ∀ x ∈ G.

Example : A subgroup H of a group G, is a normal subgroup of G


iff the product of two right (left) cosets of H in G is again a right
(left) coset of H in G.
Proof. Let 
If a, b ∈ G then Ha and Hb are two right cosets of H in G.
Let h1a ∈ Ha and h2b ∈ Hb, then
h1ah2b ∈ HaHb
But h1ah2b = h1(ah2)b
= h1(h2a)b [∵ Ha = aH ⇒ ah2 = h2 a, h2 ∈ H]
∴ HaHb ⊂ Hab ...(1)
Again for every h ∈ H
h(ab) = (ha)b = (ha)eb ∈ Ha Hb
∴ Hab ⊂ HaHb ...(2)
Hence (1) and (2) ⇒ Ha Hb = Hab
 ⇒ HaHb = Hab.

Conversely : Suppose that the product of two right cosets of H is


again a right coset of H in G. Let x ∈ G then Hx and Hx -1 are two
right cosets of H in G. Then as supposed HxHx-1 is also a right
coset of H in Q
Now x ∈ Hx and x-1 ∈ Hx-1 ⇒ xx-1 ∈ HxHx -1 = Hy (say)
⇒ e ∈ Hy.
But H itself is a right coset of H containing e and any two right
cosets are either identical or disjoint, so
HxHx-1 = Hy = H, ∀ x ∈ G

⇒ H is a normal subgroup.
Theorems on normal subgroup

Theorem : The intersection of any two normal subgroups of a


group is a normal subgroup.
Proof. Let H1 and H2 be two normal subgroups of a group G.
Earlier we have seen that the intersection of two subgroups of a
group G is again a subgroup, so H 1 ∩ H2 is also a subgroup of G.
Now if x ∈ G and h ∈ H1 ∩ H2,
then h ∈ H1 ∩ H2 ⇒ h ∈ H1 and h ∈ H2
Now, since 
∴ x ∈ G, h ∈ H1 ⇒ xhx -1 ∈ H1.

Similarly 
∴ x ∈ G, h ∈ H2 ⇒ xhx -1 ∈ H2.
Thus we see that
x ∈ G, h ∈ H1 ∩ H2 ⇒ xhx -1 ∈ H and xhx -1 ∈ H2
⇒ xhx-1 ∈ H1 ∩ H2
∴ H1 ∩ H2  G 

Corollary. The intersection of any finite family of normal subgroups


is a normal subgroup
Theorem. If H is a subgroup of G and N < G, then H ∩ N   H.
Proof. Since intersection of two subgroup is also a subgroup,
So H ∩ N is also a subgroup of G.
Also H ∩ N ⊂ H, therefore H ∩ N is also a subgroup of H.
Let h ∈ H ∩ N and x ∈ H,
then h ∈ H ∩ N ⇒ h ∈ H and h ∈ N
Again h ∈ H, x ∈ H ⇒ xhx-1 ∈ H [∵ H is a subgroup]
and h ∈ N, x ∈ H ⇒ h ∈ N, x ∈ G, [∵ H ⊂ G]
⇒  xhx-1 ∈ N. [∵ N is normal]
Therefore xhx -1 ∈ H, xhx-1 ∈ N ⇒ xhx-1 ∈ H ∩ N.
Thus we see that
h ∈ H ∩ N, x ∈ H ⇒ xhx-1 ∈ H ∩ N.
∴ 

Theorem : If H and K are two normal subgroups of a group G,


then 
Proof. HK is a sub group of G :
∵   ∴ for every k ∈ K, k ∈ G
Hk = kH ⇒ HK = KH.
⇒ HK is a subgroup of G. ...(1)

Let x ∈ G and hk ∈ HK, then


x(hk)x -1 = x(hx-1 xh)x -1
= (xhx -1)(xkx-1) ∈ HK. [∵ xhx -1 ∈ H, xhx-1 ∈ K]
Thus we see that
x ∈ G, hk ∈ HK ⇒ x(hk)x -1 ∈ HK.
From (1) and (2),   

The existence of Normal Subgroups


Theorem : Every subgroup of a group with index 2 is a normal
subgroups.
Let H be a subgroup of a group G such that its index is 2. Then, H
has only two distinct right (left) cosets in G.
So let G = H ∪ Hx, where Hx ∩ H ≠ φ
Now, Hx ∩ H = φ and x ∈ Hx ⇒ x ∉ H
⇒ H and xH are also disjoint. [∵ x ∈ xH]
∴ G = H ∪ xH
then H ∪ H x = G = H ∪ xH ⇒ Hx = xH
⇒ 

Example : Let G be a group and H be its subgroup generated


by {x-1 y-1 xy | x, y ∈ G}. Show that 

Let h ∈ H and g ∈ G, then h ∈ G, g ∈ G


Therefore h-1 ∈ G, g-1 ∈ G.
Now ghg-1 = (ghg-1 h-1h) = (ghg -1h-1)h
= [(g-1)-1(h-1)-1 g-1h-1]h
Here let g-1 = x, h-1 = y, then
ghg-1 = (x-1 y-1 xy)h ∈ H
Hence 

Example : Let G be the group of all real matrices of

types   under matrix multiplication and N

=   then prove that   

(i) N is a subgroup of G :
Let α, β ∈ N, where

  and 

then 

Therefore 

 [∵ b1 - b2 ∈ R]


Thus α ∈ N, β ∈ N ⇒ αβ-1 ∈ N.
∴ N is a subgroup of G.

(ii) 

Again if   then

∴ 

 
Thus γ ∈ G, α ∈ N ⇒ γαγ-1 ∈ N.
∴ 
Example : If H is a subgroup of a group G such that x 2 ∈ H, ∀ x
∈ G, then prove that  .

Let g ∈ G and h ∈ H.
Then g ∈ G, h ∈ H ⇒ gh ∈ G
⇒ (gh) 2 ∈ H [by the given property]

Again 
and (gh)2 ∈ H, h-1 g-2 ∈ H ⇒ (gh) 2h-1g-2 ∈ H
⇒ ghghh-1 g-1 g-1 = ghg-1 ∈ H
Thus g ∈ G, h ∈ H ⇒ ghg-1 ∈ H
⇒ 

Example : If    and K   G such that H ∩ K = {e}, then


show that hk = kh for every h ∈ H and k ∈ K.

Since H and K are subgroups, therefore


h ∈ H ⇒ h-1 ∈ H and k ∈ K ⇒ k-1 ∈ K
Now, h-1 ∈ H, k ∈ G ⇒ kh-1 k-1 ∈ H [∵  ]
⇒ h(kh-1k-1) = hkh-1 k-1 ∈ H [∵ h ∈ H]
Again k ∈ K, h ∈ G ⇒ hkh-1 ∈ K
⇒ (hkh-1)k-1 = hkh-1 k-1 ∈ K 
Thus hkh-1 k-1 ∈ H and hkh-1 k-1 ∈ K
⇒ hkh-1 k-1 ∈ H ∩ K
⇒ hkh-1 k-1 = e
⇒ hk = kh. 

Example : If H is the only subgroup with finite order in a group


G, then prove that   

We have already proved that for any x ∈ G, xHx-1 is a


subgroup of G. Since H is a subgroup of finite order, therefore
let
O(H) = m and H = {h 1, h2, .... hm}
∴ xHx-1 = {xh1x-1, xh2x-1, ..., xhmx-1}
∵ xhix-1 = xhjx-1 ⇒ hi  = hj [by cancellation law in H]
Therefore all the elements in xHx -1 are different.
Hence O(xHx -1) = m = O(H)
But H is the only subgroup of finite order in G, therefore
xHx-1 = H ⇒ 

Example : Let H and N be subgroup and normal subgroup of a


group G respectively, then prove that 

Suppose identity e ∈ H and for any arbitrary x ∈ N


Again n1 be any arbitrary element of HN, such that
h ∈ H and n ∈ N
Again n1 be any arbitrary element of N, then
(hn)n1 (hn)-1 = hn n1n-1h-1 = h(nn1n-1)h-1 ...(1)
Since   Therefore nn1n-1 ∈ N, ∀ n ∈ G and n1 ∈ N and
consequently, in particular nn 1n-1 ∈ N, ∀ n ∈ N and n1 ∈ N

Therefore h ∈ H, nn1n-1 ∈ N ⇒ h ∈ G, nn, n-1 ∈ N


⇒ h(nn1n-1)h-1 ∈ N
Therefore by (1), (hn)n(hn) -1 ∈ N, ∀ hn ∈ HN and n ∈ N
Hence 

Normal Subgroup Test

A subgroup H of G is normal in G if and only if xHx -1 ⊆ H for all x in


G.
Example : Every subgroup of an Abelian group is normal. (In this
case, ah = ha for a in the group and h in the subgroup.)
Example : The center Z(G) of a group is always normal. (Again, ah
= ha for a ∈ G and h ∈ Z(G).)

Cyclic Groups

Definition : A group G is called a cyclic group if all of its element


can be express as power of a single element say a ∈ G.
In this case ‘a’ is called to be a generator of G. i.e. if ‘a’ is
generator then for x ∈ G there is an integer k such that a k = x
Let G be a finite group of order n then G = {a, a 2, a3, ..., an-1, an = e}.
Note that order of cyclic group is equal to the order of its generator
and the generating element is not necessary unique.
e.g. {±1, ±i}.
Let a = i,
a2 = i2 = -1
a3 = i3 = i • i2 = -i  
a4 = (i2)2 = 1
Also if a = -i, then this is also generator i.e. i, - i are a generator.

Theorem : If a is an element of a group G, then :   is


a subgroup of G.

Proof. Let x, y ∈ H where x = a p, y = aq p, q ∈  


then xy-1 = ap(aq)-1 = ap.a-q = ap-q ∈ H [∵ p - q ∈  ]
∴ x ∈ H, y ∈ H ⇒ xy-1 ∈ H
Therefore H is a sub group of G.
From this theorem it is clear that every element of the group G
generates a sub group which is called cyclic group.
For every element a of the group G
H = {an | n ∈  }
is a subgroup of G which is called cyclic group generated by a.
It is denoted by [a].
A group G is called cyclic if there is an element a in G such that G
= {an | n ∈  }. Such an element a is called a generator of G. we
may indicate that G is a cyclic group generated by a by writing G =
<a>.

Example : The set of integers   under ordinary addition is cyclic.


Both 1 and -1 are generators. (Recall that, when the operation is
addition, 1n is interpreted as 

when n is positive and as  when n is


negative.)

Examples of Cyclic Group


Example :   is an infinite cyclic group with 1 and -1 as its two
generators.

Example : The multiplicative group {1, - 1, i, - i }, x) is a finite cyclic


group with i and - i as its two generators, because G = {i 1 = i, i2 = -1,
i3 = -i, i4 = 1} = [i] and G = {(-i)1= -i, (-i) 2 = -1, (-i) 3 = i,(-i)4 = 1} = [-i]

Example : The group [ 4 = {0, 1, 2, 3}, + 4] is a cyclic group with 1


and 3 as its two generators, because  4 = (1(1) = 1, 2(1) = 2, 3(1)
= 3, 4(1) = 0} = [1]
and   4 = (1(3) = 3, 2(3) = 2, 3(3) = 1, 4(3) = 0} = [3]

Properties of cyclic group

Theorem : Every cyclic group is abelian


Proof. Let G = [a] be a cyclic group and x, y ∈ G, then
x = am, y = an m, n ∈   
then xy = aman = am+n = an+m [∵ m + n = n + m]
= anam = yx
∴ G is an abelian group.

Remark : An abelian group need not be cyclic.


Example : (R,+) Is an abelian but not cyclic.
Theorem : If a is a generator of a cyclic group G, then a -1 is also its
generator.
Proof. Let G = [a] be a cyclic group and x ∈ G
Since G is a cyclic group, so there exist an integer m such that
x = am ⇒ x = (a -1)-m, [- m ∈ Z]
⇒ x can also be expressed as some integral power of a -1
⇒ a-1 is also the generator of G.
Therefore G = [a] ⇒ G = [a-1]
Theorem : The order of a finite cyclic group is equal to the order of
its generator i.e.,
Proof. Let G = [a] be a finite cyclic group and O(a) = n
Let H = {a, a 2, a3,..,an = e}
Clearly H is a sub group of G whose order is n.
Case 1. When m ≤ n : If a m ∈ G, then am ∈ H
∴ H ⊂ G ...(1)
Case 2. m > n : m = q n + r, 0 ≤ r < n, q, r ∈ 
⇒ am = aqn+r [by division algorithm]
= (an)q • ar = ear
= ar ∈ H
∴ G ⊂ H ...(2)
(1) and (2) ⇒ G = H
But O(H) = n
∴ O(G) = n = O(a)

Corollary. A finite group of order n is cyclic iff it has an element of


order n.
Proof. Let G = [a] be a finite cyclic group of order n.
Then by the above theorem, an element a exists in G such that
O(a) = O(G) = n

Conversely : Let G be a finite cyclic group of order n in which an


element a exists such that O(a) = n
Now if H = [a], then H ⊂ G and by the above theorem O(a) = n ⇒
O(H) = n
⇒ O(H) = O(G)
Similarly, G is finite group such that
H ⊂ G and O(G) = O(H)
⇒ G = H = [a]
⇒ G is a cyclic group generated by a.

Theorem : Every infinite cyclic group has two and only two
generators.
Proof. Let G = [a] be an infinite cyclic group. Then by theorem a -
1
 is also a generator of G.
To show : a ≠ a-1
Let a = a-1, then a = a-1
⇒ aa = a-1a ⇒ a2 = e
⇒ O(a) = 2 ⇒ O(G) = 2
which is not possible because G is an infinite group. Therefore a ≠
a-1
To show that G does not have any generator other than these
two :
Let, if possible, a m, m ≠ ± 1 be also a generator of g.
Then for a ∈ G there exists an integer n such that
a = (am)n = amn
⇒ aa-1 = amn a-1
⇒ e = amn-1
⇒ O(a) is finite.
⇒ O(G) is finite.
which contradicts that G is infinite.
Hence am can not a generator of G unless m = 1 or -1.
Consequently, G has exactly two generators a and a -1.

Theorem : Every subgroup of a cyclic group is also cyclic.


Proof. Let G = [a] be a cyclic group and H be a sub group of G. If H
= G or H = {e}; then clearly H also cyclic.
If H is a proper sub group of G, then H contains at least one
element am (m  , m ≠ 0) other than the identity.
m -m
a  ∈ H ⇒ a  ∈ H [∵ H is a sub group]
Since m ≠ 0, therefore m > 0 or m < 0
⇒ There exist positive integral powers of a in H.
Let m be the least positive integer such that a m ∈ H
To prove H = [am] :
Let an ∈ H, then by division algorithm, there exist two integers q
and r such that n = mq + r, 0 ≤ r < m
or n - mq = r
Now since am ∈ H ⇒ (am)q = amq ∈ H
⇒  (amq)-1 = a-mq ∈ H
∴  an ∈ H, a-mq ∈ H ⇒ an • a-mq = an-mq ∈ H
⇒ ar ∈ H [∵ n-mq = r]
But m is the least positive integer such that a m ∈ H and 0 ≤ r < m
Therefore r = 0 Consequently n = mq
and so a n = amq = (am)q ⇒ H = [am]
Therefore every subgroup of G is cyclic.

Corollary. Every proper subgroup of an infinite cyclic group is


infinite.
Proof. Let G = [a] be an infinite cyclic group and H be a proper sub
group of G. Then by the above theorem H is cyclic and H = [a m]
where m is the least positive integer such that a m ∈ H
Let, if possible,
O(H) = p, (p ∈ N)
then O(H) = p ⇒ O(am) = p
⇒ (am)p = e
⇒ amp = e
⇒ O(a) is finite
⇒ O(G) is finite.
which is contrary to the hypothesis. Therefore the order of H can
not be finite. Consequently every proper subgroup of G is infinite.

Theorem : If G = [a] is a cyclic group of order n and H = [a s], then


H is a cyclic subgroup of G is of order n/d where d is HCF of n and
s.
Proof. Let O(H) = m
then O(H) = m ⇒ O(as) = m
⇒ (as)m = ams = e
⇒ O(a) | ms
⇒ n | ms [∵ O(G) = n ⇒ O(a) = n]
Therefore d = HCF of {n, s}
Therefore there exist two integers r and q which are relatively
prime.
and n = rd and s = qd ...(1)
Now N | ms ⇒ rd | m(qd)
⇒ r | mq
⇒ r | m or r | q
⇒ r | m  [∵ r and q are relatively prime] ...(2)
Again (as)r = ars = arqd = aq(rd) = aqn
⇒ (as)r = aqn = (an)q = e
⇒ O(as) | r
= m | r ...(3)
From (1), (2) and (3), m = r = n/d
O(H) = n/d
Corollary. If a is a generator of a finite cyclic group of order n, then
the other generators of G are of the form a r, where r is relatively
prime to n.
Proof. ∵ O(G) = O(a) = n
∴ 0[ar[ = n/d, where d is HCF of n and r. [by the above theorem]
If ar is also a generator of G, then
G = [a] = [ar] ⇒ O[a] = O[ar]
⇒ n = n/d
⇒d=1
⇒ r is relatively prime to n.

Corollary. Every finite cyclic group of composite order posses


proper subgroups.
Proof. Let G = [a] be a finite cyclic group of composite order.
O(G) = mn, n ≠ 1; n ≠ 1
If H = [am] then by the above theorem,
O(H) = (mn/m) = n < mn = O(G)
Therefore, H is a proper subgroup of G.

Example : Prove that group of order 3 is cyclic.

Let G = {e, a, b} be a group and a and b are different.


Now a ∈ G , b ∈ G ⇒ ab ∈ G [∵ closed]
Therefore, three possibilities are :
(1) ab = e or, (2) ab = a or, (3) ab = b
Now ab = a ⇒ b = e,
which is against the hypothesis that e is different from a and b.
Therefore ab ≠ a,
Similarly ab ≠ b
Hence ab = e ...(1)
Again a2 ∈ G. Therefore there are three possibilities :
(i) a2 = e or, (ii) a 2 = a or, (iii) a 2 = b
(1) If a2 = e, then by (1), a 2 = ab ⇒ a = b [by cancellation law]
which is against our hypothesis. Therefore
a2 ≠ e
(2) Again a2 = a ⇒ a = e but a and e are different elements
Therefore a2 ≠ a and consequently a 2 = b
∴ G = {e, a, a 2}
Finally, a3 = aa2 = ab = e
Hence G is cyclic and a is its generator.

Example : Prove that the set G = {x | x n = 1} of n th roots of unity


is a finite multiplicative cyclic group of order n.

Let x1, x2 ∈ G,


then 
But 
⇒ (x1x2)n = 1
⇒ x1x2 ∈ G
∴ x1 ∈ G, x2 ∈ G ⇒ x1x2 ∈ G
So G is closed for multiplication.
Also 1n = 1 ⇒ 1 ∈ G which is the identity for multiplication.
Further for each x ∈ G,

which is the inverse of x. So each element of G is invertible.


Lastly, the multiplication of numbers is associative, so it is also
associative in G. Hence G is a group for multiplication.
∵ xn = 1 has exactly n roots, So O(G) = n
G is a Cyclic group : ∵ 1 = cos 2mπ + sin 2mπ, m ∈ 

Therefore G is a cyclic group of order n generated by e 2πi/n.

Example : Find all the generators of the cyclic group


(G = { 0 , 1 , 2, 3, 4, 5}, + 6)

We find that
1(0) = 0 ⇒ O(0) = 1
1(1) = 1, 2(1) = 2, 3(1) = 3, 4(1) = 4, 5(1) = 5, 6(1) = 0 ⇒ O(1)
=6
1(2) = 2, 2(2) = 4, 3(2) = 0 ⇒ O(2) = 3
1(3) = 3, 2(3) = 0 ⇒ O(3) = 2
1(4) = 4, 2(4) = 2, 3(4) = 0 ⇒ O(4) = 3
1(5) = 5, 2(5) = 4, 3(5) = 3, 4(5) = 2, 5(5) = 1, 6(5) = 0 ⇒ O(5)
=6
Observing the orders of all the elements of G, we find
O(1) = O(5) = 6 = O(G)
Therefore G = [1] = [5] i.e. 1 and 5 are two generators of g.

Example : Find all the generators of the cyclic group (G = {1, 2,


3, 4}, x 5)

Here we have O(G) = 4. Its generator is the element of G


whose order is 4.
We have 1 = 1 ⇒ O(1) = 1
21 = 2, 22 = 4, 23 = 3, 24 = 1 ⇒ O(2) = 4
31 = 3, 32 = 4, 33 = 2, 34 = 1 ⇒ O(3) = 4
42 = 1 ⇒ O(4) = 1
Clearly, 2, 3 ∈ G such that
O(2) = 4 = O(G) and O(3) = 4 = O(G)
⇒ 2 and 3 are two generators of the cyclic group G.

Example : Find all the generators of the cyclic group of order


(a) 8 (b) 12

(a) Let G = [a] be a cyclic group of order 8.


By theorem the other generators of G will be of the form
am where m < 8 relatively prime number.
So m = 3, 5, 7
Therefore all the generators of G are : a, a 3, a5, a7
(b) Again let G = [a] be a cyclic group of order 12.
The other generators of G will be of the form a m
where m < 12 and is relatively prime number.
So m = 5, 7, 11
Therefore all the generators of G are : a, a 5, a7, a11.
Theorem : Criterion for a i = aj
Let G be a group, and let a belong a belong to G. If a has infinite
order, then all distinct powers of a are distinct group elements. If a
has finite order, say, n, then <a> = {e, a, a 2, ..., a n-1} and a i = aj if
and only if n divides i - j.
Proof : If a has infinite order, there is no non-zero n such that a n is
the identity. Since a i = aj implies a i-j = e, we must have i - j = 0, and
the first statement of the theorem is proved.
Now assume that |a| = n. We will prove that <a> = {e, a, ... a n-1}.
Certainly, the elements e, a, ..., a n-1 are distinct. For if a i = aj with 0
≤ j < i ≤ n - 1, then a i-j = e. But this contradicts the fact that n is the
least positive integer such that a n is the identity.
Now, suppose that a k is an arbitrary member of <a>. By the division
algorithm, there exist integers q and r such that
k = qn + r with 0 ≤ r < n.
Then ak = aqn+r = aqn • ar = (an)q • ar = ar, so that ak ∈ {e, a, a2, ... an-
1}. This proves that <a> = {e, a, a 2, .... a n-1}.
Next, we assume that a i = aj and prove that n divides i - j. We begin
by observing that a i = aj implies ai-j = e. Again, by the division
algorithm, there are integers q and r such that
i - j = qn + r with 0 ≤ r ≤ n.
Then ai-j = aqn+r and, therefore, e = ea r = ar. Since n is the least
positive integer such that a n is the identity, we must have r = 0 so
that n divides i - j.
Conversely, if n divides i - j, then a i-j = anq = eq = e so that a i = aj.

Corollary : ak = e Implies that |a| Divides k


Let G be a group and let a be an element of order n in G. If a k = e,
then n divides k.
Proof : Since ak = e = a 0, we know by Theorem that n divides k - 0.

Generators of Cyclic Groups

Let G = <a> be a cyclic group of order n. Then G = <a k> if and only
if gad(k, n) = 1.
Proof : If gcd(k, n) = 1, we may write 1 = ku + nv for some integers
u and v. Then a = a ku+nv = aku • anv = aku. Thus, a belongs to <a k>
and therefore all powers of a belongs to <a k>. So, G = <ak> and ak
is a generator of G.
Now suppose that gcd(k, n) = d > 1. Write k = td and n = sd.
Then (ak)s = (atd)s = (asd)t = (an)t = e, so that la kl ≤ s < n. This shows
that ak is not a generator of G.

Generators of 
An integer k in   is a generator of   if and only if gcd(k, n) = 1.

Classification of Subgroups of Cyclic Groups


Fundamental Theorem of Cyclic Groups
Every subgroup of a cyclic group is cyclic. Moreover, if |<a>l = n,
then the order of any subgroup of <a> is a divisor of n; and, for
each positive divisor k of n, the group <a> has exactly one
subgroup of order k-namely, <a n/k>.

Corollary Subgroups of 


For each positive divisor k of n, the set <n/k> is the unique
subgroup of    of order k; moreover, these are the only
subgroups of  .
Example. The list of subgroups of Z 30 is
<1> = {0, 1, 2, .... 29} order 30,
<2> = {0, 2, 4, 28} order 15,
<3> = {0, 3, 6, 27} order 10,
<5> = {0, 5, 10, 15, 20, 25} order 6,
<6> = {0, 6, 12, 18, 24} order 5,
<10> = { 0 , 10, 20} order 3,
<15> = {0, 15} order 2,
<30> = {0} order 1,

Theorem : Number of Elements of Each Order in a Cyclic


Group
If d is a positive divisor of n, the number of elements of order d in a
cyclic group of order n is φ(d).
Proof : By Theorem there is exactly one subgroup of order d-call it
<a>. Then every element of order d also generates the subgroup
<a> and, by Theorem an element a k generates <a> if and only if
gcd(k, d) = 1. The number of such elements is precisely <φ>(d).
Permutation

Transformation

A transformation of a set A is a bijection from A to A i.e. itself.


Thus a function f defined on A is a transformation of A if,
(i) f : A → A. i.e., range of f is obtained in A
(ii) f is an injection i.e. a one-one map
and (iii) f is a surjection i.e. an onto map.

Permutation : Definition
A transformation of a finite set S is called a permutation on S i.e. a
permutation is a bijection on a finite set S.
Notations: Two row notation
Let A = {a 1, a2, ..., a n} be a finite set of n distinct elements. Then it
is denoted by two row notation as follows :

In this notation, the f-image of each element of the upper row


written just below it in the lower row, because f is bijective on A.
Therefore all the n elements of A are obtained in some other or in
the lower row.

For example, a function f is defined on the set {1, 2, 3, 4} as


follows :
f(1) = 2, f(2) = 3, f(3) = 4, f(4) = 1,

then   

Equality of two permutations :

The permutation which maps every element of a set to itself, is


called identity permutation. Symbolically, it is denoted by 1.
If set A = {a 1, a2, a3......an),

then 
Examples of Permutation :

Example : If A = {a, b}, then all the functions that can be defined
from A into A are :

Evidently f1 and f2 are permutations of A but f 3 and f4 are not (being


many one).

Example : Let f =    We notice that for any


two elements a and b of 
a ≠ b ⇒ a + 2 ≠ b + 2 ⇒ f(a) ≠ f(b)
Therefore f is one one.
Again the pre-image of every element x of the codomain   is (x -
2) which exists in the domain  . Therefore f is onto.
Therefore f is a bijection on 
Consequently, f is a permutation on 

Example : Let A = {1, -1, i, -i} and f be defined as f(x) = i x for


every x ∈ A.
Then f is a permutation.
Since A is a finite set, if we write f in the standard notation as given

above, then 
As all the elements of lower row are the same as those of the
upper row and no element is repeated, f is a permutation of given
set A.

Example : Let A = {0, 1, 2, 3} and


f : A → A,
f(x) = x + 4 2

then   Clearly f is a permutation on A.

Example : Let G be a finite group and corresponding to any a of G,


a function f a is defined in G as
fa(x) = ax ∀ x ∈ G
∵ a ∈ G, x ∈ G ⇒ ax ∈ G
∴ fa : G → G
We see that f a(x) = fa(y) ⇒ ax = ay
⇒ x = y [by left cancellation law]
∴ fa is a one one map.
Again, for every element x of G, an element a -1x exist in G such
that fa(a-1x) = a(a-1x)
= (aa-1)x = ex = x
∴ fa is an onto map.
Since fa is bijection on G, therefore this is a permutation on G.

Important Remark : If any finite set A contains n elements, then


every permutation of A is of degree n and number of permutations
of A is n!.

Product or Composite of permutations

If f and g be the two permutations of any set A, then by the product


of two permutations f g we mean to find their composite function f o
g. Therefore for any x ∈ A
fog(x) = (fog)(x) = f[g(x)]

For example, if   be two


permutations of the set {1, 2, 3, 4}, then
fg(1) = fog(1) = f[g(1)] = f(3) = 4
fg(2) = fog(2) = f[g(2)] = f(4) = 1
fg(3) = fog(3) = f[g(3)] = f(2) = 3
fg(4) = fog(4) = f[g(4)] = f(1) = 2

Therefore f g = fog = 
If the order of the upper row in f is made same as the order of the
lower row in g, then the lower row of fog is the lower row of f and
upper row in fog is the lower row of f and upper row in fog is the
upper row of g. Thus
Similarly gf = 

By the calculations of f g and g f, we observe that


fg≠gf
i.e. the product of the permutations is not commutative.
Associativity : The associative law is true for the product of the
permutations i.e. f, g and h are permutations, then (f g) h = f(g h)

Inverse Permutation :
If the product of the two permutations is an identity permutation,
then they are called inverse permutation of each other.
For example : The

permutations   are
mutually inverse as fg = gf = 1
Remark. The inverse permutation can be obtained by
interchanging the rows of the given permutation.

Group of Permutations

The set of all the permutations of a given non empty set A is


denoted by S A. Therefore if A = {a, b}, then
If A = {a, b, c}, then

It can be easily seen that


O(A) = n ⇒ O(SA) = n!
In the following theorem we prove that S A is a group for the product
of permutations.
Theorem : The set SA of all permutations of a non void set A is a
group for the product of permutations.
Proof. Since the composition of two bijections is also a bijection,
so the product of any two permutations of a set will always be a
permutation of that set.
∴ f ∈ SA, g ∈ SA fg ∈ SA
i.e., SA is closed for the product of permutations.

Verification of Group axioms in S A :

 [G1] Since the composite of function is associative, therefore


for any
f, g, h ∈ SA
(fg)h = (fog)oh
= fo(goh) = f(gh)
Therefore the product of permutations is associative.
 The identity permutation l A ∈ SA is the identity for the
composition because for every
f ∈ SA
(lA f)(x) = lA[f (X)] = f(x)
and (f lA)(x) = f[lA(x)] = f(x)
 Let f ∈ SA, then
f ∈ SA ⇒ f : A → A is bijection
⇒ f-1 : A → A is also bijection
⇒ f-1 ∈ SA
which is the inverse of f because f f -1 = lA = f-1 f
Therefore the inverse of every permutation exist in S A.
From the above discussion, it is proved the S A is a group for
the product of permutations.
Permutation Group Definition

The set of all permutations S A of a non void set A is a group for the
product of permutations which is called the permutation group of A.
Remark : If A is a finite set of cardinal n i.e. consisting of n distinct
elements, then S A is a group of order n! and in that case it is
generally denoted by P A.
Note. SA is a non commutative group because the composition of
functions is not commutative.

Symmetric Group

The group of permutation of set {1, 2, ..., n} is called the symmetric


group of degree n. It is denoted by S n.,
An important Symmetric group (S 3) :
Since all the groups of order less than or equal to 5 are abelian,
S3 is the first non-abelian group. If A = {1, 2, 3}, then there will be
following 6 permutations of A :

Therefore S3 = {ρ 0, ρ1, ρ2, μ1, μ2, μ3}

Composition Table for S 3 :


Calculating all the possible product of permutations of S 3, we obtain
the following composition table :

From the above table we observe that


(i) All the elements are the members of S 3. Therefore S 3 is closed
for the product of permutations.
(ii) ρ 0 is the identity permutation in S 3.
(iii) ρ0, ρ1, ρ2, μ1, μ2, μ3 are inverse of ρ 0, ρ1, ρ2, μ1, μ2,
μ3 respectively.
(iv) The table is not symmetrical wrt the principal diagonal.
Again the product of permutations is associative.
Therefore S 3 is a finite non abelian group of order 6.

Remark. It can also be easily observed from the above


composition table of S 3 that the set {ρ 0, ρ1, ρ2} of S3 also satisfies
all the axioms of a group.
Therefore this is also a group which is called the subgroup of S 3.

Cyclic Permutations or Cycles

A permutation a of a set A is a cyclic if there exists a finite subset


{a1, a2, .... ar} of A such that: 

σ(a1) = a2, σ(a2) = a3, .... σ(ar) = a1 


Therefore σ(x) = x, if x ∈ A. But x ∉ {a1, a2, .... ar}
If σ is such a permutation, then we denote it by σ = (a 1 a2 a3 ... ar) in
which σ-image of every element is its next element and the o-image
of the last element is the first element.
Again the number of elements in this row notation is called the
length of the cycle σ.

For example, σ = (2 4 5 6) ∈ S6 is a cycle of length 4 where σ(2) =


4, σ(4) = 5, σ(5) = 6, σ(6) = 2 and σ(1) = 1, σ(3) = 3.

Thus, clearly 

Remarks :

1. The length of the identity permutation is 1 which can be


represented by an element of the set.
Example : The identity permutation of S 3 may also be
represented by (2) or (3)
2. The product of two cycles may be obtained by writing the
corresponding permutations and then multiplying by the
product method.
3. A cycle may be written in many ways by maintaining the
cyclic order e.g. (1 2 3) = (2 3 1) = (3 1 2)

Order of a Cycle
For the above cycle σ(2456) ∈ S6, we notice that
σ(2) = 4, σ2(2) = 5, σ 3(2) = 6, σ4(2) = 2.
Similarly,
σ4(4) = 4, σ4(5) = 5, σ 4(6) = 6, σ4(1) = 1, σ 4(3) = 3.
Clearly σ4 is the identity permutation which is the identity element
of S6.
∴ From the group S 6, O(σ) = 4.
Therefore it can be proved that the order of any cycle is equal to its
length i.e., the order of a cycle of length r is r.

Inverse of a Cycle
The permutation obtained by writing the elements of any
permutation σ in the reverse order is called its inverse permutation.
It is represented by σ -1.
For example, If s = (2 4 3 5), then σ -1 = (5 3 4 2)

Disjoint Cycles
Two cycles are said to be disjoint if the set of numbers that
represent the cycles have no common element.
From the definition, it is clear that those element permuted by one
is not permuted by the other. 

For example. The permutations ρ = (2 3 4) and σ = (15) are


disjoint in S 5 whereas φ = (3 5 2) and ψ = (1 2 4) are not disjoint
cycles because 2 is in both the notations φ and ψ.

Product of Disjoint Cycles


From the above definition of disjoint cycles, we have seen that if ρ
and σ are disjoint cycles of same degree, then the elements that
are permuted by ρ are not permuted by a and vice versa. Therefore
for any x,
ρ(x) ≠ x ⇒ σ(x) = x ⇒ ρσ(x) = ρ(x)
and σ(x) = x ⇒ ρ(x) ≠ x ⇒ σφ(x) = ρ(x)
∴ ρσ(x) = σρ(x)
Similarly, it can be seen that when σ(x) ≠ x, then
ρσ(x) = σρ(x)
which conclude that ρσ = σρ,
i.e. The product of any two disjoint cycles is commutative.

For example, If ρ = (125) = σ = (345) are two disjoint cycles in S 6,


then
ρσ = (1 2 5)(3 4 6)

 ...(a)
and ρσ = (3 4 6)(1 2 5)

 ...(b)
From (a) and (b), it is clear that, ρσ = σρ

Theorem : Every permutation can be expressed as the product of


disjoint cycles.
Proof. Let σ be a permutation of a finite set A. Taking any invariant
element a of A, obtain a row where first element is a, next element
is σ(a), further next element is σ(σ(a))....Since the number of
elements in A are finite, therefore after some steps, we obtain an
element whose σ-image is a. Thus this row will represent a cycle. If
all the elements of A are not included in this cycle, then taking any
element b from the remaining elements, obtain another cycle as
above.
Continue this process till all the elements of A are included in any
of these cycles. Since A is finite, therefore the number of these
cycles of σ will also be finite. Again no element is common in any
two cycles, therefore the permutation σ is expressible as the
product of some finite disjoint cycles.

Remark. In the above product of disjoint cycles, if a cycle of length


1 is obtained, then it is not written.

For example, if   then


σ = (1 4 5)(2 6 3)(7 8)

Order of a permutation

The order of a permutation of a finite set is the L.C.M. of the order


of its disjoint cycles.
We have seen that the degree of any cycle is equal to its length.
Also since every permutation can be expressed as the product of
disjoint cycles, therefore it can be easily seen that
O(f) = L.C.M. of lengths of disjoint cycles of f.
For the above permutation σ
O(σ) = L.C.M. of {O(1 4 5), O(2 6 3), O(7 8)}
= L.C.M. of {3, 3, 2} = 6.

Transposition
A cyclic permutation of length two (2) is called a transposition.

For example, the cycles (a, b), (1, 2), (2, 3) etc are transposition.
It can be easily seen that every transposition is its own inverse

Every permutation is a product of transpositions :


With the help of product of permutations, it can be easily seen that
for any cycle.
(a1 a2 a3 ... an-1 an)
(a1 a2 a3 ... an-1 an) = (a1 an)(a1 an-1) ... (a1 a3)(a1 a2)
Therefore every cycle can be expressed as the product of
transpositions, but we have seen that every permutation can be
written as the product of disjoint cycles.
Hence every permutation can be expressed as the product of
transpositions.
Example : If   then
σ = (1 4 5)(2 6 3)(7 8)
= (1 5)(1 4)(2 3)(2 6)(7 8)
Finally it should be noted that this representation as a product of
transpositions is not unique.

But it can be proved that the number transpositions in the product


will always be even or odd i.e. the number of transpositions in this
type of factorisation of any permutation σ is even (odd), then the
number of transpositions of every factorisation of the permutation σ
will also have the same even (odd).
On the basis of this nature of number of transpositions even (odd),
the even and odd permutation will be defined.

Inversion : If σ ∈ Sn, then the pair (i, j), 0 < i < j ≤ n, is an inversion
for σ if σ(i) > σ(j)
Signature : The total number of such inversions for the
permutation o is called its signature and we write it as sig σ.

Example : If a =   then 2 >1 ,4 > 1,4 >3, 6 > 5.


Therefore (13), (23), (24, (56) are the inverses of σ. Hence sig σ =
4.

Even and odd permutations

A permutation is said to be an even permutation if it can be


expressed as a product of an even number of transpositions;
otherwise it is said to be an odd permutation, i.e. it has an odd
number of transpositions.

Example : The permutation   is an even


permutation because
φ = (1 2 4)(3 5)(6 8 7 9)
= (1 4)(1 2)(3 5)(6 9)(6 7)(6 8)
= product of 6 (even) transpositions.

Example : The permutation ψ =   is an


odd even permutation because
ψ = (1 2 4)(3)(5 8 9)(6 7)
= (1 4)(1 2)(5 9)(5 8)(6 7)
= product of 5 (odd) transpositions.

Remarks :

1. A cycle of length n will be even (odd) if n is odd (even).


2. Identity permutation is an even permutation because this can
be written as the product of two transpositions (sig l A = o)
3. Every transposition is an odd permutation

Product and Inverse of Even or Odd permutations

For any two permutations α, β of any set, if α is a product of p


transpositions and β is a product of q transpositions, then clearly,
α, β will be the product of (p + q) transpositions and α -1 will be the
product of p transpositions.
Therefore if p and q both are even or both are odd, then (p + q) will
be even.
Consequently, α, β will also be an even permutation.
Similarly if p is odd (even), then (p + q) will be odd.
Consequently, α, β will be an odd permutation.
From the above discussion, we arrive at the following conclusions :

(i) The product of two even (odd) permutations is an even


permutation.
(ii) The product of one even (odd) and one odd (even) permutation
is an odd permutation.
(iii) The inverse of an even (odd) permutation is even (odd)
permutation.
Alternating group (Group of even permutation)
On the basis of the above conclusions of the product of even and
odd permutations of any set, we will show that the set of
permutations is also a group.
Theorem : The set A n of all even permutations of degree n is a
group of order (1/2)n! 

for the product of permutations.


Proof. Since the product of two even permutations is even, A n is
closed for the multiplication composition of permutations.

Verification of group axioms in A n :

 [G1] : ∴ Multiplication of permutations is associative, so also


for An.
 [G2] : ∵ The Identity permutation I is an even permutation,
therefore the identity permutation e of order n is the identity
element in A n.
 [G3] = ∵ The inverse of every even permutation is also even,
∴ α ∈ An ⇒ α-1 ∈ An which is the inverse of α.
Therefore inverse of every element exist in A n.
∴ An is a group for the product of permutations.

Order A n :
Let A n = {α1, α2...... αm} and B n = {β1, β2, .... β r}
where B n is a set of odd permutations of order n.
∴ O(An) = m and O(B n) = r
But O(An) + O(Bn) = O(Sn)
∴ m + r = n! ...(1)
If β ∈ Bn then in the product
βα1, βα2, ... βαm
each permutation will be odd and no two product will be same,
because
βαi, = βαj, ⇒ αi = αj
∴ {βα1, βα2,... βαm} ⊂ Bn
⇒ O{βα1, βα2,... βαm} ≤ O(Bn)
⇒ m ≤ r ...(2)
Similarly it can be seen that
{ββ1, ββ2, ... ββr} ⊂ An
⇒ {ββ1, ββ2, ... ββm} ≤ O(An)
⇒  r ≤ m ...(3)
∴ (2) and (3) ⇒ r = m ...(4)
Now (1) and (4) ⇒ r = m = (1/2)n!
O(An) = (1/2)n!
Note : An is called the alternating group of degree n.

Important Results :

1. When n = 3, then A 3 = {(1),(1 2 3),(1 3 2)}


2. An is a simply group for n ≥ 5
Every group of prime order is a simple group because such
group has no proper subgroup.
3. The set of odd permutations of degree n is not a group
because it is not closed for multiplication.
4. If H is a sub group of G and  , then H ∩ N need not be
normal in G.

For example, let


N = A4 = {(1), (123), (124), (132), (134), (142), (143), (234),
(243), (12), (34), (13), (24), (14) (23)}
H ={(1), (1234), (13)(24), (1432), (12)(34), (14)(23), (13)(24)}
This can be easily verified that
 and H is a subgroup of S 4.
But H ∩ N is not a normal subgroup of S 4.

5. S3/A3 is a commutative and cyclic group, being group of order


2 but S3 is non abelian and not a cyclic group.
Since A3 is normal subgroup of S 3 therefore quotient group
exist

Now   which is prime number.


⇒ Quotient group S 3/A3 is cyclic
⇒ Quotient group S 3/A3 is Abelian (Every cyclic group is
abelian)
Theorem : The alternating group A n of all even permutations of
degree n is a normal subgroup of the symmetric group S n.
i.e. 
Proof. We know that A n is a subgroup of S n
Let α ∈ Sn and β ∈ An. If α is an even permutation, then α -1 will also
be even permutation.
Hence αβα -1 will also be an even permutation.
If α is an odd permutation, then α -1 will also be odd.
∴ αβα-1 ∈ An.
If α is odd permutation, then α -1 will also be odd.
Hence  αβα-1 will be an even permutation.
Thus under all the circumstances, we see that α ∈ Sn, β ∈ An ⇒
αβα-1 ∈ An.
∴ An is a normal subgroup of S n i.e,

Example : Find fg and gf when :

(a) fg = 

= (2 3)

and 

= (1 2)

(b) fg = 
= (1 2 3 5 4)

and gf = 

= (1 2 4 3 5)

Example : If α = 
Compute the following: (a) α -1 (b) αβ-2 (c) O(α)

(a) α-1

(b) 

∴ 

Therefore 

(c) ∵ 
∴ O(a) = L.C.M. of {O(1 2 3), O(4 5)}
= L.C.M. of {3, 2} 
Example : If σ = (1 7 2 6 3 5 8 4) and ρ =   
then prove that :
ρσρ-1 = (ρ(1) ρ(7) ρ(2) ρ(2) ρ(6) ρ(3) ρ(5) ρ(8) ρ(4))

∵  σ = (1 7 2 6 3 5 8 4)

∴ 

 ...(1)

Again 
∴ ρσρ-1 = (ρσ)ρ-1

= (2 6 5 7 4 8 1 3)
= (ρ(1) ρ(7) ρ(2) ρ(6) ρ(3) ρ(5) ρ(8)ρ(4))

Example : If ρ =   σ = (1 3 4)(5 6)(2 7 8 9)


-1
then find σ  ρσ and by expressing the permutation ρ as the
product of disjoint cycles, find whether ρ is an even
permutation or odd permutation. Also find its order.
σ = (1 3 4)(5 6)(2 7 8 9)

∴   ...(1)

Again 

 ...(2)
-1 -1
σ ρσ = σ (ρσ)

= (1 8 4 2 9 7) (3 5 6)

Again 
= (1 7 2 8 3 9)(4 6 5)
= (1 9)(1 3)(1 8)(1 2)(1 7)(4 5)(4 5)
= product of 7 (odd) transpositions.
Since ρ is equal to the product of odd transpositions, therefore
this is a odd permutation.
Finally, O(p) = L.C.M. of {O(1 7 2 8 3 9), 0(4 6 5)}
= L.C.M. of {6, 3}
=6

Example : Express the following permutations as the product


of disjoint cycles. Also find whether these permutations are
even or odd.
(a) 
(b) ψ = (1 2 3 4 5)(1 2 3)(4 5) ∈ S5

(a) We see that


φ = (1 2 4)(3 5)(6 8 7 9)
Again ( 1 2 4) = (1 4)(1 2)
and (6 8 7 9) = (6 9)(6 7)(6 8)
∴ φ = (1 4)(1 2)(3 5)(6 9)(6 7)(6 8)
= product of 6 (even) transpositions.
Therefore φ is an even permutation.
(b) We see that
ψ = (1 2 3 4 5)(1 2 3)(4 5)

= product of 3 (odd) transpositions.


Therefore ψ is an odd permutation.

Example : Show that the set of the permutations (a), (ab), (cd),
(ab) (cd) of the set {a, b, c, d} forms a group for the product of
permutations.

Let (a) = f1, (ab) = f2, (cd) = f3, (ab)(cd) = f4 and G = {f 1, f2, f3, f4}
Calculating the products of permutations of G,
f1 f2 = f2, f1 f3 = f3, f1, f4 = f4, f2 f1 = f2, f3 f1 = f3, f4 f1 = f4
f2 f2 = (ab)(ab) = f1
f2 f3 = (ab)(cd) = f4
f2 f4 = (ab)(ab)(cb) = (cd) = f 3
f3 f2 = (ad)(ab) = (ab)(cd) = f 4
f3 f3 = (cb)(cd) = f 1
f3 f4 = (cd)(ab)(cd) = (ab) f 2
f4 f2 = (ab)(cd)(ab) = (cd) f 3
f4 f4 = (ab)(cb)(ab)(cd) = f 1
From the above products, the composition table of G is :

Observing the table, it is clear that :


(i) All its elements are in G, therefore G is closed for the
multiplication of permutations.
(ii) f1 is the identity element.
(iii) Every element is inverse of itself.
Again the multiplication of permutation is associative,
therefore this is associative in G also.
Hence G is a group for the multiplication of permutation s.

Example : Show that the set A 3 = {(a), (a b c), (a c b )} of even


permutations of the set {a, b, c} is a group for the product of
permutations.

Calculating the possible product in A 3


 (a)(a) = (a),  (a)(a b c) = (a b c),  (a)(a c b) = (a c b)
 (a b c)(a) = (a b c)(a b c);  (a b c) = (a c b),  (a b c)(a c b) = (a)
 (a c b)(a) = (a c b)(a c b);  (a b c) = (a),  (a c b)(a c b) = (a b c)

Therefore, the composition table of A 3 is as follows:

Observing the table,

1. All its elements are in A 3,


Therefore A 3 is closed for the multiplication of
permutations
2. The identity element (a) exists.
3. The inverses of (a), (abc), (acb) are respectively (a),
(acb), (abc).

Again multiplication of permutations is associative, there fore it


is in A3 also. Hence A 3 is a group for the multiplication of
permutation.

Example : Prove that the set P n of all permutations of degree n


is a group of order n! for the product of permutations.

Let S = {a1, a2, ..., an} be a finite set of n elements. There fore
every permutation of S is of n degree. Since n elements of S
can be arranged in n! ways, there fore the total number of
permutations is equal to the number of permutations n! of
degree n. Let Pn be the set of all the permutations of degree n.

Let   be any two


permutations of degree n. Since b 1, b2, .... bn and c1, c2, .... cn,
are only different arrangements of n elements of S, there fore
elements of first row (keeping the column sun altered) in g are
arranged in the same order in which the elements appear in
the second row of f. Now by multiplication of

permutations, 
which is a permutation of degree n.
Therefore g ∈ Pn, f ∈ Pn ⇒ g f ∈ Pn
∴ Pn is closed for the multiplication of permutation.

Verification of group axioms in P n :

[G1] Let 
be any three permutations of degree n where
b1, b2, ..., bn; c1, c2 ... cn; d1, d2, ... dn
are different arrangements of elements of S
Then 

∴ 

and g f 

∴ h(g f) = 
Thus (hg)f = h(gf)
which shows that the multiplication of permutations is
associative.

[G2] Let e =   
be the identity permutation of degree n, therefore e ∈ Pn such
that

and 
Therefore the identity permutation of degree n exist in P n.

[G3] Let 

Then 

and 
Therefore f-1 is the inverse of f wrt multiplication of
permutations.
Therefore inverse of every element exist in P n.
From the above discussion, P n is a group for the multiplication
of permutations. Again since the total number of permutations
in Pn is n!. Therefore this is a group of order n!.

Example : If A = {a, b, c, d} and S A is the permutation group of


A, then prove that T a = {ρ ∈ SA; ρ(a) = a} is a sub group of S A.

By definition, Ta contains those permutations of S A where the


image of a is a. Therefore T a will contain the following
permutations :
ρ1 = (a). ρ2 = (bed), ρ3 = (bde)
ρ4 = (be), ρ5 = (cd), ρ6 = (bd)
∴ TA = {ρ1, ρ2, ρ3, ρ4, ρ5, ρ6}
Multiplying the permutations, the composition table of T a is as
follows :

Observing the table, it is clear that all its elements are in T a.


Therefore Ta is closed for the multiplication of permutations
i.e., x ∈ Ta, y ∈ Ta ⇒ xy ∈ Ta
Therefore by theorem T a is a subgroup of S A.

Example : Find the quotient group G/H and also prepare its
operation table when G = S 3 and H = A 3. 

We know that
S3 = {(1), (123), (132),(12), (23), (32)}
and A3 = {(1), (123), (132)}
We have earlier proved that 
Therefore S 3/A3 exist.
The following cosets of A 3 are obtained in S 3 :
(1) A3 = A3(1) ={(1)(1), (123)(1), (132)(1)}
= {(1), (123), (132)} = A 3.
(123)A3 = A3(123) = {(1), (123), (123)(123), (132)(123)}
= {(123), (132), (1)} = A 3.
(132)A3 = A3(132) = {(1)(132), (123)(132), (132)(132)}
= {(132), (1), (123)} = A 3.
(12)A 3 = A3(12) = {(1),(12), (123)(12), (132)(12)}
= {(12), (13), (23)}
(23)A 3 = A3(23) = {(1) (23), (123) (23), (132) (23)}
= {(23), (12),(31)}, = A 3(12).
(31)A 3 = A3(31) = {(1)(31), (123)(31), (132)(31)}
= {(31 ),(23),(12)} = A3(12).
Thus we see that
G/H = S3/A3 = {A3, A3}(12)}
Its composition table is as under :

Dihedral Group

Another special type of permutation group is the dihedral group.


Such group consist of the rigid motions of a regular n-sided
polygon or n-gon. For n = 3, 4, ..., we define the nth dihedral group
to be the group of rigid motions of a regular n-gon. We will denote
this group by D n. We can number the vertices of a regular n-gon by
1, 2, ..., n (Figure). Notice that there are exactly n choice to replace
the first vertex. If we replace the first vertex by k, then the second
vertex must be replaced either by vertex k + 1 or by vertex k - 1;
hence, there are 2n possible rigid motions of the n-gon. We
summarize these results in the following theorem.

Theorem : The dihedral group, D n, is a subgroup of S n of order 2n.

Theorem : The group D n, n > 3, consists of all products of the two


elements r and s, satisfying the relations
rn = e
s2 = e  
srs = r-1.

Proof : The possible motions of a regular n-gon are either


reflections or rotations (Figure). Therefore are exactly n possible
rotations:

We will denote the rotation 360°/n by r. The rotation r generates all


of the other rotations. That is,
rk = k.(360°/n)
Label the n reflections s 1, s2, .... sn, where sk is the reflection that
leaves vertex k fixed. There are two cases of reflection, depending
on whether n

Types of reflections of
a regular n-gon

is even or odd. If there are an even number of vertices, then 2


vertices are left fixed by a reflection. If there are an odd number of
vertices, then only a single vertex is left fixed by a reflection
(Figure). In either case, the order of s k is two. Let s = s 1. Then s2 =
e and rn = e. Since any rigid motion t of the n-gon replaces the first
vertex by the vertex k, the second vertex must be replaced by
either k + 1 or by k - 1. If the second vertex is replaced by k + 1,
then t = r k-1. If it is replaced by k - 1, then t = r k-1 s. Hence, r and s
generate Dn; that is, Dn consists of all finite products of r and s. We
will leave the proof that srs = r -1 as an exercise.
Example : The group of rigid motions of a square, D, consists of
eight elements. With the vertices numbered 1, 2, 3, 4 (Figure), the
rotations are
r = (1234)
r2 = (13)(24)  
r3 = (1432)
r4 = e
and the reflections are
s1 = (24)
s2 = (13).
The order of D 4 is 8. The remaining two elements are
rs1 = (12) (34).

The Motion Group of a Cube


We can investigate the group of rigid motions as geometric objects
other than a regular n-sided polygon to obtain interesting examples
of permutation groups. Let us consider the group of rigid motions of
a cube. A cube has 6 sides. If a particular side is facing upward,
then there are four possible rotations of the cube that will preserve
the upward-facting side. Hence, the order of the group is 6.4 = 24.
We have just proved the following proposition.

Proposition : The group of rigid motions of a cube contains 24


elements.
Theorem : The group of rigid motions of a cube is S 4.

Proof : From Proposition, we already know that the motion group


of the cube has 24 elements, the same number of elements as
there are in S 4. There are exactly four diagonals in the cube. If we
label these diagonals 1, 2, 3, and 4, we must show that the motion
group of the cube will give us any permutation of the diagonals
(Figure). If we can obtain all of these permutations, then S 4 and the
group of rigid motions of the cube must be the same. To obtain a
transposition we can rotate the cube 180° about the axis joining the
midpoints of opposite edge (Figure). There are six such axes,
giving all transpositions in S 4. Since every element in S 4 is the
product of a finite number of transitions, the motion group of a cube
must be S 4.

Cosets
Coset of H is G

Let G be a group and H a subgroup of G. For any a ∈ G, the set


{ah I h ∈ H} is denoted by aH. Analogously, Ha = {ha I h ∈ H} and
aHa-1 = {aha-1 | h ∈ H}. When H is a subgroup of G, the set aH is
called the left coset of H in G containing a whereas Ha is called the
right coset of H in G containing a. In this case, the element a is
called the coset representative of aH(or Ha). We use |aH| to denote
the number of elements in the set aH, and |Ha| to denote the
number of elements in Ha.
Remarks : 1. In general aH ≠ Ha but in an abelian group G, aH =
Ha, ∀ a ∈ G
Sometimes the cosets (left and right) are equal even if the group is
not abelian.
If the composition of the group G is addition (+), then aH and Ha
are written as a + H and H + a respectively.

Example : Let G = (Z, +) and H = 2Z = {0, ±1, ± 2, ± 4, ...} then the


right cosets of H in G are:
H + 0 = {0, ±2, ±4, ...,} = H
H + 1 = {0 + 1, ±2 + 1, ±4 + 1, ...}
H + 2 = {0 + 2, ±2 + 2, ±4 + 2, ...} = H etc.
It can be easily observed that any right coset and its corresponding
left coset are equal i.e.,
H + 1 = 1 + H, H + 2 = 2 + H etc.
Again H + 3 = {..., - 6, - 3, - 1, 1, 3, 7, 9, ...} = H + 1
H + 4 = {..., - 5 , - 2 , 0, 2, 4, 6, 8, 10, ...} = H + 2 etc.
Similarly H + 5 = H + 1,H + 6 = H
and H + (-1) = H + 2, H + (-2) = H + 1 etc.
Thus H has only two distinct cosets H, H + 1 in Z.
Clearly, G = H ∪ (H + 1)

Example : If H = {1, - 1 } and G = [{1, - 1 , i, - i } , x] then different


cosets of H in G are as follows:
1H = H1 = {1-1, -1-1} = {1, -1} = H
-1H = H(-1) = {1(-1), -1 (-1)} = {-1, 1} = H
iH = Hi = { 1 i , -1 -i} = { i , - i}
- iH = H (-i) = {1 *(-i). -1(-i)} = {-i, i)
Example : Klein’s 4 group :
The subgroup {e, a} of the group {e, a, b, c} has right cosets {e, a}
and {b, c} which are left cosets also.
Similarly, cosets (right and left) of the subset {e, b} are {e, b} and
{a, c} and {e, c} and {a, b} are cosets of the sub group {e, c}.

Properties of cosets

Theorem : If H is a subgroup of a group G and a ∈ G then :


a ∈ aH and a ∈ Ha
Proof. Let e be the identity element of G so also of H.
Then for every a ∈ G, e ∈ H ⇒ ae = a ∈ aH
and e ∈ H ⇒ ea = a ∈ Ha
Remark. From the above theorem, it is clear then
aH ≠ φ, Ha ≠ φ, ∀ a ∈ G
Theorem : If H is a subgroup of a group G, then G is equal to the
union of all left (right) cosets of H, i.e.

Proof : ∵ a ∈ G ⇒ a ∈ aH [by above Theorem]

 ...(1)
Again since aH ⊂ G, V a ∈ G

∴   ...(2)

(1) and (2) ⇒ G = 

Similarly it can also be proved that 


Theorem : If H is a subgroup of a group G, then for any a, b ∈ G :
(a) aH ∼ bH and Ha ∼ Hb
(b) aH ~ Ha
(A ~ B mean that there exist one to one correspondence between A
and B)
Proof : (a) In order to establish one to one correspondence
between aH and bH, we shall show that there exists one one onto
mapping between aH and bH.
Define a mapping f from aH to bH as follows :
f : aH → bH, f(ah) = bh, ∀ b ∈ H
For h1, h2 ∈ H, we observe that
f(ah1) = f(ah 2) ⇒ bh1 = bh2
⇒ h1 = h2 [by cancellation law in G]
⇒ ah1 = ah2
∴ f is one one.
Again x ∈ bH ⇒ H contains an element h such that x = bh
⇒ ah ∈ aH such that f(ah) = bh = x
∴ f is onto.
Therefore f is a bijection.
Consequently, aH ~ bH i.e., there is a one to one correspondence
between any two left cosets of a H in G.
Similarly this can be proved that Ha ~ Hb also.
(b) Define a mapping f from aH to Ha as follows :
f : aH → Ha, f(ah) = ha, ∀ h ∈ H
For h1, h2 ∈ H, we have
f(ah1) = f(ah 2) ⇒ h1a = h2a
⇒ h1 = h2 [by cancellation law in G]
⇒ ah1 = ah2
f is one one.
Again for each ha ∈ Ha, aH contains an element ah such that
f(ah) = ha
∴ f is onto.
which proves that aH ~ Ha.

Corollary 1. There is one to one correspondence between H and


every left (right) coset of H.
H ~ aH, H ~ Ha, a ∈ G
i.e., there exist one to one correspondence between H and left
(right) cosets of H.
Since H is a left and right coset of H itself, therefore by the above
theorem
H ~ aH and H ~ Ha, ∀ a ∈ G

Corollary 2. If H is a finite subgroup of a group G, then :


O(aH) = O(H) = O(Ha), ∀ a ∈ G
The function from the set aH to the set bH given by ah ↦ bh is
clearly onto. It is one-to-one since bh 1 = bh2 ⇒ h1 = h2 ⇒ ah1 = ah2.
So |aH| = |bH|.
Theorem : Any two left (right) cosets of a subgroup are either
identical or disjoint.
Proof. Let H be a subgroup of a group G having two left cosets aH
and bH which are not disjoint.
To Prove : aH = bH.
∵ aH ∩ bH = φ ⇒ there exist atleast one common element in bH
and aH.

Let x ∈ aH and x ∈ bH, where x = ah 1, x = bh 2, h1, h2 ∈ H.


Now x = ah 1 = bh2 ⇒ a = bh 2h1-1 and b = ah1h2-2 ...(1)
Therefore for any h ∈ H
ah ∈ aH ⇒ ah = (bh2h1-1) h
= b(h2h1-1h) ∈ bH [∵ h2h1-1 h ∈ H]
∴ aH ⊂ bH  ...(2)
Again bh ∈ bH ⇒ bh = (ah 1h2-1)h
= a (h1h2-1h) ∈ aH [∵ h2h1-1 h ∈ H]
∴ bH ⊂ aH ...(3)
(2) and (3) ⇒ aH = bH
Therefore aH ∩ bH ≠ φ ⇒ aH = bH
i.e., any two left cosets of H are identical if they are not disjoint.
Similarly this property can also be proved for the right cosets.

Example : Let G = S3 and H = {(1), (13)}. Then the left cosets of H


in G are:
(1) H = H,
(12) H = {(12), (12)(13)} = {(12), (132)} = (132)H,
(13) H = {(13), (1)} = H,
(23) H = {(23), (23)(13)} = {(23), (123)} = (123)H.

Lemma : Properties of Cosets


Let H be a subgroup of G, and let a and b belong to G. Then,

 a ∈ aH,
 aH = H if and only if a ∈ H,
 aH = bH or aH ∩ bH = θ,
 aH = bH if and only if a -1b ∈ H,
 |aH| = |bH|,
 aH = Ha if and only if H = aHa -1
 aH is a subgroup of G if and only if a ∈ H.

Lagrange's Theorem And Consequences

Theorem : Lagrange’s Theorem : |H| Divides |G|

If G is a finite group and H is a subgroup of G, then |H| divides |G|.


Moreover, the number of distinct left (right) cosets of H in G is
IGI/IH.

Or
For any finite group G, the order (number of elements) of every
subgroup H of G divides the order of G.

Proof : Let a1H, a2H, .... arH denote the distinct left cosets of H in
G. Then, for each a in G, we have aH = a iH for some i. Also, by
property 1 of the lemma, a ∈ aH. Thus, each member of G belongs
to one of the cosets a iH. In symbols, G = a 1H ∪ ... ∪ arH.
Now, property 3 of the lemma shows that this union is disjoint, so
that |G| = |a1H| + |a2H| + ... + |arH|.
Finally, since |a iH| = |H| for each i, we have |G| = r|H|.
Hence, the result follows.

OR

Proof : Suppose there are m cosets of H in G. Since G is the


disjoint union of them and each coset has |H| elements, it follows
that |G| = m.|H| and so |H| divides |G|.
Note : The converse of Lagrange’s Theorem is not true in general,
it is true only for cyclic groups.

Corollary 1 : |a| Divides |G|


In a finite group, the order of each element of the group divides the
order of the group:
Proof: Recall that the order of an element is the order of the
subgroup generated by the element.

Corollary 2 : Group of Prime Order Are Cyclic


A group of prime order is cyclic.
Proof : Suppose that G has prime order. Let a ∈ G and a ≠ e.
Then, |<a>| divides |G| and |<a>| ≠ 1. Thus, |<a>| = |G| and the
corollary follows.

Corollary 3 : a |G| = e
Let G be a finite group, and let a ∈ G. Then, a|G| = e.
Proof : By Corollary 1, |G| = |a|k for some positive integer k.
Thus, a |G| = a|a|k = ek = e.

Corollary 4 : Fermat’s Little Theorem


Let p be a prime number, and a any integer. Then a p - a is always
divisible by p. This can also be written as a p ≡ a (mod p).

Proof by induction:

First we will show that the theorem is true for all positive integers a
by induction. The base case (when a = 1) is obviously true: 1 P ≡ 1
(mod p).
By the binomial theorem,

But   because p|p! but   and   So


the middle terms vanish mod p:
(a + 1)p = (ap + 1) (mod p).
Substituting a p ≡ a (mod p) by the inductive hypothesis gives (a +
1)p ≡ (a + 1) (mod p)
So the result holds for all positive a by induction. But every integer
is congruent mod p to a positive integer, so the result holds for
every integer.
Corollary 5 : Every group of prime order is cyclic

Proof : Let G be a group of order p, where p is prime.


Now O(G) = p prime ⇒ p > 1
⇒ G has atleast one element other than the identity.
Therefore let a ∈ G, a ≠ e. If H = [a], then
O(H) | O(G) [by Lagrange’s theorem]
⇒ O(H) | p
⇒ O(H) = p or O(H) = 1 [∵ p is prime]
⇒ O(H) = p = O(H) [∴ H ≠ {e}]
Similarly H ⊂ G and O(H) | O(G)
⇒ G = H = [a]
Therefore G is a cyclic group with its every element as generator
except the identity. 

Corollary 6. A group of prime order has no proper subgroup.


Proof. Let G be a group of order p, where p is prime.
If H a subgroup of G order m, then by Lagrange’s theorem m is a
divisor of p. But p is prime, therefore m = p or 1.
Hence H = G or H = {e}
which are improper subgroup of G.
As such G has no proper subgroup.

Index of a subgroup. Definition :


Let H be a subgroup of a group G. Then the number of the distinct
right (left) cosets of H in G, is known as the index of H in G.
This is denoted by [G : H].
If G is a finite group and H has k different left (right) cosets in G,
then by the Lagrange’s theorem.
O(G) = k.O(H)

⇒ 
⇒ O(G) = O(G)[G:H]
For example, If H = {1, - 1} and G = [{1, - 1, i - i}, x], then [G : H] =
2 because the number of distinct cosets of H is 2.
Euler’s φ-function. Definition :
The Euler’s φ-function. φ(n), in defined for n ∈ N by
(1) φ (1) = 1
(2) For n > 1, φ(n) = the number of positive integers less than n and
relatively prime to n.

Theorem Euler’s Theorem :


If n ∈ N and any integer a is relatively prime to n, then a φ(n) = 1
(mod n), where φ is an Euler φ- function.
Proof. For any integer x, let
[x] = Residue class of the set of integers mod n and G = {[a] : a is
an integer relatively prime to n}
We know that residue class G is a group of order φ(n) wrt
multiplication and its identity element is residue class [1].
Now [a] ∈ G ⇒ [a]o(G) = [1]
⇒ [a]φ(n) = [1]
⇒  [a][a][a]...(φ(n) times) = [1]
⇒ [a.a.a. ...(φ(n) times]) = [1]
⇒ [aφ(n)] = [1]
⇒ aφ(n) = 1 (mod n)

Theorem Fermat’s Theorem :


If p is a prime and a is any integer, then a p ≡ a (mod p)
Proof. Let G = ({[1], [2], [3],..., [p - 1]}, x p) = Set of non zero
residue class of integers modulo p
If p is a prime number, then G is a group wrt multiplication of
residue classes whose order is (p - 1) and identity element is [1].
Now let a be an integer.

Case I. p | a (p is a divisor of a)


a ≡ 0 (mod p) i.e., [a] = [0]
So [a] is not an element of G.
But p | a ⇒ p | ap ⇒ p | (a p - a)
⇒ ap ≡ a (mod p)

Case II. p   a (P is not a divisor of a) :


In this case [a] ≠ [0]
So [a] is an element of G.
Therefore [a] o(G) = [1] ⇒ [a]p-1 = [1]
⇒ [ ap-1] = [1]
⇒ ap-1 = 1 (mod p)
⇒ p | (ap-1 - 1)
⇒ p | a(ap-1 - 1)
⇒ p I (a p - a) ⇒ ap ≡ a(mod p)

Theorem : If H is a subgroup of a group G and a, b ∈ G, then :


(a) Ha = Hb ⇔ ab-1 ∈ H
(b) aH = bH ⇔ b-1 a ∈ H
Proof. (⇒) Let Ha = Hb, then
Ha = Hb ⇒ Hab-1 = Hbb-1
⇒ Hab-1 = He = H
⇒ ab-1 ∈ H [∵ ab-1 ∈ H(ab -1) = H]
Conversely : (⇐) Let ab -1 ∈ H
then ab -1 ∈ H ab-1 [∵ x ∈ Hx, ∀ x ∈ H]
∴ ab-1 ∈ H and ab -1 ∈ H ab-1
⇒ Hab-1 = H [∵ any two right cosets are either identical or disjoint]
⇒ Hab-1b = Hb
⇒ Ha = Hb
∴ Ha = Hb ⇔ ab-1 ∈ H
Similarly (b) can also be proved.

Corollary : (a) Ha = H ⇔ a ∈ H
(b) aH = H ⇔ a ∈ H
Proof. (a) Let Ha = H, then
Ha = H ⇒ Ha = He
⇒ ae-1 ∈ H [by the above theorem]
⇒a∈H
Again a ∈ H ⇒ ae-1 ∈ H
⇒ Ha = He [by the above theorem]
∴ Ha = H ⇔ a ∈ H  
Similarly (b) can also be proved.

Relation of congruence modulo w.r.t. a subgroup in a group


Let H be a subgroup of a group G and a, b ∈ G, then a is said to be
“congruent mod H”, to b if ab -1 ∈ H.
It is denoted by a = b(mod H)

Theorem : The relation of congruency in a group G, defined below,


is an equivalent relation, where H is a subgroup of G:
a ≡ b(mod H) ⇔ ab-1 ∈ H
Proof : (i) Reflexive : Let a ∈ G, then
aa-1 = e ∈ H ⇒ a ≡ a(mod H)
∴ the relation is reflexive.
(ii) Symmetric : Let a ≡ b (mod H). then
a ≡ b (mod H) ⇒ ab-1 ∈ H
⇒ (ab-1)-1 i H [∵ H is a subgroup]
⇒ ba-1 ∈ H
⇒ b ≡ a (mod H).
∴ the relation is symmetric.
(iii) Transitive : Let a = b (mod H) and b ≡ c (mod H). then a ≡
b(mod H), b ≡ c(mod H) ⇒ ab-1 ∈ H and bc-1 ∈ H
⇒ (ab-1)(bc -1) ∈ H
⇒ a(b-1b)c-1 ∈ H
⇒ ac-1 ∈ H
⇒ a ≡ c(mod H)
Therefore the relation is transitive.
Therefore the above congruence relation defined in G is an
equivalence relation.
As such this partitions G into mutually disjoint equivalence classes.
We now show that corresponding to the equivalence class of any a
∈ G i.e., C(a) = {x ∈ G | x ≡ a (mod H)} is some as right coset Ha,
because
x ∈ C(a) ⇒ x ≡ a (mod H) ⇒ xa-1 ∈ H
⇒ xa-1 a ∈ Ha ⇒ x ∈ Ha
∴ C(a) ⊂ Ha ...(1)
and x ∈ Ha ⇒ xa-1 ∈ Haa -1 = H
⇒ x ≡ a (mod H ) ⇒ x ∈ C(a)
Ha ⊂ C(a) ...(2)
(1) and (2) ⇒ C(a) = Ha, ∀ a ∈ G
Remark. Similarly it can also be proved that the relation in G
defined by a ≡ b (mod H) ⇔ a-1 b ∈ H is also an equivalence
relation which partitions the group G into mutually disjoint
equivalence classes wrt H. In this case the equivalence class c(a)
corresponding to a ∈ G is the left coset aH corresponding to a.

Example : Find all the cosets of 3  in the group ( , +).

Let H = 3  = {..., 6, -3, 0, 3, 6, ...}.


The following distinct cosets of H in G are obtained :
0+ H = H + 0= {..., -6, -3, 0, 3, 6, ...} = H
1 + H = H + 1 = {..., -5, -2, 1, 4, 7, ...}
2 + H = H + 2 = {...,  -4, -1, 2, 5, 8, ...}
3 + H = H + 3 = {..., -3, 0, 3, 6, 9, ...} = H + 0
4 + H = H + 4 = {..., -2, 1, 4, 7, 10, ...} = H + 1
5 + H = H + 5 = {..., -1, 2, 5, 8, 11, ...} = H + 2
6 + H = H + 6 = {..., 0, 3, 6, 9, ...} = H + 0

-1 + H = H + (-1) = {..., -7, -4, -1, 2, 5, ...} = H + 2


-2 + H = H + (-2) = {..., -8, -5, -2, 1, 4, ...} = H + 1
-3 + H = H + (-3) = {..., -9, -6, -3, 0, 3, ...} = H + 0

On observing the above cosets, it is clear that


H + 0 = H + 3 = H + 6 = H + 9 . . . = H + (-3) = H + (-6)...
H + 1 = H + 4 = H + 7 = H +10 . . . = H + (-2) = H + (-5)...
H + 2 = H + 5 = H + 8 = H + 11 ... = H + (-1) = H + (-4)...
Therefore three cosets of H in G are the following :
H, H + 1, H + 2

Remark : There are n cosets of 

Example : Find all the cosets of H = {0, 4} in the group G


Here G = {0, 1, 2, 3, 4, 5, 6, 7} and H = {0, 4}, G


The following distinct cosets of H in G are obtained :
0 + H = H + 0 = {0, 4} = H
1 + H = H + 1 = {0 + 8 1, 4 +8 1} = {1, 5}
2 + H = H + 2 = { 0 + 8 2, 4 +8 2} = {2, 6}
3 + H = H + 3 = {0 + 8 3, 4 +8 3} = {3, 7}
4 + H = H + 4 = {0 + 8 4, 4 +8 H} = {4, 0} = H
5 + H = H + 5 = {0 + 8 5, 4 +8 5} = {5, 1} = H + 1
6 + H = H + 6 = {0 + 8 6, 4 +8 6} = {6, 2} = H + 2
7 + H = H + 7 = {0 + 8 7, 4 +8 7} = {7, 2} = H + 3
On observing the above cosets, it is clear that four coset of H
in G are :
H, H + 1, H + 2, H + 3.

Example : If H is a subgroup of a group G, then prove that T =


{x ∈ G I xH = Hx} is a subgroup of G.

∵ e ∈ G ⇒ eH = H = He ⇒ e ∈ T ⇒ T ≠ Φ
∴ Let x1, x2, ∈ T; then by the definition of T,
x1H = Hx1 and x2H = Hx2
then   

Again   
 [∵ x2 ∈ T]
-1
= H(x1x2 )
∴ x1x2-1 ∈ T
Similarly x 1 ∈ T, x2 ∈ T ⇒ x1x2-1 ∈ T
∴ T is a sub group of G.

Example : If H is a subgroup of a group G and g ∈ G, then


prove that :
(a) gHg -1 = {ghg-1 | h ∈ H} is a subgroup of G.
(b) If H is finite, then O(H) = O(gHg -1).

(a) Let x = gh 1g-1 ∈ gHg -1


and y = gh 2g-1 ∈ gHg -1 where h 1h2 ∈ H
then xy-1 = (gh1g-1)(gh2g-1)-1
= (gh1g-1)(g-1)-1, h2-1g-11
= (gh1g-1)(gh2-1g-1)
= gh1(g-1g)h2-1g-1 [by associativity]
= gh1eh2-1g-1
= g(h1h2-1)g-1 ∈ gHg-1 [∵ h1h2-1 ∈ H]
Similarly x ∈ gHg -1, y ∈ gHg -1 ⇒ xy-1 ∈ gHg -1
∴ gHg-1 is a subgroup of G.
(b) Define a map f from H to gHg -1 as follows :
f : H → gHg -1, f(h) = ghg -1, ∀ h ∈ H.
∵ f(h1) = f(h2) ⇒ gh1g-1 = gh2g-1
⇒ h1 = h2 [by cancellation law]
∴ f is one one.
Again for every ghg -1 ∈ gHg-1, there exist h ∈ H such that
f(h) = ghg -1
∴ f is onto.
Hence f is a bijection which proves that O(H) = O(gHg -1).

Example : If H and K are two finite subgroups of a group G


such that H ⊂ K then prove that : [G : H] = [G : K] [K : H]

Let H and K be two subgroups of a group G and H ⊂ K,


∴ H is also a subgroup of K.
Now by definition of index of a subgroup in a group.
[G:H] = O(G)/O(H) ...(1)
[G:H] = O(G)/O(K) ...(2)
[K:H] = O(K)/O(H) ...(3)
From (2) and (3),

= [G : H] [by (1)]

Example : If H is a subgroup of a group G with index 2 i.e., I G :


H I = 2, then prove that aH = Ha, ∀ a ∈ G.

Since H is a subgroup of index 2 of a group G, therefore the


number of distinct left (right) cosets of H in G is 2.
Since H itself is a left and right coset, therefore let
G = H ∪ Ha, where H ∩ Ha = φ ...(1)
Again if H and bH are two disjoint left cosets, then
G = H ∪ bH ...(2)
From (1) and (2), G = H ∪ Ha = H ∪ bH
⇒ G - H = Ha = bH [∵ H ∩ Ha = F = H ∩ bH]
Now ∵ a ∈ Ha
∴ a ∈ Ha, Ha = bH ⇒ a ∈ bH
⇒ bH = aH [∵ a ∈ aH]
∴ Ha = bH = aH
∴ Ha = aH, ∀ a ∈ G

Example : If H and K are two finite subgroups of an abelian


group G, then prove that :

O(HK) = 

Let D = H ∩ K and Dk 1, Dk2,..., DKi


be all the distinct right cosets of D in K. Therefore

 ...(1)

Now   [∵ D ⊂ H ⇒ HD = H] ...(2)


Again we see that any two cosets of Hk 1, Hk2, .... Hk t are
pairwise disjoint because
Hki = Hkj ⇒ ki . kj-1 ∈ H
⇒ kj. kj-1 ∈ H ∩ K = D [∵ kikj-1 ∈ K]
⇒ Dki = Dkj
which is a contradiction to our supposition. Thus Hk 1, Hk2......
Hki are all distinct.
Therefore by (2)
O(HK) = O(Hk 1) + O(Hk 2) + ... + O(HK i)
= t . O(H) [∵ O(Hki) = O(H), ∀ i]

 [by (1)]
Example : If H is a subgroup of a group G and a ∈ G, then
prove that :
(Ha) -1 = a-1H

Let ha ∈ Ha, where h ∈ H then (ha) -1 ∈ (Ha) -1,


but (ha) -1 = a-1h-1 ∈ a-1 H [∵ h ∈ H ⇒ h-1 ∈ H]
(Ha) -1 ⊂ a-1 H ...(1)
Again if a -1 h ∈ a-1 H, then
a-1h = a-1(h-1)-1 = (h-1a)-1 ∈ (Ha) -1 [∵ h-1 ∈ H]
∴ a-1 H ⊂ (Ha) -1
From (1) and (2), (Ha) -1 = a-1H

Example : If H is a subgroup of a group G, then show that any


two right cosets Ha and Hb are distinct iff left cosets a -1 H and
b-1 H are distinct.

If possible, let Ha and Hb be the two right cosets of H in G


when left cosets a -1H and b -1H are equal, then
a-1H = b-1H ⇒ (b-1)-1 a-1 ∈ H [∵ aH = bH ⇒ b-1 a ∈ H]
⇒ a-1b ∈ H
⇒ (ba-1)-1 ∈ H [∵ H is a sub group]
⇒ (a-1)-1 b-1 ∈ H ⇒ ab-1 ∈ H
⇒ Ha = Hb [∵ ab-1 ∈ H ⇒ Ha = Hb]
which is a contradiction. Therefore
Ha ≠ Hb ⇒ a-1 H ≠ b-1 H
Conversely, it can also be proved that
a-1H ≠ b-1H ⇒ Ha ≠ Hb
Therefore Ha ≠ Hb ⇔ a-1H1b-1H.
Morphism

In many fields of mathematics, morphism refers to a structure-


preserving map from one mathematical structure to another. The
notion of morphism recurs in much of contemporary mathematics.
In set theory, morphisms are functions; in linear algebra, linear
transformations; in group theory, group homomorphisms; in
topology, continuous functions, and so on.
In category theory, morphism is a broadly similar idea, but
somewhat more abstract: the mathematical objects involved need
not be sets, and the relationship between them may be something
more general than a map, although has to behave similarly to
maps, e.g. has to admit associative composition.
The study of morphisms and of the structures (called "objects")
over which they are defined is central to category theory. Much of
the terminology of morphisms, as well as the intuition underlying
them, comes from concrete categories, where the objects are
simply sets with some additional structure, and morphisms are
structure-preserving functions. In category theory, morphisms are
sometimes also called arrows.

Some special morphisms

A morphism f of a group G into G’ is called :

1. Monomorphism, If f is injection (one one).


2. Epimorphism, If f is surjection (onto).
Here G’ is called the homomorphic image of the group G. 
3. Isomorphism, If f is bijection (one one onto). 
4. Endomorphism, If G’ = G i.e., f is a homomorphism from G to
itself.
5. Automorphism, If G’ = G and f is bijection.

Group Homomorphism

Homomorphism
A mapping f from a group (G, *) to a group (G', o) is called a group
homomorphism (or group morphism) from G to G’ if
f(a * b) = f(a) o f(b), ∀ a, b ∈ G.
Thus, if f is a morphism from G to G', then it preserves the
composition in both the groups G and G’ i.e.,

Example : Let  +) be the additive group of real numbers and 


*) be the multiplicative group of non zero real numbers.
The mapping f :  is a homomorphism
of   into   because for any x 1, x2 ∈ 

Various Morphisms

A morphism f of a group G into G’ is called

(i) Monomorphism, if f is injection (one one).


(ii) Epimorphism, if f is surjection (onto).
Here G’ is called the homomorphic image of the group G.
Example : Let G = {1, -1} be a multiplicative group. Then the map f
: (Z, +) → (G, x)

is an epimorphism from   to G, because for any x 1, x2 ∈ 

1. when x1, x2 both are even, then f(x 1 + x2) = 1 = 1.1 = f(x 1).f(x2)
2. when x1 is even and x 2 is odd, then f(x 1 + x2) = -1 = 1 (-1) =
f(x1).f(x2)
3. when x1 is even and x 2 is odd, then f(x 1 + x2) = -1 = 1.(-1) =
f(x1).f(x2)
4. when x1 is odd and x 2 is even, then f(x 1 + x2) = -1 = -1.1 = f(x 1)
f(x2)
Thus f(x1 + x2) = f(x1).f(x2), ∀ x1,x2 ∈ Z
Again f(Z) = G ⇒ f is onto.

Hence f is an epimorphism from Z onto G.

(iii) Isomorphism, if f is bijection (one one onto).


Example : For any group G, the identity mapping l G is an
isomorphism of G onto itself, because for any x 1,x2 in G and lG is
obviously a bijection.

(iv) Endomorphism, if G’ = G i.e. f is a homomorphism from G to


itself.
Example : The map f :   is a
homomorphism from   to itself because for x 1, x2 ∈ 
f(x1 + x2) = 2(x1 + x2) = 2x1 + 2x2 = f(x1) + f(x2)
(v) Automorphism, if G’ = G and f is bijection.

Properties of homomorphism

Theorem : If f is a homomorphism from a group G to G’ and if e


and e’ be their respective identities, then :

(a) f(e) = e'


(b) f(a-1) = [f(a)] -1, ∀ a ∈ G
Proof : (a) Let a ∈ G, then ae = a = ea
⇒ f(ae) = f(a) = f(ea)
⇒ f(a) f(e) = f(a) = f(e) f(a) [∵ f is homomorphism]
⇒ f(e) is the identity in G’ ⇒ f(e) = e'
Therefore the image of the identity of G under the group morphism
f is the identity of G’.
(b) Let a-1 be the inverse of a ∈ G, then
aa-1 = e = a-1 a ⇒ f(aa-1) = f(e) = f(a -1a)
⇒ f(a) f(a -1) = e’ = f(a -1) f(a) ⇒ f (a-1) = [f(a)] -1
therefore, the f-image of the inverse of any element of G under is
the inverse of the f-image of a in G'.

Theorem : If f is a homomorphism of a group G to a group G’, then


(a) H is a subgroup of G ⇒ f(H) is a subgroup of G'.
(b) H’ is subgroup of G’ ⇒ f-1(H’) = {e ∈ G | f(x) ∈ H’} is a subgroup
of G.
Proof : (a) Clearly f(H) ⊂ G’ and f(H) = φ, because
e ∈ H ⇒ f(e) = e’ ∈ f(H) where e’ is identity in G’
If a’, b’ ∈ f(H), then a’, b’ ∈ f(H) ⇒ there exist a, b in H such that
f(a) = a’ and f(b) = b’
⇒ a’(b’) -1 = f(a)[f(b)] -1
= f(a) f(b-1) [∵ [f(b)] -1 = f(b-1)]
= f(ab-1) [∵ f is homomorphism]
But a ∈ H, b ∈ H ⇒ ab-1 ∈ H
⇒ f(ab-1) ∈ f(H)
Thus a’, b ∈ f(H) ⇒ f(ab-1) = a’(b’) -1 ∈ f(H)
∴ f(H) is a subgroup of G’
(b) Obviously f-1(H’) ⊂ G and f-1(H’) ≠ φ because atleast e ∈ f-1 (H’)
If a, b ∈ f-1(H’), then
a, b ∈ f-1(H’) ⇒ f(a) ∈ H’ and f(b) ∈ H'
⇒ f(a)[f(b)] -1 ∈ H’ [∵ H is a subgroup]
⇒ f(a) f(b -1) ∈ H'
⇒ f(ab-1) ∈ H’ [∵ f is homomorphism]
⇒  ab-1 ∈ f-1 (H’)
Thus a, b ∈ f-1(H') ⇒ ab-1 ∈ f-1 (H’)
∴ f-1(H’) is a subgroup of G

Corollary : If f is a homomorphism from a group G to G \ then f(G)


is a subgroup of G’.
This can be easily proved by taking H = G in part (a) of the above
theorem.

Kernel of Homomorphism
Let f be a homomorphism of a group G into G', then the set K of all
those elements of G are mapped to the identity e’ of G’ is called the
kernel of the homomorphism f.
It is denoted by Ker f or Ker (f)
Ker f = {x ∈ G | f(x) = e’}

Example : The mapping f :


is a homomorphism of   because for z 1, z2 ∈ 
f(z1 z2) = |z1 z2| = |z1| |z2| = f(z1) f(z2)
Again Ker (f) = {z ∈ C0 | f(z) = 1}
= {z ∈ C0 ||z| = 1}

Example : The mapping f :   is a


homomorphism on  , because for any x 1, x2 ∈ 
f(x1x2) = (x1x2)4 = x14.x24 = f(x1).f(x2) and
Ker(f) = {x ∈   | f(x) = 1}
= {x ∈   | x4 = 1} = {1 -1}

Example : The mapping f : 

Again
Example : If f :   f(x + iy) = x, then f is a homomorphism
from to  because for any (x 1 + iy1), (x2 + iy2) ∈ 
f[(x1 + iy1) + (x2 + iy2)] = f[(x1 + x2) + i(y1 + y2)]
= x1 + x2
= f(x1 + iy1) + f(x2 + iy2)
Again Ker (f) = {(x + iy) ∈   | f(x + iy) = 0}
= {(x + iy) ∈   | x = 0}
= the set of imaginary numbers.

Example : Let S n be the symmetric group of degree n and G = {1,


-1} multiplicative group, then the mapping f defined by

is a homomorphism of S n onto G and Ker(f) = A n


(Alternating group i.e., the set of even permutations of degree n)

Theorem : [First theorem on homomorphism]

If f is a homomorphism from a group G to G’ with kernel


then  .
Proof : Let e and e' be the identities of G and G’ respectively
Ker(f) K = {x ∈ G | f(x) = e’} ⊂ G.
∵ f(e) = e’ ⇒ e ∈ K ⇒ K ≠ 4 φ
∵ Let a, b ∈ K; then f(a) = e’ and f(b) = e’.
Again f(ab -1) = f(a) f(b -1) [∵ f is homomorphism]
= f(a)[f(b)] -1
= e’(e’) -1 = e’e' = e’
ab-1 ∈ K
Thus we see that a ∈ K, b ∈ K ∈ ab-1 ∈ K
Therefore the Ker(f) is a subgroup of G.
 : Let x ∈ G and a ∈ K, then f(xax -1) = f(x).f(a) f(x -1) [∵ f is
homomorphism]
= f(x)e’[f(x)] -1 [∵ a ∈ K ⇒ f(a) = e’]
= f(x) [f(x)] -1 = e'
∴ x ∈ G, a ∈ K ⇒ xax-1 ∈ K.
Therefore 
Theorem : If f is a homomorphism of a group G onto a group and g
is a homomorphism of G’ onto a group G", then gof is a
monomorphism of G onto G”.
Also the kernel of f is a subgroup of that of gof.

Proof : Since f : G → G’ and g : G’ → G” are onto mappings so are


composite mapping g o f : G → G” exist and it is also onto.
Again for any a, b ∈ G
(gof)(ab) = g[f(ab)]
= g[f(a) f(b)] [∵ f is homomorphism]
= g[f(a)].g[f(b)] [∵ g is homomorphism]
= (gof)(a).(gof)(b)
∴ gof is also a group homomorphism from G to G”.
If K and K’ are kernel’s of f and gof respectively, then K = {x ∈ G |
f(x) = e’}, where e’ is the identity of G’
and K = {x ∈ G | (gof) (x) = e”}, where e’’ is the identity of G”
By theorem, K and K’ are subgroups of G.
K is a subgroup of K’ : Let x ∈ K.,
then x ∈ K ⇒ f(x) = e’
⇒ g[f(x)] = g(e’) = e”
⇒ (gof)(x) = e” ⇒ x ∈ K’
∴ K ⊂ K’.
which shows that K is a subgroup of K'.

Theorem : A homomorphism f defined from a group G to a group


G’ is a monomorphism iff Ker f = {e}, where e is the identity of G.
Proof : (⇒) : First suppose that f is a monomorphism of G onto G
and K is the kernel of f. If a ∈ K, then
a ∈ K ⇒ f(a) = e’, where e’ is the identity of G'
⇒ f(a) = f(e) [∵ f(e) = e’]
⇒ a = e [∵ f is one one]
or K = {e}

Conversely (⇐) : Suppose f is a homomorphism from G to G’ and


kernel of f i.e. K = {e}. If a, b ∈ G, then
f(a) = f(b) ⇒ f(a) [f(b)] -1 = f(b)[f(b)] -1
⇒ f(a) f(b -1) = e’
⇒ f(ab-1) = e’ ⇒ ab-1 ∈ K
⇒ ab-1 = e [∵ K = {e}]
⇒a=b

∴ f is one one mapping.


Therefore f is monomorphism ⇔ Ker f = {e}.

Theorem : Every homomorphic image of an abelian group is


abelian but not conversely.
Proof. Let f be a homomorphism of an abelian group G to a group
G’. By theorem, the homomorphic image f(G) of G is a subgroup of
G'.
f(G) is abelian :
Let a’, b’ ∈ f(G), then there exist a, b in G such that
f(a) = a’ and f(b) = b'.
Now a’b’ = f(a) f(b)
= f(ab) [∵ f is homomorphism]
= f(ba) [∵ G is commutative]
= f(b) f(a)
= b’ a'
Thus a’, b’ ∈ f(G) ⇒ a’ b’ = b’ a’
∴ f(G) is also an abelian group.
Conversely (⇐) : The converse is not necessarily true as can be
seen from the following example :
f : S3 → {1,-1}

is a homomorphism of S 3 onto {1, -1}


Moreover the multiplicative group f(S 3) = {1, -1} is abelian cyclic,
but S 3 is neither abelian not cyclic.

Theorem : Every homomorphic image of a cyclic group is cyclic but


not conversely.
Proof : Let f be a homomorphism of a cyclic group G = [a] to a
group G\ By theorem f(G) is a subgroup of G’.
f(G) is cyclic :
Let x ∈ f(G), then x = f(a n), where n ∈ Z
Again f(a n) = f(aaa.... (n times))
= f(a) f(a)...f(a) (n times) [∵ f is homomorphism]
= [f(a)] n
which shows that every element of f(G) is some integral power of
f(a)
i.e. f(G) = [f(a)]
Hence f(G) is also a cyclic group.

Conversely (⇐) : The converse is not necessarily true as can be


seen from the following example :
f :S3 → {1,-1}

is a homomorphism of S 3 onto {1, -1}


Moreover the multiplicative group f(S 3) = {1, -1} is abelian cyclic,
but S 3 is neither abelian nor cyclic.

Natural morphism : Definition :

Let  , then f : G → G/N s.t. f(x) = Nx, ∀ x ∈ G is called a


natural morphism of canonical mapping or projection mapping of G
onto G/N. 

Theorem : Every group is homomorphic to its quotient group.


Proof : Let N be a normal subgroup of a group G.
Consider a mapping p from G to G/K defined as :
p : G → G/N, p(x) = Nx, ∀ x ∈ G.
We see that Na ∈ (G/N), there exists a e G such that p(a) = Na
∴ p is onto.
Again for any a, b ∈ G
p(ab) = Nab = NaNb = p(a) p(b).
∴ p is a epimorphism of G onto G/N.

Corollary. If p is a homomorphism of G onto G/N defined as


above, then Ker p = N.
Proof : As proved above, the mapping p : G → G/N, p(x) = Nx, ∀ x
∈ G is a homomorphism of G onto G/N. Let Ker (p) = K, then K = {x
∈ G | p(x) = N} [N is the identity in G/N]
K=N:
If x ∈ K ⇒ px = N
⇒ Nx = N ⇒ x ∈ N
∴ K ⊂ N ...(1)
Again, if x ∈ N ⇒ p(x) = Nx
⇒ p(x) = N [∵ x ∈ N ⇒ Nx = N]
⇒x∈K
N ⊂ K ...(2)
(1) and (2) ⇒ K = N.

Theorem : Let f : G → G’ be a morphism of groups. Let  ⊂ K,


the kernel of f; then there exists a unique morphism
φ : G/N → G' such that f = φ o p
Proof : Let Ker f = K = N, therefore N ⊂ L ...(1)

Let p be a canonical map


defined by
p : G → G/N, p(x) = xN. ...(2)
Let φ : G/N → G', φ(xN) = f(x) ...(3)
φ is well defined :
∵ x1 N  = x2 N ⇒ x2-1 x1 ∈ N ⊂ K
⇒ f(x2-1x1) = e' [e' ∈ G']
⇒ f(x2-1)f(x1) = e'
⇒ f(x1) = f(x2)
⇒ f(x1N) = f(x2N)
Therefore φ is well defined,
and (φ o p)x = φ[p(x)] x ∈ G
= φ(xN) = f(x)
∴f=φop

φ is a morphism :
f(x1 Nx2 N) = f(x1 x2 N)
= f(x1 x2)
= f(x1) f(x2)
= φ(x1 N)φ(x2 N)
∴ φ is a morphism.

Corollary : Ker
Proof : Let x ∈ K then φ(x N) = f(x) = e’
∴ x N ∈ Ker φ
⇒ (K/N) ⊂ Ker φ ...(i)
and x N ∈ Ker φ ⇒ f(x N) = e'
⇒ f(x) = e’
⇒ x ∈ Ker f

⇒ Kerφ ⊂ K/N ...(ii)

(i) and (ii) ⇒ Ker f = 

Corollary : Image φ = Image f


Proof : x ∈ Image φ ⇒ x = f(g), g ∈ G
⇒ Image φ ⊂ Image f
and f(g) = φ(g N)

Image φ = Image f

Theorem : Every homomorphic image of a group G is isomorphic


to some quotient group of G.
Proof: Let G’ be a homomorphic image of a group G and f be the
corresponding onto morphism from G onto G'.
If K is the kernel of f, then   
Hence G/K is a quotient group of G.

To prove that G / K = G'.:


Define a map φ from G/K to G’ as follows :
φ : G/K → G', f(Kx) = f(x), x ∈ G.
φ is well defined :
i.e., Ka = Kb ⇒ φ(Ka) = φ(Kb), a, b ∈ G
We see that Ka = Kb ⇒ ab-1 ∈ K
⇒ f(ab-1) = e', where e' is identity in G’
⇒ f(a) f(b -1) = f(a) [f(b)] -1 = e’
⇒ f(a) = f(b)
 ⇒ f(Ka) = φ(Kb)
∴ φ is a well defined mapping.
Again φ(Ka) = f(Kb) ⇒ f(a) = f(b)
⇒ f(a) [f(b)] -1 = f(b) [f(b)] -1 = e
⇒ f(a) f(b) -1 = e’
⇒ f(ab-1) = e’
⇒ ab-1 ∈ K  [∵ K is the kernel of f]
⇒ Ka = Kb [∵ Ka = Kb ⇔ ab-1 ∈ K]
∴ φ is one one.

φ is onto :
Lastly, if a ∈ G’ then a ∈ G such that
f(a) = a’ [∵ f is onto]
Hence Ka ∈ G/K such that
ψ(Ka) = f(a) = a’
∴ φ is onto

φ is a homomorphism : Now for any Ka, Kb ∈ G/K


φ[Ka Kb] = φ [Kab] = f(ab) = f(a) f(b) = φ(Ka) φ(Kb)
∴ φ is a morphism from G/K to G.
Therefore φ is an isomorphism from G/K to G’.
Hence G/K ≅ G’.

Example : If Sn be the symmetric group of degree n and G =


[{1, -1}, x], then show that the mapping f : S n → G defined by
is a homomorphism of S n onto G. Also find its kernel.

Let α, β ∈ Sn. Here four cases arise :


Case 1. When α, β will also be an even permutation :
In this case αβ will also be an even permutation.
Therefore f(αβ) = 1 =1.1 = f(α).f(β)
Case 2. When α, β both are odd permutations :
In this case αβ will be an even permutation.
Therefore f(αβ) = 1 = (-1).(-1) = f(α).f(β)
Case 3. When is α even and β is odd permutation :
In this case αβ will be an odd permutation.
Therefore f(αβ) = -1 = 1.(-1) = f(α).f(β)
Case 4. When α is odd and β is even permutation :
In this case αβ will be an odd permutation.
Therefore f(αβ) = -1 = (-1).1 = f(α).f(β)
Thus we see that f(αβ) = f(α).f(β) ∀ α, β ∈ Sn
Therefore f is a morphism from S n to G.
By definition, Ker(f) = {α ∈ Sn | f(α) = 1}
= {α ∈ Sn | (α) is even permutation}
= group of even permutations of degree n.
= An.

Example : If   denote the multiplicative group of non-zero


real numbers, then show that the
mapping   is a homomorphism in  .
Also find its kernel.

Let x1, x2 ∈  . then


f(x1x2) = (x1x2)4 = x14x24 = f(x1).f(x2)
∴ f is a morphism in 
Again kernel of 
or ker f = {1, -1}

Example : If f is a homomorphism of a group G to G’ with


kernel K and a ∈ G, a' ∈ G’ are such that f(a) = a’; then prove
that the set of those elements of G which are mapped to a’ is
the coset Ka of K in a

We have to prove that f -1(a’) = {x ∈ G | f(a) = a’} = Ka


Let e and e’ be the identities of G and G’ respectively,
Let y ∈ f-1(a’) then f(y) = a’ and
f(ya-1) = f(y) f(a -1)
= a’[f(a)] -1
= a’(a’) -1 = e’  
∴ ya-1 ∈ K ⇒ y ∈ Ka
Thus y ∈ f-1(a’) ⇒ y ∈ Ka ⇒ f-1(a’) ⊂ Ka ...(1)
Again, let x ∈ Ka, then
x ∈ Ka ⇒ x = ka where k is any element of K
⇒ f(x) = f(ka) = f(k) f(a) = e’a’ = a’
⇒ x ∈ f-1(a’)
∴ Ka ⊂ f-1(a’) ...(2)
(1) and (2) ⇒ f-1(a’) = Ka.

Example : If f is a homomorphism of a group G to Gr with


kernel K then prove that for any a, b ∈ G; 
f(a) = f(b) ⇔ ab-1 ∈ K

For any a, b ∈ G
f(a) = f(b) ⇔ f(a)[f(b)] -1 = f(b)[f(b)] -1 = e' where e’ is the identity of G
⇔ f(a) f(b-1) = e’
⇔ f(ab-1) = e’ [∵ f is a morphism]
⇔ ab-1 ∈ K
A homomorphism φ from a group G to a group   is a mapping
from G into   that preserves the group operation; that is, φ(ab) =
φ(a)φ(b) for all a, b in G.

Kernel of a Homomorphism
The kernel of a homomorphism φ from a group G to a group with
identity e is the set {x ∈ G | φ(x) = e}. The kernel of φ is denoted by
Ker φ.
Example : Any isomorphism is a homomorphism that is also onto
and one-to-one. The kernel of an isomorphism is the identity.
Example : Let G = GL(2,   and let   be the group of non-zero
real number under multiplication. Then the determinant mapping A
→ det A is a homomorphism from G to  . The kernel of the
determinant mapping is SL(2,  .

where

GL(2, R) : is the group of non-singular real-valued matrices under


multiplication. It contains all non- singular transformations,
including expansions or contractions, reflections, and translations.
SL(2, R) : is the group of determinant 1 real-valued matrices under
multiplication. Since the determinant of a product is the product of
the determinants, this is a closed group. It contains all rotations
without any reflections. This is all rotation-translations, and it
preserves  
Also OL(2, R) : is the group of orthogonal matrices, whose
transposes are also their inverses. Obviously their determinant will
be +1 (As det(A) 2 = det(A)det(A T) = det(AA T) = det(AA -1) = det(ld) =
1).
Also SOL(2, R) : is the group of special orthogonal matrices,
whose transposes are also their inverses and determinant is +1. It
is the group of rotation matrices or equivalently, the unit circle.

Properties of Homomorphisms

Theorem : Properties of Elements under Homomorphism’s


Let φ be a homomorphism from a group G to a group   and let g
be an element of G. Then

1. φ carries the identity of G to the identity of  .


2. φ(gn) = (φ(g)) n.
3. If |g| = n, then |φ(g)| divides n.
4. If φ(g) = g', then φ -1(g') = {x ∈ G | φ(x) = g’} = gKer φ.
Theorem : Properties of Subgroups under Homomorphism
Let be a homomorphism from a group G to a group   and let H be
a subgroup of G. Then

1. φ(H) = {φ(h) | h ∈ H} is a subgroup of  .


2. If H is cyclic, then φ(H) is cyclic.
3. If H is Abelian, then φ(H) is Abelian.
4. If H is normal in G, then φ(H) is normal in φ(G).
5. If |Ker φ| = n, then φ is an n-to-1 mapping from G onto φ(G).
6. If |H| = n, then lφ(H)| divides n.
7. If   is a subgroup of  , then φ-1( ) = {k ∈ G | φ(k) ∈  } is a
subgroup of G.
8. If   is a normal subgroup of   then φ-1( ) = {k ∈ G | φ(k) ∈ 
} is a normal subgroup of G.
9. If φ is onto and Ker φ = {e}, then φ is an isomorphism from G
to  .

Kernels Are Normal

Let φ be a group homomorphism from G to  . Then Ker φ is a


normal subgroup of G.

The First Isomorphism Theorem


Theorem : Frist Isomorphism Theorem
Let φ be a group homomorphism from G to  . Then, the mapping
from G/Ker φ to φ(G), given by φKer φ → φ(g), is an isomorphism.
In symbols, G/Ker φ ≈ φ(G).
Proof : Let us use ψ to denote the correspondence gKer φ → φ(g).
First, we show that ψ is well defined (that is, the correspondence is
independent of the particular coset representative chosen).
Suppose xKer φ = yKer φ. Then y -1x ∈ Ker φ and e = φ(y -1x) =
(φ(y))-1φ(x). Thus φ(x) = φ(y), and ψ is indeed a function. To show
that y is operation-preserving, observe that ψ(xKer φ yKer φ) =
ψ(xyKer φ) = φ(xy) = φ(x)φ(y) = ψ(xKer φ)ψ(yKer φ). Finally, if
ψ(g1 Ker φ) = ψ (g 2Ker φ), then φ(g 1) = φ(g2), so that g2-1g1, ∈ Ker
φ. It follows that ψ is one -to -one.
Example : 
Consider the mapping from   defined in Example. Clearly, its
kernel is <n>. So, by First Isomorphism Theorem  .

Theorem : Normal Subgroup Are Kernels


Every normal subgroup of a group G is the kernel of a
homomorphism of G. In particular, a normal subgroup N is the
kernel of the mapping g → gN from G to G/N.

Proof: Define φ:G → G/N by φ(g) = gN. (This mapping is called the


natural homomorphism from G to G/N.) Then, φ(xy) = x(xy)N =
xNyN = <φ(x)φ(y).

Isomorphism : Definition :

A morphism f of a group (G, *) to a group (G', o) is an isomorphism


if

1. f is one-one i.e., f(a) = f(b) ⇒ a = b, f is a morphism i.e., f(a*b)


= f(a) o f(b), V a,b e G a, b ∈ G
2. f is onto i.e, f(G) = G’
3. f is a morphism i.e., f(a*b) = f(a) o f(b), ∀ a, b ∈ G

From the above definition, it is clear that a group morphism is an


isomorphism if f is a bijection.

Isomorphic Groups

A group G is said to be isomorphic to a group G \ if there exists an


isomorphism of G onto G’. Symbolically, we write it as G ≅ G’.

Examples of Isomorphism

Example : For every group G, the identity mapping l G defined by


lG: G → G, lG(x) = x, ∀ x ∈ G
is an isomorphism of G onto itself, because l G is clearly one-one
onto and for a, b ∈ G, lG(ab) = ab = l G(a) lG(b)
Example : The map f :    is an
isomorphism from Z onto 2 , because for any x 1, x2 ∈ 

1. f(x1 + x2) = 2(x1 + x2) = 2x1 + 2x2 = f(x1) + f(x2)


∴ f is a group morphism.
2. f(x1) = f(x2) ⇒ 2x1 = 2x2 ⇒ x1 = x2
∴ f is one-one.
3. f(Z) = 2 ∴ f is onto.

Hence Z ≅ 2 .

Example : The map f :   is an


isomorphism, because for x 1, x2 ∈ 

1. f(x1 x2) = log(x 1 x2) = log x1 + log x2 = f(x1) + f(x2)


2. f(x1) = f(x2) ⇒ log x1 = log x2 ⇒ x1 = x2
3. f(R+) = R

Properties of Isomorphism

Theorem : A homomorphism f defined from a group G onto G’ is an


isomorphism iff Ker(f) = {e}.
Proof : (⇒): Suppose f is an isomorphism of G onto G’ and K is the
kernel of f. If a ∈ G, then  
a ∈ K ⇒ f(a) = e’, where e’ is the identity of G
⇒ f(a) = f(e) [∵ f(e) = e’]
⇒ a = e [∵ f is one-one]

This shows that K contains only the identity e i.e., ∴ K = {e}

Conversely (⇐) : Suppose that K = {e}.


Let a, b ∈ G, then
f(a) = f(b) ⇒ f(a) [f(b)] -1 = f(b)[f(b)] -1
⇒ f(a) f(b -1) = e’ ⇒ f(ab-1) = e’
⇒ ab-1 ∈ K
⇒ ab-1 = e [∵ K = {e}]
⇒a=b
∴ f is bijective homomorphism.
Hence it is an isomorphism.

Theorem : The relation of isomorphism '≅' in the set of group is an


equivalence relation.
Proof : (i) Reflexive : For any group G, the identity mapping
lG defined by l G(x) = x is an isomorphism because l G is a bijection
and for any a, b ∈ G, lG(ab) = ab = l G(a) lG(b)
∴G≅G
⇒ relation ≅ is reflexive.

(ii) Symmetric : Let G and G’ be two groups such that G ≅ G and


let f be the corresponding isomorphism. Since by definition f is a
bijection, so its inverse f -1 : G' → G exists and it is also a bijection.
Further if a, b ∈ G and a', b’ ∈ G’ such that
f(a) = a’ and f(b) = b'.
then a = f -1 (a’) and b = f -1 (b’)
then f-1 (a’b’) = f -1 [f(a) f(b)] [by (1)]
= f-1 [f(ab)]
= ab
= f-1(a’) f-1(b’)
Therefore f -1 is an isomorphism from G' onto G.
Hence G’ ≅ G.
∴ G ≅ G’ ⇒ G' ≅ G
Therefore the relation ≅ is symmetric.

(iii) Transitive : Let G, G’ and G” be three groups such that


G ≅ G’ and G’ ≅ G”
Also let f and g be their respective isomorphism. Since by definition
f and g are bijections, so gof : G → G” is also a bijection. Further,
gof is a homomorphism of G to G”. Hence gof is an isomorphism
from G to G”.
∴ G ≅ G”
Therefore the relation ≅ is transitive.
From the above discussion, the relation of isomorphism '≅’ is an
equivalence relation.
Remark : The relation of isomorphism ≅ in a family of groups being
an equivalence relation, partitions that family into disjoint
equivalence classes. If G ≅ G’ i.e., if G and G’ belong to the same
equivalence class, then we simply say that G and G’ are
isomorphic. When two groups are isomorphic, then their structures
will be basically identical. In such a case, we sometimes say that
two groups are abstractly identical.

Theorem : Cayley Theorem :

Every group is isomorphic to some permutation group or G ≅ PA.


Proof : Let G be a group. Corresponding to every a in G, we define
a map fa as follows :
fa(x) = ax, ∀ x ∈ G

∵ a ∈ G, x ∈ G ⇒ ax ∈ G
∴ fa : G → G is a function
Further for any x, y ∈ G
fa(x) = fa(y) ⇒ ax = ay
⇒ x = y [by cancellation law in G]
∴ f is one-one and for every x ∈ G, there exists a -1x ∈ G such that
fa(a-1 x) = a(a-1 x) = (aa -1)x = x
∴ fa is onto.
As such fa is a one one mapping of G onto G itself.
Hence fa is a permutation of G.
Let S G be the group of all permutations of G.
Let us now consider the mapping φ from G to S G, defined by φ : G
→ SG, φ(x) = f x, ∀ x ∈ G.
Now for any x, y ∈ G
φ(xy) = f xy = fxfy = φ(x) φ(y)
∴ φ is a homomorphism from a group G onto S G.
Consequently φ(G) = G’ is a subgroup of the permutation group
SG and φ is an epimorphism from G onto G’.
Also for any a, b ∈ G
φ(a) = φ(b) ⇒ fa = fb
⇒ fa(x) = fb(x), ∀ x ∈ G
⇒ ax = bx ⇒ a = b
∴ φ is one-one.
Hence φ is an isomorphism from a group G onto permutation group
G'. Consequently, G ≅ G’.

Theorem : Every infinite cyclic group is isomorphic to the additive


group of integers i.e.  +).
Proof. Let G = [a] be an infinite cyclic group generated by a.
First we show that no two distinct integral powers of a can be
equal.
For if possible, let a i = aj where i, j ∈   i > j, then
ai = aj ⇒ ai a-j = aj a-j
⇒ ai-j = a0 = e
⇒ O(a) is finite.
⇒ G is finite which is a contradiction.
∴ G = {...a-3, a-2, a-1, a0 = e, a, a2, a3,...} has all distinct elements.
Consider the mapping f from the group G to the group   defined
as

f : G → Z, f(a n) = n, ∀ an ∈ G
We see that for any a m, an ∈ G
am ≠ an ⇒ m ≠ n ⇒ f(am) ≠ f(an)
∴ f is one-one
and for any n ∈  , there exists a" such that f(a n) = n
∴ f is onto.
Again, f(am an) = f(am+n) = m + n = f(a m) + f(a n)
∴ f is a homomorphism from the group G to Z
Hence f is an isomorphism of G onto   which proves thatG ≅ 

Remark. Since the relation of isomorphism is an equivalence


relation, so with the help of theorem, we conclude that any two
infinite cyclic groups are isomorphic to each other. Hence we can
say that there exists one and only one infinite cyclic group.

Example : If m is a fixed integer then show that the following


mapping f is an isomorphism from 
For any x1 x2 ∈ 
f(x1) = f(x2) ⇒ mx1 = mx2 ⇒ x1 = x2
∴ f is one-one and for any mx ∈ mZ there exists x ∈ Z such
that
f(x) = mx
∴ f is onto.
Again f(x1 + x2) = m(x1 + x2) = mx1 + mx2 = f(x1) + (x2)
∴ f is homomorphism
Thus we see that f is a homomorphism from the group   onto
m  which is bijective.
Consequently, 

Example : Show that the additive group   of real numbers is


isomorphic to the multiplicative group   of positive real
numbers.

Consider the mapping f from the group    to group   


defined as

We know that for any x 1, x2 ∈ 

∴ f is one-one.
and for every   there exist log   such that
f(log x) = elog x = x
∴ f is onto.
Again, f(x 1 + x2) = 
∴ f is homomorphism.
As such f is a homomorphism from the group   to the
group   which is bijection. Hence f is an isomorphism.
Consequently 

Example : Prove that the following groups are isomorphic


groups :
(a) ({1, - 1}, x ) and ({0, 1}, + 2)
(b) ({0, 1, 2, 3},+ 4) and ({1, 2, 3, 4}, x 5)

(a) Let G = {1, -1} and G’ = {0, 1}


Define a map f from G to G' as f(1) = 0, f(-1) = 1.
Clearly f is bijection.
The composition table of both the groups G and G’ are as
below :

 
In table II, if we replace 0 by 1 and 1 by -1, then the two
tables become identical. This show that f preserves
composition and is a bijection.
Therefore f is isomorphism.
Hence G ≅ G’
(b) Let G = {0, 1, 2, 3} and G’ = {1, 2, 3, 4}
Define a map f from G to G’ as
f(0) = 1, f(1) = 2, f(2) = 3 and f(3) = 4
Clearly f is bijection. The composition table of both the groups
are :

Replacing 1, 2, 4, 3 by 0, 1, 2, 3 respectively in the table II,


the two tables become identical.
This shows that f preserves composition and is a bijection.
Therefore f is isomorphism.
Hence G ≅ G’
Example : Prove that the following group of permutations (G’)
are isomorphic to multiplicative group G = {1, ω, ω 2}.
(a) G’ = ({(1), (123), (132)}, o)
(b) G’ = ({e, (abc), (acd)}, o)

(a) Let G {1, ω, ω 2} and G’ = {(1),(123),(132)}


Define a mapping f from G to G’ as follows :
f(1) = (1), f(ω) = (123) and f(ω 2) = (132).
Clearly f is bijection. The composition table of both the groups
are:

 
Replacing (1), (123), (132) by 1, ω and ω2 respectively in the
table II, the two tables become identical.
This shows that f preserves composition and is a bijection.
Therefore f is isomorphism.
Hence G ≅ G’
(b) Define a mapping f from G to G’ as follows :
f(1) = e, f(ω) = (abc) and f(ω 2) = (acb)
Clearly, f is bijection. The composition table of the group G’
is :

Replacing e, (abc) and (acb) by 1, ω and ω 2 respectively in


the above table of G \ the two tables of group G and G\ the
two tables of group G and G’ become identical. This show that
f preserves composition and is a bijection.
Therefore f is isomorphism.
Hence G ≅ G'
Example : Find the permutation group isomorphic to the group
({1, -1, i, -i} ,x)

Let G = {1, -1, i, -i}. Define a map f for a ∈ G as


fa : G → G, f a(x) = ax, ∀ x ∈ G
Then we shall have

Hence by Cayley’s theorem, the permutation group


isomorphic to G is G' = {f 1, f(-1), f(i), f(-i)}
It can be easily seen that the corresponding isomorphism φ is
defined as φ(x) = f x, ∀ x ∈ G.
Note. The group G’ is generally known as the Regular
permutation group isomorphic to the group G.

Example : Prove that a finite cyclic group of order n is


isomorphic to the additive group of residue classes modulo n.
Solution : Let G = [a] = {a, a 2, a3 ..., an = e} be a cyclic group of
order n. Let G ’ be the additive group of residue classes modulo n.
Then G’ = {[0], [1], [2],...,[n - 1]}
Define a map f : G → G’.f(a r) = [r], ∀ ar ∈ G
Clearly f(a n) = [n] = [0]. We see that for any a r, as ∈ S
f(ar) = f(as) ⇒ [r] = [s] ⇒ r = s ⇒ ar = as
∴ f is one one.
and O(G) = n = O(G’) and f is one-one ⇒ f is onto
Again f(a r.as) = f(ar+s) = [r + s] = [r] + [s] = f(a r) + f(as).
Therefore f is an isomorphism from G to G'.
Hence G ≅ G’.
Automorphism
An isomorphic mapping of a group G onto G itself is called an
automorphism of G.

It is denoted by A G or Aut (G)


A mapping f : G → G is called automorphism of G if
(i) f is a bijection
and (ii) f(ab) = f(a) f(b), ∀ a, b ∈ G

Example : The identity mapping l G is an automorphism on G


because lG is a bijection and l G(ab) = ab = l G (a) lG (b), ∀ a, b ∈ G

Example : The mapping f : (C, +) → (C, +), f(z) =   ∀ z ∈ G is an


automorphism on C because for any z 1, z2 ∈ C

∴ f is one-one.
and for every z ∈ C there exists   ∈ C such that f( ) = ( ) = z
∴ f is onto
Again 
Therefore f is an isomorphism on the group C
i.e., this is an automorphism on the group C.

Theorem : The set AG of all automorphisms of a group G with


composite of functions as binary operation is a group.
Proof : Let A G be the set of all automorphisms on G and f, g ∈ AG,
then f, g ∈ AG ⇒ f and g are one-one mappings of G onto itself
⇒ gof is also a one-one mapping of G onto itself.
Further if a, b ∈ G, then
(gof)(ab) = g[f(ab)]
= g[f(a) f(b)] [∵ f is automorphism]
= g[f(a)] g[f(b)] [∵ g is automorphism]
= (gof)(a)(gof)(b)
∴ gof is an automorphism of G.
Therefore f ∈ AG, g ∈ AG ⇒ gof ∈ AG
i.e., AG is closed for the operation of composite of functions.

Verification of Group axioms in A G :


[G1] Since the composite of mappings is always is always
associative, so this is also associative in A G i.e., (fog)oh = fo(goh),
∀ f, g, h ∈ AG.
[G2] Since the identity mapping l G of G is an automorphism such
that for every f ∈ AG, lG of = f = f o l G
∴ lG is the identity element of A G
[G3] Let f ∈ AG, then
f ∈ AG ⇒ f is a bijection on G.
Again, for any a, b ∈ G, let
f-1(a) = a 1 and f-1(b) = b1 ∀ a1, b1 ∈ G.
⇒ a = f(a1) and b = f(b 1)
Now f-1 (ab) = f -1 [f(a1) f(b1)]
= f-1 [f(a1 b1)] [∵ f is automorphism]
= a1 b1 = f-1 (a) f-1 (b)
∴ f-1 is also an automorphism of G.
Hence f-1 ∈ AG, Thus the inverse for each f exists in A G.
From the above discussion, A G is also a group for the composite of
functions.

Example : If a be an element of a group G, then the mapping


fa : G → G, f a(x) = axa -1, ∀ a ∈ G is an automorphism of g.

Let x1, x2 ∈ G, then


fB(x1) = fa(x2) ⇒ aax 1a-1 = ax2a-1
⇒ x1 = x2 [by cancellation law]
∴ fa is one-one.
Again for every x ∈ G, there exists a -1 xa ∈ G such that
fa(a-1 xa) = a(a -1 xa)a-1 = (aa-1)x(aa-1) = exe = x.
∴ fa is onto.
Lastly, we see that for any x 1, x2 ∈ G
fa(x1 x2) = a(x1 x2)a-1
= (ax1 a-1)(ax2 a-1) = fa(x1) fa(x2)
∴ fa is an automorphism on the group G.
Remark : The above automorphism f a is called the Inner
automorphism of the group G determined by a.
Example : In a group G, show that, the mapping f : G → G,
defined by f(x) = x -1, ∀ x ∈ G is an automorphism iff G is
abelian.

(⇒) Suppose that G is an abelian group and a, b ∈ G. Then


f(a) = f(b) ⇒ a-1 = b-1 (a-1)-1 = (b-1)-1 ⇒ a = b.
∴ f is one-one and for every a ∈ G there exists a -1 ∈ G such
that
f(a-1) = (a-1)-1 = a.
∴ f is onto.
Again, f(ab) = (ab) -1 = b-1 a-1 = a-1 b-1  [∵ G is cumulative]
= f(a) f(b).
∴ f is an isomorphism on G i.e., this is an automorphism on G.
Conversely (⇐) : Suppose that f is an automorphism of G,
then for any a, b ∈ G,
f(ab) = (ab) -1 [by definition of f]
= b-1 a-1 = f(b) f(a)
= f(ba) [∵ f is an automorphism]
Now, since f is one-one, therefore
f(ab) = f(ba) ⇒ ab = ba
⇒ G is an abelian group.

FACTOR GROUPS

We have already seen that if  , then every left (right) coset of H
is equal to its corresponding right (left) coset in G. Again the
product of any two right (left) cosets of H is also a right (left) coset
of H. This leads to a natural definition of a binary composition in the
set of all cosets of H in G. As such the set of all left cosets of H
coincide with the set of all right cosets of H. In the next theorem,
we shall prove that this set is a group for the natural binary
composition. This group is called the Quotient group or Factor
group of G by H and is denoted by G/H.

Theorem : The set of all cosets of   is a group with respect to
multiplication of cosets defined as follows :
Ha Hb = Hab, a b ∈ G
Proof : Binary composition is well defined :
Let G/H be the set of all cosets of H in G.
i.e. G/H = {Ha | a ∈ G} = {aH | a ∈ G}.
∵ a ∈ G, b ∈ G ⇒ ab ∈ G ⇒ Hab ∈ G/H.
∴ Ha ∈ G/H, Hb ∈ G/H ⇒ HaHb = Hab ∈ G/H.
Therefore G/H is closed for the defined binary operation.

Verification of group axioms in G/H :

G1 : Let Ha, Hb, Hc ∈ G/H; a, b, c ∈ G, then


(HaHb)Hc = HabHc = H(ab)c
= Ha(bc) [by associativity in G]
= Ha Hbc = Ha(HbHc)
∴ the composition is associative in G/H.
G2 : H = He ∈ G/H is the identity element because for Ha ∈ G/H
HaHe = Hae = Ha
and HeH = Hea = Ha
G3 : We see that
Ha ∈ G/H ⇒ Ha-1 ∈ G/H [∵ a ∈ G ⇒ a-1 ∈ G]
and HaHa -1 = Haa-1 = He = H
Ha-1 Ha = Ha-1 a = He = H
∴ Ha-1 is the inverse of Ha in G/H.
Thus every element of G/H is invertible.
Therefore G/H is a group for the defined composition.
On the basis of the above theorem, the quotient group may also be
defined as follows :

Quotient group

Let G be a group and   then the set G/H of all cosets of H in G


together with the binary composition defined by
HaHb = Hab, where Ha ∈ G/H, Hb ∈ G/H is a group, and is called
the quotient group of G by H.

Remarks :
1. If the composition in the group is addition (+), then the
composition in G/H is defined as (H + a) + (H + b) = H + (a +
b).
2. For the existence of the quotient group G/H, it is necessary
that H is a normal subgroup of G.

Theorem : If N is a normal subgroup of a finite group G, then :

Proof : By definition,


O(G/N) = the number of different right (left) cosets of N in G.
= the index of N is G
= No. of elements in G/No. of elements in N [by Lagrange’s
theorem]
= O(G)/O(N)

Properties of Quotient group

Theorem : Every quotient group of an abelian group is abelian but


not conversely.
Proof : Let H be a normal subgroup of an abelian group G and a ∈
G, b ∈ G, then
a ∈ G, b ∈ G ⇒ Ha ∈ G/H, Hb ∈ G/H
∵ HaHb = Hab
= Hba [∵ G is commutative ⇒ ab = ba]
= HbHa
Thus, we see that
Ha ∈ G/H, Hb ∈ G/H ⇒ HaHb = HbHa
∴ G/H is also commutative.
Conversity (⇐) : The converse is not necessarily true.

For example : S3/A3 is an abelian group, while S 3 is a non abelian


group. The order of group S 3/ A3 is 2 and every group of order 2 is
an abelian.
Theorem : Every quotient group of a cyclic group is cyclic but not
conversely.
Proof : Let H be a normal subgroup of any cyclic group G = [a].
Since every element of G is of the form a n, n ∈ Z
therefore let Ha n ∈ G/H then
Han = H(a.a...(n times))
= HaHa...Ha (n times) [∵ Hab = HaHb]
= (Ha)n
∴ G/H = [Ha]
Therefore G/H is also a cyclic group whose generator is Ha.

Conversely : The converse is not necessarily true.


For example : S3/A3, being a group of order 2, necessarily cyclic
but S 3 is not cyclic group.

Example : Find the quotient group G/H and also prepare its
operation table when :
G = ({1, -1, i, -i},*)
H = ({1, -1}, *)

Since G is a commutative group, therefore 


Consequently, G/H exist having the following cosets of H:
H.1 ={1.1, -1.1} = {1,-1} = H
H(-1) = {1.(-1), -1 -(-1)} = {-1.1} = H
H.i. ={1.i, -1, i} = {i, -i] = Hi  
H(-i) = {1 -(-i), -1 (-i))} = (-1, i} = Hi
Thus we see that G/H = {H, Hi}
The composition table of G/H is as follows :

Example : Find the quotient group G/H and also prepare its
operation table when G =  , H = 

Since    is a commutative group, therefore 


Hence G/H exists.
The cosets of H in G are as follows :
0+ H = H + 0 = {..., -8, -4, 0, 4, 8, ...} = H
1 + H = H + 1 = {..., -7, -3, 1, 5, 9, ...}
2 + H = H + 2 = {..., -6, -2, 2, 6, 10, ...}
3 + H = H + 3 = {..., -5, -1, 3, 7, 11, ...}
We further observe that
H = H + 4 = H + 8 = H + 12 = ... = H + (-4) = H + (-8) = ...
H + 1 = H + 5 = H + 9 = H + 13 = ... = H + (-3) = H + (-7) = ...
H + 2 = H + 6 = H + 10 = H + 14 = ... = H + (-2) = H + (-6) = ...
H + 3 = H + 7 = H + 11 = H + 15 = ... = H + (-1) = H + (-5) = ...
Thus H has four distinct cosets and so
G/H = {H, H + 1, H + 2, H + 3}
The composition table of G/H is as shown below :

When the subgroup H of G is normal, then the set of left (or right)
cosets of H in G is itself a group-called the factor group of G by H
(or the quotient group of G by H).

Theorem : Factor Group

Let G be a group and H a normal subgroup of G. The set G/H = {aH


| a ∈ G} is a group under the operation (aH)(bH) = abH.
Proof : Our first task is to show that the operation is well defined;
that is, we must show that the correspondence defined above from
G/H x g/H into G/H is actually a function. To do this, assume that
aH = a’H and bH = b’H. Then a’ = ah 1 and b’ = bh 2 for some h 1,
h2 in H, and therefore a’b’H = ah 1bh2H = ah1bH = ah 1Hb = aHb =
abH , eH = H is the identity; a -1 H is the inverse of aH; and
(aHbH)cH = (ab)HcH = (ab)cH = a(bc)H = aH(bc)H = aH(bHcH).
This proves that G/H is a group.

Example : Let G =   and H = <6> = {0, 6, 12}. Then G/H = {0 + H,


1 + H, 2 + H, 3 + H, 4 + H, 5 + H}. To illustrate how the group
elements are combined, consider (5 + H) + (4 + H). This should be
one of the six elements listed in the set G/H. Well, (5 + H) + (4 + H)
= 5 + 4 + H = 9 + H = 3 + 6 + H = 3 + H, since H absorbs all
multiples of 6.

Example : A4 has no subgroup of order 6.


The group A 4 of even permutations on the set {1, 2, 3, 4} has no
subgroup H of order 6. To see this, suppose that A 4 does have a
subgroup H of order 6. We know that   Thus, the factor group
A4/H exists and has order 2. Since the order of an element divides
the order of the group, we have (αH) 2 = α2H = H for all α ∈ A4.
Thus, a 2 ∈ H for all α in A 4. However, we observe that A 4 has nine
different elements of the form α 2, all of which must belong to H, a
subgroup of order 6. This is clearly impossible, so a subgroup of
order 6 cannot exist in A 4.

Theorem : The   Theorem


Let G be a group and let   be the center of G. If   is cyclic,
then G is Abelian.
Proof. Suppose that G/Z(G) is cyclic. Then there is some aZ(G) ∈
G/Z(G) such that <aZ(G)> = G/Z(G).
Let x, y ∈ G We wish to show that xy = yx. It follows that there exist
integers j, k such that xZ(G) = (aZ(G)) i and yZ(G) = (aZ(G)) k. So
there exist z 1, z2 ∈ Z(G) such that x = a jz1 and y = ak z2. Now
consider their product:
xy = (a j z1)(ak z2)
= aj (z1ak)z2
= (aj ak)(z1z2)
= (ak aj)(z2z1)
= (ak z2)(ajz1) = yx
Note that in the third line above we are able to commute a k and
z1 since z1 ∈ Z(G).
Similarly we are able to commute from line four to five.
Thus, we have shown that G is Abelian.

Theorem :  (G) ≈ Inn(G)


Let G be a group. For g ∈ G, let Tg : G → G be the automorphism
(h)Tg = g-1hg. Let Inn(G) = {T g : g ∈ G}. This is a subgroup of Aut(G)
called the group of inner automorphisms. Show that Inn(G) is
isomorphic to G/Z(G) where Z(G) = {g ∈ G : gh = hg for every h ∈
G} is the center of G.

OR
For any group G,  (G) is isomorphic to Inn(G).
Proof. The map T : G → Aut(G) which sends g to T g is a
homomorphism with image Inn(G) and kernel Z(G). This implies
Inn(G) ≈ G/Z(G) as claimed.

Theorem : Existence of Elements of Prime Order

Let G be a finite Abelian group and let p be a prime that divides the
order of G. Then G has an element of order p.
Proof. Our proof uses two facts about finite cyclic groups. If G is
cyclic and p divides IGI then G has an element of order p since G
has exactly one (cyclic) subgroup for every divisor of |G|. If G =
<a> has order m and a n = e then min.
We proceed by induction on |G|. The case |G| = 1 is vacuous since
p does not divide |G| in this case. Suppose m ≥ 1 and that theorem
holds for all abelian groups of order less than or equal to m. Let G
be an abelian group such that |G| ≤ m + 1 and suppose that p
divides |G|. Then |G| > 1 so we may chose an a ∈ G with a ≠ e. If p
divides |<a>| then <a>, hence G, has an element of order p.
Suppose p does not divide |<a>|. Since G is abelian H = <a> ≥ G.
Since |G| = |G/H||H| and |H| > 1 it follows that p divides |G/H| and |
G/H| < |G|. Since G/H is abelian, by our induction hypothesis there
is an element bH ∈ G/H or order p. Let n = |<b>|. Then (bH) n = bnH
= eH = H from which we deduce pin. Thus <b> has an element of
order p.

Theorem : If N is normal subgroup of a finite group G,

then 
Proof : By definition, O(G/N) = the number of different right (left)
cosets of N in G.
= the index of N is G
= (No. of elements in G/No. of elements in N)(O(G)/O(N)) [by
Lagrange’s theorem]

Properties of Quotient Group


Every quotient of an abelian group is abelian but not conversely.

Every quotient group of a cyclic group is cyclic but not conversely.


For example S 3/A3 being a group of order 2 necessarily cyclic but
S3 is not cyclic group.

GROUP ACTION

An action of a group G on a set S is a rule for combining an


element g of G and an element s of S to get another element of S.
In other words, it is a map G x S → S. For the moment we denote
the result of applying this law to elements g and s by g*s. An action
is required to satisfy the following axioms:

(a) id * s = s for all s in S. (Here id is the identity of G.)


(b) associative law: (gg’) *s = g*(g’*s), for all g and g’ in G and all s
in S.
Given an operation of a group G on a set S, an element s of S will
be sent to various other elements by the group operation. We
collect together those elements, obtaining a subset called the orbit
O3 of s:
Os = {s’ ∈ S | s’ = gs for some g in G}.
The orbits for a group action are equivalence classes for the
equivalence relation s ~ s’ if s’ = gs, for some g in G.
So, if s ~ s’, that is, if s' = gs for some g in G, then the orbits of s
and of s’ are the same.
Since they are equivalence classes the orbits partition the set S.
The stabilizer of an element s of S is the set of group elements that
leave s fixed. It is a subgroup of G that we often denote by G 3:
G3 = {g ∈ G | gs = s}.
Example: Let G be the group of nonzero real numbers under
multiplication, and let S be the set of all vectors in 3-space. Thus S
= {(a; b; c) | a, b, c ∈ R}. Then G acts on S via scalar multiplication.
That is, g(a; b; c) = (ga; gb; gc) if g is a nonzero real number. Then
for any vector v, we have 1* v = v and if g, h 2 G then (gh)v =
g(h*v), since if v = (a; b; c) then (gh)v = (gha; ghb; ghc) = g(ha; hb;
he) = g(hv).

Example : Let G be a group and S = G. Then G acts on S by left


multiplication. That is, gs is defined to be the ordinary product of g
and s in G. Associativity of multiplication in G and properties of the
identity then show this is an action of G on S. This example leads
to the proof of Cayley’s theorem.

Example : Conjugation, the operation of G on itself defined by (g,


x) → gxg-1. The stabilizer of an element x of G for the operation of
conjugation is called the centralizer of x. The orbit of x for
conjugation is called the conjugacy class of x, and is often denoted
by Cl(x). It consists of all of the conjugates gxg -1:
Cl(x)= {x’ ∈ G | x’ = gxg -1 for some g in G}.

Note :

 The centralizer C(x) of an element x of G contains x, and it


contains the center Z(G).
 An element x of G is in the center if and only if its centralizer
C(x) is the whole group G, and this happens if and only if the
conjugacy class Cl(x) consists of the element x alone.

Since the conjugacy classes are orbits for a group operation, they
partition the group. This fact gives us the class equation of a finite
group:

If we number the conjugacy classes, writing them as C 1,........... C k,


the class equation reads Order(G) = cardinality(C 1) + cardinality(C 2)
+ . . . + cardinality(C k)
The numbers on the right side of the class equation divide the
order of the group, and at least one of them is equal to 1 (∵ id. Has
only 1 conjugate).
Example : In S3 id. has 1 conjugate. (1 2), (1 3), (2 3) are
conjugates to each other (Since they have same cycle type) and (1
2 3) & (1 3 2) are conjugates. Thus, class equation of S 3 = 1 + 2 +
3.

p-groups:
The class equation has several applications to groups whose
orders are positive powers of a prime p. They are called p-groups.

Theorem: The center of a p-group is not the trivial group.

Proof: Say that order(G) = p e with e > 1. Every term on the right


side of the class equation divides p e, so it is a power of p too,
possibly p° = 1. The positive powers of p are divisible by p. If the
class C1 of the identity made the only contribution of 1 to the right
side, the equation would read p e = 1 + ∑(multiples of p).
This is impossible, so there must be more 1’s on the right. The
center is not trivial.

Theorem: Every group of order p 2 is abelian.


Proof: Let G be a group of order p 2. According to the previous
theorem, its center Z is not the trivial group. So, the order of Z must
be p or p2. If the order of Z is p 2, then Z = G, and G is abelian as
the proposition asserts. Suppose, that the order of Z is p, and let x
be an element of G that is not in Z. The centralizer C(x) contains x
as well as Z, so it is strictly larger than Z. Since |Z(x)| divides |G|, it
must be equal to p 2, and therefore Z(x) = G. This means that x
commutes with every element of G, so it is in the center after all,
which is a contradiction. Therefore, the center cannot have order p.
Simple group :
A group G is simple if it is not the trivial group and if it contains no
proper normal subgroup- no normal subgroup other than <1> and
G.

Example: An is a simple group ∀n ≥ 5. Any group of prime order is


simple.

Sylow’s thorem :
Let G be a group of order n, and let p be a prime integer that
divides n. Let p e denote the largest power of p that divides n, so
that n = p em where m is an integer not divisible by p.
A Sylow p-subgroup is a p-group whose index in the group isn’t
divisible by p.

Theorem 1: A finite group whose order is divisible by a prime p


contains a Sylow p-subgroup.

Theorem 2: Let G be a finite group whose order is divisible by a


prime p.
(a) The Sylow p-subgroups of G are conjugate subgroups.
(b) Every subgroup of G that is a p-group is contained in a Sylow p-
subgroup.

Theorem 3: Let G be a finite group whose order n is divisible by a


prime p. Say that n = p em, where p does not divide m, and let
np denote the number of Sylow p-subgroups. Then n p divides m and
np is congruent to 1 modulo p: n p = kp + 1 for some integer k > O.
Further, np is the index in G of the normalizer N G(H) for any Sylow
p-subgroup H, hence n p divides m.
Corollary: A group G has just one Sylow p-subgroup H if and only
if that subgroup is normal.

Example: Let G be a group of order 35. According to the Third


Sylow Theorem, the number of its Sylow7-subgroups divides 5 and
is congruent 1 modulo 7. The only such integer is 1. Therefore,
there is one Sylow 7-subgroup, say H, and it is a normal subgroup.
For similar reasons, there is just one Sylow 5-subgroup, say K, and
it is normal. The subgroup H is cyclic of order 7, and K is cyclic of
order5. The intersection H ∩ K is the trivial group. Then G is
isomorphic to the product group H X K. So, all groups of order 35
are isomorphic.to the product C 7 X C5 of cyclic groups and to each
other. The cyclic group of order 35 is one such group, so all groups
of order 35 are cyclic.

You might also like